You are on page 1of 83

MyPasTest: MRCS A Online - Jan Exam 2015

22. Surgical Specialities; Trauma & Orthopaedic Surgery (76Qs)


----------------------------------------------------------------------------------------------------------------------------------
01. Theme: Treatment options in fracture management
A Broad arm sling (polysling)
B Cast-brace
C Cerclage wires
D Dynamic screw fixation
E External fixation
F Hanging cast Page |
G Hemiarthroplasty 1305
H Intramedullary nailing
I K-wires
J Plaster cast
K Plate and screw fixation
L Screw fixation
M Tension band wiring
N Traction.

All of the above are employed in the management of fractures. For the following fractures please choose
the most appropriate method of fracture fixation from the list. Each item may be used once, more than
once, or not at all.

Scenario 1
A closed, two-part fracture to the middle third of the clavicle (low-impact mechanism).
A - Broad arm sling (polysling)« CORRECT ANSWER.

A – Broad arm sling:


Fractures of the clavicle, despite representing 5% of all fractures (and 44% of shoulder girdle fractures),
seldom excite much interest. They are usually treated conservatively in a broad arm sling (or polysling),
although surgical fixation may be indicated. Operative treatment is reserved for those patients with: open
fractures, polytrauma, neurovascular injury (NB proximity of brachial plexus), compromise of the
overlying skin, floating shoulder, symptomatic non-union and fractures of the lateral third proximal to, or
between, the conoid and trapezoid ligaments.

Scenario 2
An inter-trochanteric fracture to the left neck of the femur in a 75-year-old woman.
B - Cast-brace« YOUR ANSWER
D - Dynamic screw fixation« CORRECT ANSWER.

D – Dynamic screw fixation:


For an extracapsular fracture, where blood supply to the femoral head is
not significantly compromised (such as that described in this scenario), the
ideal method of fixation is with dynamic screw fixation, specifically a
dynamic hip screw. This is a plate and sliding screw fixator that permits
compression at the fracture site. It allows good anatomical fixation of the
fracture and early mobilisation of the patient.

Scenario 3
An isolated femoral shaft fracture in a 4-year-old boy.
C - Cerclage wires« YOUR ANSWER
---------------------------------------------------------------------------------------------------------------------------------------------------------------------------------------------------------------
Dr Mohammed Shamsul Islam Khan, Medical Officer, Clinical Neuro-Surgery, National Institute of Neuro-Sciences and Hospital
Sher-E-Bangla Nagar, Dhaka-1207, Bangladesh. Mobile: +880 1713 455 662, +880 1685 811979. E-mail: drsikhan@gmail.com
MyPasTest: MRCS A Online - Jan Exam 2015
22. Surgical Specialities; Trauma & Orthopaedic Surgery (76Qs)
----------------------------------------------------------------------------------------------------------------------------------
N - Traction« CORRECT ANSWER.

N – Traction:
Paediatric femoral shaft fractures are commonly treated by skin or skeletal traction. This allows fracture
union before the child then commences mobilisation in an appropriate cast. It is important to note that
fixation of fractures in young children can disturb bone growth (particularly intramedullary nailing
through an epiphyseal growth plate), leading to shortening and malformation of the affected limb. Hence Page |
it is restricted to the management of the polytraumatised child when plate fixation or external fixation may 1306
be used with care.

Scenario 4
A Gustilo III comminuted tibial fracture in a 35-year-old man.
D - Dynamic screw fixation« YOUR ANSWER
E - External fixation« CORRECT ANSWER.

E – External fixation:
Indications for external fixation in trauma encompass: compound (open) long bone fractures with
extensive tissue devitalisation (especially of the tibia), closed fractures with degloving skin injuries, ‘open
book’ pelvic fractures, polytrauma, peri-articular and metaphyseal fractures. The unique characteristics of
external fixation include: rapid skeletal stabilisation using connecting frames and percutaneous pins,
remote from the site of injury; versatility (different injuries with differing anatomy); ability to adjust
alignment and fixation during fracture healing; and ease of access to surrounding soft tissues.

02. Theme: The painful knee


A Anterior cruciate injury
B Chondromalacia patellae
C Infrapatellar bursitis
D Lateral collateral injury
E Medial collateral injury
F Meniscal tear
G Osgood–Schlatter disease
H Osteoarthritis
I Osteochondritis dissicans
J Pre-patellar bursitis
K Recurrent dislocation of the patella
L Rheumatoid arthritis
M Septic arthritis
N Tendinitis.

The following are descriptions of patients with a painful knee(s). Please select the most appropriate
diagnosis from the above list. The items may be used once, more than once, or not at all.

Scenario 1
A 43-year-old woman is seen with a history of chronic pain and swelling of the knee. On examination, flexion and
extension of the knee are limited and a marked valgus deformity is noted. This is particularly apparent on
standing.
A - Anterior cruciate injury« YOUR ANSWER
L - Rheumatoid arthritis« CORRECT ANSWER
---------------------------------------------------------------------------------------------------------------------------------------------------------------------------------------------------------------
Dr Mohammed Shamsul Islam Khan, Medical Officer, Clinical Neuro-Surgery, National Institute of Neuro-Sciences and Hospital
Sher-E-Bangla Nagar, Dhaka-1207, Bangladesh. Mobile: +880 1713 455 662, +880 1685 811979. E-mail: drsikhan@gmail.com
MyPasTest: MRCS A Online - Jan Exam 2015
22. Surgical Specialities; Trauma & Orthopaedic Surgery (76Qs)
----------------------------------------------------------------------------------------------------------------------------------
L – Rheumatoid arthritis:
This can occasionally start in the knee as a monoarticular synovitis. With chronicity, the joint may
become increasingly deformed. Although deformity can also occur (with chronic pain and swelling) in
osteoarthritis, a valgus deformity is characteristic of rheumatoid arthritis whereas a varus deformity is
frequently seen with severe osteoarthritis.

Scenario 2 Page |
A 14-year-old girl is seen with a 3-month history of knee pain. There is no history of trauma. The pain is felt 1307
principally in front of the knee and is exacerbated on ascending and descending stairs.
B - Chondromalacia patellae« CORRECT ANSWER.

B – Chondromalacia patellae:
Softening of the articular cartilage of the patella is often associated with anterior knee pain in teenage
girls. The exact aetiology is unknown, however, it is thought to result from overload of the patellar
articular surface as a result of mal-tracking of the patella during flexion and extension. On clinical
examination, pain can be elicited by the patella friction test. Treatment is rest, analgesia and
physiotherapy.

Scenario 3
A 19-year-old man presents with a history of intermittent pain and swelling in his left knee. In addition, he
complains of his knee locking, which he relieves by manoeuvring the leg. He also complains of his knee „giving
way.‟ There is no history of trauma. On examination a small effusion is noted and a small mobile „body‟ is felt in
the suprapatellar pouch.
C - Infrapatellar bursitis« YOUR ANSWER
I - Osteochondritis dissicans« CORRECT ANSWER.

I – Osteochondritis dissicans:
This is a condition where a small osteocartilaginous fragment separates from one of the femoral condyles
(usually the medial condyle) and is rendered avascular. Patients tend to be young and present with
intermittent pain and swelling of the knee. Attacks of ‘locking’ may occur as the loose body becomes
trapped between the joint surfaces. Between attacks the loose body may be palpable, particularly in the
suprapatellar pouch. Treatment involves removal of the loose body if small. Large fragments may be fixed
back into position, particularly if complete separation has not occurred.

03. Theme: Common fracture eponyms


A Barton’s fracture
B Bennett’s fracture
C Colles’ fracture
D Galeazzi’s fracture dislocation
E Garden II fracture
F Garden III fracture
G Garden IV fracture
H Hill–Sachs fracture
I Lisfranc fracture dislocation
J Monteggia’s fracture dislocation
K Rolando’s fracture
L Smith’s fracture
M Weber A fracture
---------------------------------------------------------------------------------------------------------------------------------------------------------------------------------------------------------------
Dr Mohammed Shamsul Islam Khan, Medical Officer, Clinical Neuro-Surgery, National Institute of Neuro-Sciences and Hospital
Sher-E-Bangla Nagar, Dhaka-1207, Bangladesh. Mobile: +880 1713 455 662, +880 1685 811979. E-mail: drsikhan@gmail.com
MyPasTest: MRCS A Online - Jan Exam 2015
22. Surgical Specialities; Trauma & Orthopaedic Surgery (76Qs)
----------------------------------------------------------------------------------------------------------------------------------
N Weber B fracture
O Weber C fracture.

The following are descriptions of fractures. Please select the most appropriate fracture eponym from the
above list. Each item may be used once, more than once, or not at all. These are all commonly used in
current clinical practice (and so remain important).
Page |
Scenario 1 1308
A comminuted, intra-articular fracture to the base of the first metacarpal.
A - Barton‟s fracture « YOUR ANSWER
K - Rolando‟s fracture« CORRECT ANSWER.

K – Rolando’s fracture:
This description could be one of a Bennett’s fracture, but the comminuted nature distinguishes it as a
Rolando’s fracture. There are usually three fragments forming either a Y-shape or a T-shape on
radiograph.

Scenario 2
A distal fibular fracture at the level of the syndesmosis, with or without a malleolar fracture.
B - Bennett‟s fracture « YOUR ANSWER
N - Weber B fracture« CORRECT ANSWER.

N – Weber B fracture:
The Weber (or Danis–Weber) classification describes the severity of tibio- fibular ligament injury by the
level of fibular fracture. This is a Weber B fracture as it is sited at the level of the syndesmosis. A Weber A
fracture is infra-syndesmotic while a Weber C fracture occurs above the level of the syndesmosis. The
Lauge–Hansen classification is an alternative method of describing fractures of the distal tibia and fibular.
It takes into account foot position and direction of deforming forces, and is preferred by senior
orthopaedic surgeons. For your purposes the Weber system is sufficient as the Lauge–Hansen is
complex and not all fractures fit the classical pattern.

Scenario 3
A complete fracture through the femoral neck, with rotation of the femoral head within the acetabulum,
demonstrating minimal displacement.
C - Colles‟ fracture « YOUR ANSWER
F - Garden III fracture « CORRECT ANSWER.

F – Garden III fracture:


The Garden classification is used to describe intra-capsular fractures of the neck of the femur. It is
important to distinguish these from extra-capsular fractures, as there is a bearing on blood supply, and
ultimately, treatment. The capsule contributes the majority of the blood supply to the head of the femur,
via the medial and lateral circumflex arteries from the profunda femoris. A compromise in the blood
supply can result in avascular necrosis.

The Garden system consists of four grades (I–IV) as follows:


1. Garden I – incomplete or impacted fracture
2. Garden II – non-displaced fracture through both cortices
3. Garden III – minimally displaced fracture with rotation of the femoral head in the acetabulum
---------------------------------------------------------------------------------------------------------------------------------------------------------------------------------------------------------------
Dr Mohammed Shamsul Islam Khan, Medical Officer, Clinical Neuro-Surgery, National Institute of Neuro-Sciences and Hospital
Sher-E-Bangla Nagar, Dhaka-1207, Bangladesh. Mobile: +880 1713 455 662, +880 1685 811979. E-mail: drsikhan@gmail.com
MyPasTest: MRCS A Online - Jan Exam 2015
22. Surgical Specialities; Trauma & Orthopaedic Surgery (76Qs)
----------------------------------------------------------------------------------------------------------------------------------
4. Garden IV – completely displaced fracture of the head of femur (no continuity between the
proximal and distal fragments).
5.
Scenario 4
An intra-articular fracture of the volar or dorsal margin of the distal radius. The fracture extends obliquely to the
radio-carpal joint with a striking dislocation of the carpus.
D - Galeazzi‟s fracture dislocation « YOUR ANSWER Page |
A - Barton‟s fracture « CORRECT ANSWER. 1309
A – Barton’s fracture:
This injury results from a fall onto an outstretched hand. The Barton fracture can be sub-divided into
volar and dorsal types. It can be distinguished radiographically from Colles’ or Smith’s fractures by the
presence of a dislocation or subluxation. This involves the rim of the distal radius, which can be dorsally
or volarly displaced with the hand and carpus. The majority of these fractures require surgical reduction
and fixation.

04. Theme: Mono- and polyarthritis


A Behçet’s syndrome
B Drug allergies
C Gout
D Osteoarthritis
E Pseudogout
F Psoriasis
G Reiter’s syndrome
H Rheumatoid arthritis
I Septic arthritis
J Spondyloarthritides
K Still’s disease
L Systemic lupus erythematosus
M Trauma
N Viral illness.

The following patients all present with symptoms of arthritis. From the list above, select the most likely
diagnosis. The items may be used once, more than once, or not at all.

Scenario 1
A 43-year-old man attends The Emergency Department with a painful right knee. He reports a sudden onset of
severe, constant pain in the right knee, and that he is no longer able to mobilise. His temperature is 38.5°C.
Examination of the knee reveals an increased temperature over the right knee, but no erythema. He is tender in
the joint line, and he is unable to actively move the joint.
A - Behçet‟s syndrome « YOUR ANSWER
I - Septic arthritis« CORRECT ANSWER.

I – Septic arthritis:
Joints can become infected by direct extension from a wound, by direct introduction (joint
injection/arthroscopy), or by spread from acute osteomyelitis or haematogenously. Staphylococci and
Haemophilus influenzae predominate, although other Gram-positive cocci and Gramnegative bacilli may
be implicated.The infection usually starts in the synovial membrane, and a seropurulent exudate develops
---------------------------------------------------------------------------------------------------------------------------------------------------------------------------------------------------------------
Dr Mohammed Shamsul Islam Khan, Medical Officer, Clinical Neuro-Surgery, National Institute of Neuro-Sciences and Hospital
Sher-E-Bangla Nagar, Dhaka-1207, Bangladesh. Mobile: +880 1713 455 662, +880 1685 811979. E-mail: drsikhan@gmail.com
MyPasTest: MRCS A Online - Jan Exam 2015
22. Surgical Specialities; Trauma & Orthopaedic Surgery (76Qs)
----------------------------------------------------------------------------------------------------------------------------------
in the synovial fluid. There is progressive destruction of the articular cartilage, and vascular damage may
lead to death of epiphyseal bone; hence the need for urgent diagnosis and appropriate treatment. The
classic clinical feature is reluctance to move the joint, but swelling and erythema are commonly present.
The joint is the maximal site of tenderness (differentiating it from acute osteomyelitis), and all
jointmovements are restricted. White cell count and erythrocyte sedimentation rate/C-reactive protein are
invariably raised. Radiography is often unremarkable in the early stages. Treatment involves analgesia,
appropriate antibiotics, joint aspiration, or occasionally formal open drainage. Page |
1310
Scenario 2
A 52-year-old woman attends the clinic with a long history of intermittent pain and swelling affecting her left knee.
Past history includes hypothyroidism. Examination is unremarkable. A plain radiograph reveals the presence of
intra-articular calcium deposition, but no other abnormality.
B - Drug allergies « YOUR ANSWER
E - Pseudogout « CORRECT ANSWER.

E – Pseudogout:
This condition involves acute or chronic arthritis secondary to deposition of calcium pyrophosphate or
basic calcium phosphates. Pyrophosphate is generated in cartilage by enzyme activity, and combines
with calcium ions to form crystals. In many cases it is idiopathic, although it may occur in
hyperparathyroidism, hypothyroidism, acromegaly, or haemochromatosis. The clinical presentation is
less severe and usually more chronic than with gout (urate crystal arthropathy in patients with
hyperuricaemia). The fibrocartilage of the knee, pubic symphysis and intervertebral discs are most
commonly affected, particularly in women over the age of 60 years. Radiography reveals calcification that
usually appears as a line across the joint, but the synovial capsule and surrounding tendons may be
calcified. The diagnosis is confirmed by demonstrating weakly positive bi-refringent rhomboid crystals in
plane-polarised light.

Scenario 3
A 13-year-old girl is referred with a 6-month history of pain and swelling affecting the joints of her upper limbs. Her
mother also informs you that she also suffers with a „grumbling appendix‟. On examination, there is bilateral
involvement of the shoulder joints, elbows and wrists. Abdominal examination reveals splenomegaly.
C - Gout « YOUR ANSWER
K - Still‟s disease« CORRECT ANSWER.

K – Still’s disease:
Juvenile chronic arthritis (Still’s disease) affects 1 in 1000 children, with 70% of those affected being
female. At least two of joint pain, swelling, or limitation of movement, need to affect more than four joints
for at least 3 months for the diagnosis to be accepted. Associated systemic symptoms of fever, macular
rash and lymphadenopathy may predominate. Recurrent abdominal pain may occur secondary to bouts of
mesenteric adenitis, as in the case presented. Hepatosplenomegaly, myocarditis and uveitis may
complicate this condition. In 90% of cases the condition is seronegative and can only be confirmed by
synovial biopsy. Chronic disease may lead to joint destruction, fibrosis and ankylosis, resulting in
deformity. In addition, end-organ failure may supervene. It is differentiated from Reiter’s syndrome
(polyarthritis, urethritis, conjunctivitis) on account of no urethral involvement.

05. Which of the following is a common childhood fracture (under 12 years)?


Barton‟s fracture« YOUR ANSWER
---------------------------------------------------------------------------------------------------------------------------------------------------------------------------------------------------------------
Dr Mohammed Shamsul Islam Khan, Medical Officer, Clinical Neuro-Surgery, National Institute of Neuro-Sciences and Hospital
Sher-E-Bangla Nagar, Dhaka-1207, Bangladesh. Mobile: +880 1713 455 662, +880 1685 811979. E-mail: drsikhan@gmail.com
MyPasTest: MRCS A Online - Jan Exam 2015
22. Surgical Specialities; Trauma & Orthopaedic Surgery (76Qs)
----------------------------------------------------------------------------------------------------------------------------------
Colles‟ fracture
Fracture of the femoral shaft
Hairline fracture of temporal bone
Supracondylar fracture« CORRECT ANSWER.
Page |
The skull and long bones in young children are still ossifying and therefore more malleable than in adults. 1311
Fractures to these areas in young children should therefore be treated with suspicion and non-accidental
injury considered. Colles’ fractures are classical, although not exclusive, to osteoporosis and therefore
elderly adults. Barton’s fractures are intra-articular fractures of the distal radius in adults, similar
fractures in children would be classified using the Salter Harris classification system.

06. Theme: Lower limb trauma


A Anterior compartment syndrome
B Posterior compartment syndrome
C Venous bleed
D Traumatic nerve damage
E Deep venous thrombosis
F Lateral compartment syndrome.

For each of the patients described below, select the single most likely diagnosis from the options listed
above. Each option may be used once, more than once, or not at all.

Scenario 1
A motorcyclist rides into a lamppost and is unable to bear weight on his left leg. Lachman‟s test is positive, there is
no bony injury on X-ray, passive dorsiflexion of the ankle causes considerable pain and the foot is cold and numb
on palpation.
A - Anterior compartment syndrome« YOUR ANSWER
B - Posterior compartment syndrome« CORRECT ANSWER.

This patient is likely to have posterior compartment syndrome, as the muscles of the posterior
compartment are stretched when the foot is passively dorsiflexed, thus eliciting pain.

Scenario 2
A patient suffers a displaced tibial fracture following a road traffic accident, is admitted and intramedullary nailing
is done. You are on call and are asked to see him after he complains of severe pain in the leg that was operated
on. He has severe pain on passive flexion of the toes, and also complains of paraesthesia in the first web space.
B - Posterior compartment syndrome« YOUR ANSWER
A - Anterior compartment syndrome« CORRECT ANSWER.

This patient is likely to have an anterior compartment syndrome, especially due to the mechanism of
injury (i.e. tibial fracture). The deep peroneal nerve which supplies the first web space runs in the
extensor or anterior compartment of the leg. Passive stretching of the muscles of the anterior
compartment (i.e. when passively flexing the toes) is responsible for the pain in this instance.

07. Theme: Upper limb injuries


A Anterior dislocation of the shoulder
---------------------------------------------------------------------------------------------------------------------------------------------------------------------------------------------------------------
Dr Mohammed Shamsul Islam Khan, Medical Officer, Clinical Neuro-Surgery, National Institute of Neuro-Sciences and Hospital
Sher-E-Bangla Nagar, Dhaka-1207, Bangladesh. Mobile: +880 1713 455 662, +880 1685 811979. E-mail: drsikhan@gmail.com
MyPasTest: MRCS A Online - Jan Exam 2015
22. Surgical Specialities; Trauma & Orthopaedic Surgery (76Qs)
----------------------------------------------------------------------------------------------------------------------------------
B Acromioclavicular joint dislocation
C Colles’ fracture
D Fracture of the clavicle
E Fracture dislocation of the elbow
F Fracture of the distal humerus
G Fracture of the proximal humerus
H Fracture of the radial head Page |
I Fractured scapula 1312
J Fractured shaft of humerus
K Olecranon fracture
L Posterior dislocation of the shoulder
M Scaphoid fracture
N Smith’s fracture
O Sternoclavicular dislocation
P Supracondylar fracture of the humerus.

The following patients have all fallen, injuring their upper limb. Please select the most appropriate
fracture or dislocation from the above list. The items may be used once, more than once, or not at all.

Scenario 1
A 20-year-old epileptic presents to The Emergency Department following a seizure. On recovering from his
seizure he complains of pain in his right arm. In addition, he is unable to move the affected limb. On examination
his right arm is medially rotated at the shoulder. Active and passive movement is not possible at the shoulder joint.
There is no obvious neurovascular deficit. A plain anteroposterior radiograph shows no obvious defect.
A - Anterior dislocation of the shoulder« YOUR ANSWER
L - Posterior dislocation of the shoulder« CORRECT ANSWER.

L – Posterior dislocation of the shoulder:


This frequently missed injury should be suspected following a seizure or electric shock. The mechanism
involves either a direct blow to the front of the shoulder or forced internal rotation when the arm is
abducted. Clinical features are as described. In addition, the anterior contour of the shoulder may be
flattened with prominence of the coracoid. Anteroposterior radiographs may look virtually normal, but the
medially rotated humeral head appears globe-shaped (light bulb sign). Treatment involves closed
reduction, which is maintained by a shoulder spica.

Scenario 2
A 60-year-old woman has fallen onto her outstretched hand. On presentation she has marked bruising and
tenderness of the upper arm. Neurovascular examination reveals wrist drop.
C - Colles‟ fracture« YOUR ANSWER
J - Fractured shaft of humerus« CORRECT ANSWER.

J – Fractured shaft of humerus:


This injury occurs at all ages but is most common in elderly osteoporotic individuals. The extensive
bruising is characteristic. The diagnosis is supported in this case by the finding of a wrist drop, which
has been caused by a radial nerve injury where it lies in the spiral groove of the humerus. Treatment is
with a hanging cast, the weight of which maintains reduction. In closed injuries the nerve is seldom
divided and the wrist may be splinted while the injury recovers. Fractures to the humeral shaft may be
pathological and any occurring following minor trauma should be viewed with suspicion.
---------------------------------------------------------------------------------------------------------------------------------------------------------------------------------------------------------------
Dr Mohammed Shamsul Islam Khan, Medical Officer, Clinical Neuro-Surgery, National Institute of Neuro-Sciences and Hospital
Sher-E-Bangla Nagar, Dhaka-1207, Bangladesh. Mobile: +880 1713 455 662, +880 1685 811979. E-mail: drsikhan@gmail.com
MyPasTest: MRCS A Online - Jan Exam 2015
22. Surgical Specialities; Trauma & Orthopaedic Surgery (76Qs)
----------------------------------------------------------------------------------------------------------------------------------
Scenario 3
A 24-year-old man presents after falling onto his outstretched hand. He complains of pain at the elbow.
Examination reveals swelling around the elbow. Of note, the patient is unable to extend the elbow against
resistance. There is no obvious neurovascular deficit.
B - Acromioclavicular joint dislocation« YOUR ANSWER
K - Olecranon fracture« CORRECT ANSWER.
Page |
K – Olecranon fracture: 1313
Two types of injury are commonly seen. The first is a comminuted fracture following direct trauma to the
point of the elbow. The second is a traction injury to the olecranon resulting in a transverse fracture. This
typically occurs following a fall onto the hand with the triceps muscle contracted. Transverse fractures
tend to cause disruption to the extensor mechanism of the elbow as in the case described. Displaced
fractures may result in a palpable gap. Treatment of non-displaced fractures involves immobilisation in a
cast followed by supervised mobilisation. Displaced fractures are treated by open reduction and internal
fixation. Methods available include tension band wiring and plating.

08. Theme: Trauma procedures in the resuscitation room


A Pericardiocentesis
B Tube chest drainage
C Needle thoracocentesis
D Emergency thoracotomy
E Venous cutdown
F Cricothyroidotomy
G Diagnostic peritoneal lavage
H Intraosseous infusion
I Central line placement
J Passage of a nasogastric tube
K Endotracheal intubation.

Match the most appropriate procedure from the list above to each clinical situation described below. Each
option may be used once only, more than once or not at all.

Scenario 1
A patient with a systolic blood pressure of 60 mmHg following stab wound to chest, distended neck veins,
reasonable bilateral air entry, central trachea.
A - Pericardiocentesis« CORRECT ANSWER.

Pericardiocentesis:
A cardiac tamponade must be aspirated by pericardiocentesis with a large needle through a sub-xiphoid
approach.

Scenario 2
A patient with a systolic blood pressure of 60 mmHg following blunt chest trauma, distended neck veins, no air
entry on the right side, tracheal deviation to the left.
B - Tube chest drainage« YOUR ANSWER
C - Needle thoracocentesis« CORRECT ANSWER.

Needle thoracocentesis:
---------------------------------------------------------------------------------------------------------------------------------------------------------------------------------------------------------------
Dr Mohammed Shamsul Islam Khan, Medical Officer, Clinical Neuro-Surgery, National Institute of Neuro-Sciences and Hospital
Sher-E-Bangla Nagar, Dhaka-1207, Bangladesh. Mobile: +880 1713 455 662, +880 1685 811979. E-mail: drsikhan@gmail.com
MyPasTest: MRCS A Online - Jan Exam 2015
22. Surgical Specialities; Trauma & Orthopaedic Surgery (76Qs)
----------------------------------------------------------------------------------------------------------------------------------
A tension pneumothorax is a clinical diagnosis and is treated initially by needle aspiration through the
second intercostal space in the mid-clavicular line. Formal chest drainage is performed later towards the
end of the primary survey.

Scenario 3
A prerequisite for diagnostic peritoneal lavage.
C - Needle thoracocentesis« YOUR ANSWER Page |
J - Passage of a nasogastric tube« CORRECT ANSWER. 1314
Passage of a nasogastric tube:
Before diagnostic peritoneal lavage can be performed both a urinary catheter and a nasogastric tube
must be in place.

Scenario 4
The recommended route for fluid replacement in children after peripheral cannulation fails.
D - Emergency thoracotomy« YOUR ANSWER
H - Intraosseous infusion« CORRECT ANSWER.

Scenario 5
The recommended method for fluid replacement in adults after peripheral cannulation fails during the
management of major traumatic injuries.
E - Venous cutdown« YOUR ANSWER
H - Intraosseous infusion« CORRECT ANSWER.

Intraosseous infusion:
After two attempts at peripheral cannulation in the child intraosseous infusion is recommended. Central
lines should be avoided. This method is also recommended as the preferred means of achieving IV
access in adults in whom peripheral cannulation is unsuccessful in the acute traumatic setting (i.e. where
prompt and urgent fluid resuscitation is required).

Scenario 6
The first procedure in a pulseless apnoeic patient after a chest stabbing.
F - Cricothyroidotomy« YOUR ANSWER
K - Endotracheal intubation« CORRECT ANSWER.

Scenario 7
The second procedure in a pulseless apnoeic patient after a chest stabbing.
G - Diagnostic peritoneal lavage« YOUR ANSWER
D - Emergency thoracotomy« CORRECT ANSWER.

Emergency thoracotomy:
According to ATLS principles of ABCDE, securing the airway (A) would be the first consideration together
with securing intravenous access using large bore simple intravenous cannulae (not an option here).
Second to this, a thoracotomy to attempt to arrest the bleeding would be performed. In the case of open
trauma, this would be of more use than inserting a chest drain.

Scenario 8
The procedure required for a large haemothorax.
---------------------------------------------------------------------------------------------------------------------------------------------------------------------------------------------------------------
Dr Mohammed Shamsul Islam Khan, Medical Officer, Clinical Neuro-Surgery, National Institute of Neuro-Sciences and Hospital
Sher-E-Bangla Nagar, Dhaka-1207, Bangladesh. Mobile: +880 1713 455 662, +880 1685 811979. E-mail: drsikhan@gmail.com
MyPasTest: MRCS A Online - Jan Exam 2015
22. Surgical Specialities; Trauma & Orthopaedic Surgery (76Qs)
----------------------------------------------------------------------------------------------------------------------------------
H - Intraosseous infusion« YOUR ANSWER
B - Tube chest drainage« CORRECT ANSWER.

Tube chest drainage:


In all trauma, the first procedure is to secure the airway. In penetrating chest injury, up to 30% of
pulseless patients can be saved by emergency thoracotomy if performed by a qualified operator when
there is still some electrical cardiac activity. Page |
1315
09. Theme: Haemorrhagic shock
A Blood loss of 2.3 litres
B Blood loss of 1.7 litres
C Blood loss of 2.5 litres
D Blood loss of 0.5 litre
E Blood loss of 1 litre.

For each of the above volumes of blood loss, select the most appropriate physiological change in a 70kg
adult male. Each option may be used once, more than once, or not at all.

Scenario 1
Normal heart rate
A - Blood loss of 2.3 litres« YOUR ANSWER
D - Blood loss of 0.5 litre« CORRECT ANSWER.

Scenario 2
Unconscious
B - Blood loss of 1.7 litres« YOUR ANSWER
C - Blood loss of 2.5 litres« CORRECT ANSWER.

Scenario 3
Reduced systolic pressure
C - Blood loss of 2.5 litres« YOUR ANSWER
B - Blood loss of 1.7 litres« CORRECT ANSWER.

Scenario 4
Confused and lethargic
D - Blood loss of 0.5 litre« YOUR ANSWER
B - Blood loss of 1.7 litres« CORRECT ANSWER.

Scenario 5
Respiratory rate 20 - 30 breaths/min
E - Blood loss of 1 litre« CORRECT ANSWER.
The effects of blood loss can be monitored in terms of physiology, ie pulse rate/blood pressure/pulse
pressure/respiratory rate/urine output/CNS or mental status. Shock is graded I–IV depending on the
amount of blood loss, and each grade is associated with certain physiological changes. Class I shock
occurs when up to 15% of the blood volume is lost (up to 750 ml in a 70-kg adult); class II between 15 and
30% (750–1500 ml); class III between 30 and 40% (1500–2000 ml), class IV > 40% (2000 ml). The
percentages are best remembered as the scoring system in a tennis match.
---------------------------------------------------------------------------------------------------------------------------------------------------------------------------------------------------------------
Dr Mohammed Shamsul Islam Khan, Medical Officer, Clinical Neuro-Surgery, National Institute of Neuro-Sciences and Hospital
Sher-E-Bangla Nagar, Dhaka-1207, Bangladesh. Mobile: +880 1713 455 662, +880 1685 811979. E-mail: drsikhan@gmail.com
MyPasTest: MRCS A Online - Jan Exam 2015
22. Surgical Specialities; Trauma & Orthopaedic Surgery (76Qs)
----------------------------------------------------------------------------------------------------------------------------------
10. Theme: Orthopaedic trauma
A Intramedullary nail
B Traction (Thomas' splint)
C Internal fixation with plate and screws
D Debridement and external fixation
E Below-knee amputation
F Repair of vessel and nerves Page |
G Above-knee amputation 1316
H Plaster of Paris cast.

Select the most appropriate treatment for the patients described below. Each option may be used once,
more than once, or not at all.
Scenario 1
A 24-year-old male involved in a road traffic accident (RTA) is brought to the emergency department with head
injury, flail chest and comminuted femoral shaft fracture. His Glasgow Coma Score is 8. He is intubated for his
head injury and a chest drain is inserted. He is now stabilised. How is his fracture best treated?
A - Intramedullary nail« YOUR ANSWER
B - Traction (Thomas' splint)« CORRECT ANSWER.
Scenario 2
A 30-year-old male involved in an RTA is brought to the emergency department with a Grade IIIC open tibial
fracture, an insensate sole, and a pale and cold foot. A tyre mark is visible across his calf. His blood pressure is
90/66 mmHg and his pulse rate is 130/min. At surgery, the popliteal nerve is noticed to be crushed and divided.
There is also injury to the popliteal artery.
B - Traction (Thomas' splint)« YOUR ANSWER
E - Below-knee amputation« CORRECT ANSWER.
Scenario 3
A 25-year-old man fell from a height sustaining, a pneumo/haemothorax, fractured left humerus and a closed
comminuted fracture to the mid-shaft of the left femur. The pneumo/haemothorax is treated with a chest drain and
a collar and cuff is used to treat the humerus fracture. The patient is stable. How is the femoral fracture best
treated?
C - Internal fixation with plate and screws« YOUR ANSWER
A - Intramedullary nail« CORRECT ANSWER.
Patients with multisystem trauma and multiple fractures (of long bones) are best stabilised by having
their fractures treated by internal fixation. This allows for more rapid improvement, reduction in incidence
of acute respiratory distress syndrome and earlier mobilisation. However, in the first example, the 24-
year-old man’s other injuries (chest and head) take precedence and the fracture should be initially treated
with traction.

In severe lower limb trauma, a careful evaluation of the extent of injury is needed. Reconstruction is
determined by the nature and severity of the injuries. In the second example, there is gross soft tissue
(muscle and nerve) and bony damage which precludes any possibility of limb salvage. In addition, the
patient is also haemodynamically unstable, possibly due to blood loss from the fracture site and injury to
the popliteal artery Hence below-knee amputation is the most appropriate treatment option.

---------------------------------------------------------------------------------------------------------------------------------------------------------------------------------------------------------------
Dr Mohammed Shamsul Islam Khan, Medical Officer, Clinical Neuro-Surgery, National Institute of Neuro-Sciences and Hospital
Sher-E-Bangla Nagar, Dhaka-1207, Bangladesh. Mobile: +880 1713 455 662, +880 1685 811979. E-mail: drsikhan@gmail.com
MyPasTest: MRCS A Online - Jan Exam 2015
22. Surgical Specialities; Trauma & Orthopaedic Surgery (76Qs)
----------------------------------------------------------------------------------------------------------------------------------
11. A 50-year-old man presents with pain in his right shoulder. The pain started 2 days ago, after he had
painted a ceiling. The pain is present on forward flexion and abduction between 700-1200. Hawkin's test is
positive, Jobe’s test, internal and external rotation lag signs, and Gerber’s lift-off tests are negative. What
is the most likely diagnosis? Select one answer only.
Acromioclavicular joint arthritis« YOUR ANSWER
Acute calcific tendonitis Page |
Glenohumeral joint arthritis 1317
Rotator cuff tear
Subacromial impingement« CORRECT ANSWER.

Subacromial impingement is most common between 40 and 50 years of age, it causes a painful arc of
movement from 700-1200. It can be precipitated by unaccustomed overhead activity, e.g. painting a ceiling.
Hawkin’s test is for subacromial impingement. Jobe’s test is for supraspinatus, the internal rotation lag
sign and Greber’s lift-off test assess subscapularis, and the external rotation lag test assesses
infraspinatus. Acute calcific tendonitis tends to present with sudden, acute, severe shoulder pain, with
pain present even at rest and on all movements.

12. Theme: Knee injuries


A Medial meniscus
B Lateral meniscus
C Medial collateral ligament
D Lateral collateral ligament
E Patella fracture
F Anterior cruciate ligament
G Posterior cruciate Ligament.

Pick the most appropriate option from the above list. Each option may be used once only, more than once
or not at all.

Scenario 1
A young footballer injures his right knee in a tackle and develops swelling of the knee a few days later. He
presents to casualty with an area of tenderness 2–3 cm above the joint line on medial aspect. He has increased
valgus deformity on examination.
A - Medial meniscus« YOUR ANSWER
C - Medial collateral ligament« CORRECT ANSWER.

The history in the young footballer is most suggestive of a medial collateral ligament injury as there is a
valgus deformity and tenderness above the joint line. Tenderness over the joint line is suggestive of
meniscal injury.

Scenario 2
A 60-year-old develops a painful left knee following a fall. On examination in the orthopaedic clinic a few days
later, swelling on the left knee is seen. Flexion views of both knees show that the left tibia plateau lies more
anteriorly than on the right.
B - Lateral meniscus« YOUR ANSWER
F - Anterior cruciate ligament« CORRECT ANSWER.
---------------------------------------------------------------------------------------------------------------------------------------------------------------------------------------------------------------
Dr Mohammed Shamsul Islam Khan, Medical Officer, Clinical Neuro-Surgery, National Institute of Neuro-Sciences and Hospital
Sher-E-Bangla Nagar, Dhaka-1207, Bangladesh. Mobile: +880 1713 455 662, +880 1685 811979. E-mail: drsikhan@gmail.com
MyPasTest: MRCS A Online - Jan Exam 2015
22. Surgical Specialities; Trauma & Orthopaedic Surgery (76Qs)
----------------------------------------------------------------------------------------------------------------------------------
Anterior cruciate ligament:
The anterior cruciate ligament prevents backward displacement of the femur on the tibial plateau and
limits extension of the lateral condyle of the femur.

Scenario 3
A patient involved in a road traffic accident hits his knee on the dashboard, producing an immediate
haemarthrosis. X-ray shows no bony injury, but he later experiences difficulty walking down stairs. Page |
C - Medial collateral ligament« YOUR ANSWER 1318
G - Posterior cruciate Ligament« CORRECT ANSWER.

Posterior cruciate ligament:


The posterior cruciate ligament prevents the femur from sliding forwards off the tibial plateau. In the
weight bearing flexed knee it is the only stabilising factor for the femur and its attached quadriceps. In
walking down stairs, the upper knee is flexed and weight bearing, while the lower knee is straight.
Immediate haemarthrosis is indicative of cruciate ligament rupture or fracture.

13. Theme: Shoulder pain


A Acromioclavicular joint disruptions
B Biceps rupture
C Subacromial bursitis
D Supraspinatus rupture
E Supraspinatus tendonitis.

For each of the patients described below, choose the most suitable diagnosis from the list of options
above. Each option may be used once, more than once, or not at all.

Scenario 1
A 52-year-old plasterer complains of a 1-year history of shoulder pain and difficulty in lifting his arm up while
performing his job. On examination, he had marked tenderness over the acromion. He has to bend over to the
affected side to initiate shoulder abduction. He has no difficulty in passive abduction of his arm.
A - Acromioclavicular joint disruptions« YOUR ANSWER
D - Supraspinatus rupture« CORRECT ANSWER.

D – Supraspinatus rupture:
A complete tear of the supraspinatus tendon may occur after a long period of chronic tendonitis. Active
abduction is impossible and attempting it produces a characteristic shrug; however, passive abduction is
full and once the arm has been lifted to above a right angle the patient can keep it raised using the deltoid
(abduction paradox).
Scenario 2
A 32-year-old man presents with a 1-month history of shoulder pain especially on lifting the arm. On examination
he has marked tenderness lateral to the acromial process with a painful arc of 60–120°.
B - Biceps rupture« YOUR ANSWER
E - Supraspinatus tendonitis« CORRECT ANSWER.

E – Supraspinatus tendonitis:
Supraspinatus tendonitis usually occurs in patients < 40 years of age who develop shoulder pain after
vigorous/strenuous exercise. On active abduction the scapulohumeral rhythm is disturbed and pain is
aggravated as the arm traverses an arc between 60 and 120°.
---------------------------------------------------------------------------------------------------------------------------------------------------------------------------------------------------------------
Dr Mohammed Shamsul Islam Khan, Medical Officer, Clinical Neuro-Surgery, National Institute of Neuro-Sciences and Hospital
Sher-E-Bangla Nagar, Dhaka-1207, Bangladesh. Mobile: +880 1713 455 662, +880 1685 811979. E-mail: drsikhan@gmail.com
MyPasTest: MRCS A Online - Jan Exam 2015
22. Surgical Specialities; Trauma & Orthopaedic Surgery (76Qs)
----------------------------------------------------------------------------------------------------------------------------------
14. Theme: Bone and connective tissue tumours
A Chondrosarcoma
B Ewing’s sarcoma
C Fibrosarcoma
D Leiomyosarcoma
E Malignant giant cell tumour
F Osteochondroma Page |
G Osteoclastoma 1319
H Osteoid osteoma
I Osteosarcoma
J Rhabdomyosarcoma.

For each of the following statements, select the most likely cause of pain/swelling from the above list.
Each option may be used once, more than once, or not at all.

Scenario 1
A 65-year-old presents with a swelling and pain over her left proximal humerus. The pain is worse at night. She
also gives a history of weight loss. She has a raised ESR and she is undergoing treatment for Paget‟s disease.
A - Chondrosarcoma« YOUR ANSWER
I - Osteosarcoma« CORRECT ANSWER.

I – Osteosarcoma:
Osteosarcoma, the most frequently encountered malignant lesion of bone, is characterised by the direct
formation of bone or osteoid tissue by a sarcomatous stroma. It typically affects the knee and the
proximal humerus in the metaphyseal region. The incidence is highest in the 10–25-years age group.
Secondary osteosarcoma, however, may arise in the bones of the elderly affected by Paget’s disease (in
approximately 10% of patients affected for > 10 years) or after irradiation.

The characteristic symptoms of pain, local tenderness, a soft tissue mass and a decreased function may
be present for variable periods of time. On examination, the affected part is swollen and the overlying skin
may be shiny and warm. The lump is tender and has irregular edges. The ESR may be raised. X-ray shows
bone destruction and new bone formation, often with marked periosteal elevation (‘Sunray spiculation’
and ‘Codman’s triangle’, respectively). Surgical excision is the treatment of choice.

Scenario 2
A 13-year-old adolescent is brought by his parents to the GP with loss of weight, pain and fever. On examination,
a soft but tender, ill-defined mass is palpable over his mid-thigh region.
B - Ewing‟s sarcoma « CORRECT ANSWER.

B – Ewing’s sarcoma:
Ewing’s sarcoma is a malignant tumour arising from the vascular endothelium of the bone marrow. The
tumour is common in the 10–20-years age group and occurs in the diaphysis of the long bones. Clinical
features include pain and swelling; the lump is warm and tender, with ill-defined edges. The ESR may be
elevated, thus spuriously suggesting an inflammatory or infective cause such as osteomyelitis, although
osteomyelitis usually affects the metaphyseal region in children. X-rays often show a large soft-tissue
mass with concentric layers of new bone formation – known as ‘onion-peel’ sign. Treatment includes
chemotherapy and surgical excision.

---------------------------------------------------------------------------------------------------------------------------------------------------------------------------------------------------------------
Dr Mohammed Shamsul Islam Khan, Medical Officer, Clinical Neuro-Surgery, National Institute of Neuro-Sciences and Hospital
Sher-E-Bangla Nagar, Dhaka-1207, Bangladesh. Mobile: +880 1713 455 662, +880 1685 811979. E-mail: drsikhan@gmail.com
MyPasTest: MRCS A Online - Jan Exam 2015
22. Surgical Specialities; Trauma & Orthopaedic Surgery (76Qs)
----------------------------------------------------------------------------------------------------------------------------------
Scenario 3
A 23-year-old presents with mild discomfort and a lump over his right knee. He accidentally discovered the lump
one week ago. On examination, it is bony hard and non-tender. He is systemically well.
C - Fibrosarcoma « YOUR ANSWER
F - Osteochondroma« CORRECT ANSWER.

F – Osteochondroma: Page |
Osteochondroma (cartilage-capped exostosis) is the most common benign tumour of the bone. The usual 1320
site for the tumour is the metaphysis of the long bones. The lesion may be single or multiple (hereditary
multiple exostoses). The usual history is of a lump that is discovered accidentally. The lump is bony hard
and non-tender. X-ray reveals a well-defined swelling; however, the swelling looks smaller than it feels
because of the invisible cartilaginous cap.

Scenario 4
A 28-year-old lady presents with weight loss, fever and a swelling over her right knee of a few weeks duration.
Movements of her knee are severely restricted. X-ray reveals thinning of the cortex and a fracture of the distal
femur. An extraosseous soft tissue mass is seen on MRI scan.
E - Malignant giant cell tumour « YOUR ANSWER
G - Osteoclastoma« CORRECT ANSWER.

G – Osteoclastoma:
Osteoclastoma (giant cell tumour) is an uncommon, aggressive, locally destructive lesion seen in the
metaphyseo-epiphyseal region of long bones. It frequently occurs in young adults 20–40 years of age and
is more common in women. The principal sites are the distal femur, proximal tibia, proximal fibula, distal
radius and proximal humerus. On examination, a vague swelling is felt at the end of long bones and the
neighbouring joint is often inflamed.

Although < 5% of these tumours metastasise, the lesions are extremely destructive, sometimes locally
resulting in pathological fractures (approximately 10% of cases) as seen in this patient. X-ray shows
thinning of the cortex giving it an expanded appearance – the characteristic ‘soap bubble’ appearance.
Surgical excision is the treatment of choice, with concurrent bone grafting undertaken, if necessary. If
this is not possible, radiotherapy may be attempted.
15. Theme: Joint and back pain
A Ankylosing spondylitis
B Intervertebral disc herniation
C Metastatic disease
D Multiple myeloma
E Osteoarthritis
F Paget’s disease
G Reiter’s syndrome
H Rheumatoid arthritis
I Spinal stenosis
J Spondylolisthesis.
For each of the following statements, select the most likely cause of joint/back pain from the above list.
Each option may be used once, more than once, or not at all.

Scenario 1
---------------------------------------------------------------------------------------------------------------------------------------------------------------------------------------------------------------
Dr Mohammed Shamsul Islam Khan, Medical Officer, Clinical Neuro-Surgery, National Institute of Neuro-Sciences and Hospital
Sher-E-Bangla Nagar, Dhaka-1207, Bangladesh. Mobile: +880 1713 455 662, +880 1685 811979. E-mail: drsikhan@gmail.com
MyPasTest: MRCS A Online - Jan Exam 2015
22. Surgical Specialities; Trauma & Orthopaedic Surgery (76Qs)
----------------------------------------------------------------------------------------------------------------------------------
A 25-year-old presents with an unstable fracture of the fourth lumbar vertebra following a fall. He is on long-term
NSAIDs for a painful and stiff back. On examination, he has kyphosis and serology for human leukocyte antigen
B27 (HLA-B27) is positive.
A - Ankylosing spondylitis « CORRECT ANSWER.

A – Ankylosing spondylitis:
Ankylosing spondylitis, predominantly seen in the young, affects more men than women (6 : 1). The Page |
patient presents with morning stiffness, backache, progressive loss of spinal movements leading to 1321
kyphosis, and hyperextension of the neck (question mark posture). Occasionally, patients may present
with unstable fractures after minor trauma. X-rays show a ‘bamboo spine’ with squaring of the vertebrae.
Blood test reveals raised ESR, normochromic anaemia and positive HLA-B27 antigen.

Scenario 2
A 33-year-old previously fit patient presents with an acute low backache, which commenced when he was lifting a
heavy object at work. He also complains of shooting pain radiating down the back of his right thigh.
B - Intervertebral disc herniation « CORRECT ANSWER.

B – Intervertebral disc herniation:


Lumbar disc herniation (prolapse) commonly occurs in fit young adults usually when lifting heavy weight
or while straining. A sudden acute pain is felt in the lower back, and it may be accompanied by shooting
pain radiating to the buttock or down the leg along the appropriate nerve roots. Examination reveals
paravertebral muscle spasm, leading often to a ‘spinal tilt’ and a global reduction in spinal movements.
Straight leg raise (SLR) is often restricted to < 50° and reproduces the radicular symptoms.

Scenario 3
A 25-year-old presents with a painful right knee, and vesicles and pustules over the soles of his feet. He also
complains of soreness in his eyes and burning on micturition. He gives a history of unprotected sex 2 months ago.
C - Metastatic disease « YOUR ANSWER
G - Reiter‟s syndrome« CORRECT ANSWER.

G – Reiter’s syndrome:
Reiter’s syndrome (sexually acquired reactive arthritis) is a triad of urethritis, conjunctivitis and
seronegative arthritis. The patient is usually young; the disease affects large joints and causes oligo- or
mono-arthritis. Other features include iritis, keratoderma blenorrhagica (brown, aseptic abscesses on
soles and palms), circinate balanitis (painless serpiginous penile rash), plantar fasciitis and Achilles
tendonitis. Management includes NSAIDs, rest, and splintage of the affected joint.

Scenario 4
A 12-year-old gymnast is brought by her parents to the orthopaedic clinic with lower backache of 1-year duration.
On examination, she has L5 root pain and a hamstring spasm. Straight leg raise is reduced.
D - Multiple myeloma « YOUR ANSWER
J - Spondylolisthesis« CORRECT ANSWER.

J – Spondylolisthesis:
Spondylolisthesis is the slipping forward of one vertebra in relation to another. Isthmic spondylolisthesis
is one type (of five) and appears to be a form of repetitive stress fracture; the incidence is much higher in
teenage gymnasts and other athletes. It commonly occurs between the ages of 7 and 10. The signs and
symptoms include low backache, hamstring spasm, fifth lumbar nerve root pain, and disturbance in the
---------------------------------------------------------------------------------------------------------------------------------------------------------------------------------------------------------------
Dr Mohammed Shamsul Islam Khan, Medical Officer, Clinical Neuro-Surgery, National Institute of Neuro-Sciences and Hospital
Sher-E-Bangla Nagar, Dhaka-1207, Bangladesh. Mobile: +880 1713 455 662, +880 1685 811979. E-mail: drsikhan@gmail.com
MyPasTest: MRCS A Online - Jan Exam 2015
22. Surgical Specialities; Trauma & Orthopaedic Surgery (76Qs)
----------------------------------------------------------------------------------------------------------------------------------
sagittal profile of the spine with an acute kyphosis. In some cases, neurological symptoms may affect the
legs. On examination, there may be a step in the line of the spinous processes; straight leg raising may
be reduced because of hamstring spasm.

16. Theme: Knee injury


A Medial meniscal tear
B Lateral meniscal tear Page |
C Anterior cruciate ligament rupture 1322
D Posterior cruciate ligament rupture
E Medial collateral ligament tear
F Lateral collateral ligament tear
G Knee effusion.

What is the injury in the following patients? Match the appropriate diagnosis for each scenario. Each
option may be used once, more than once, or not at all.

Scenario 1
A 25-year-old man sustains injury to his right knee during a football tackle. However, he continues to play albeit
with severe discomfort. He presents to the emergency department after 24 h with a swollen, tender right knee. He
is unable to straighten the knee fully and tenderness is elicited over the anterior medial aspect of the knee at the
level of the joint line.
A - Medial meniscal tear« CORRECT ANSWER.

Scenario 2
A 25-year-old man sustains severe injury to his left knee from an awkward fall while skiing. He reports hearing a
pop and swelling in this knee was noticed immediately. There is no localised joint line tenderness, but he is unable
to fully extend this knee.
B - Lateral meniscal tear« YOUR ANSWER
C - Anterior cruciate ligament rupture« CORRECT ANSWER.

A history of twisting injury and pain at the antero-medial aspect of the joint is indicative of a medial
meniscal injury. If bleeding has occurred into the joint space, the knee swells within 1–2 h. This is usually
suggestive of a serious intra-articular damage such as an anterior cruciate ligament rupture. If the knee
swells over a 12–24 h period, the swelling is more likely to be due to a simple effusion. As fluid collects in
the knee, movement becomes more restricted and painful and only a few degrees of movement may be
possible. The knee is usually held in 10° of flexion. In collateral ligament injuries, the pain is usually
localised; however, no clear tenderness can be elicited in cruciate ligament injuries.
17. Theme: Knee injuries
A Anterior cruciate rupture
B Haemarthrosis
C Injury to the medial meniscus
D Medial ligament rupture
E Patellar fracture
F Tibial plateau fracture.

For each of the clinical situations described below, select the most likely diagnosis from the above list.
Each option may be used once, more than once, or not at all.
---------------------------------------------------------------------------------------------------------------------------------------------------------------------------------------------------------------
Dr Mohammed Shamsul Islam Khan, Medical Officer, Clinical Neuro-Surgery, National Institute of Neuro-Sciences and Hospital
Sher-E-Bangla Nagar, Dhaka-1207, Bangladesh. Mobile: +880 1713 455 662, +880 1685 811979. E-mail: drsikhan@gmail.com
MyPasTest: MRCS A Online - Jan Exam 2015
22. Surgical Specialities; Trauma & Orthopaedic Surgery (76Qs)
----------------------------------------------------------------------------------------------------------------------------------
Scenario 1
A 29-year-old man was hit by a car as he ran across the road. He was subsequently unable to weight-bear. His
knee was bruised, swollen and tender.
A - Anterior cruciate rupture « YOUR ANSWER
F - Tibial plateau fracture« CORRECT ANSWER.

Scenario 2 Page |
A 45-year-old woman stumbled over an uneven paving stone and landed heavily on her knee. She walked aided 1323
into the Emergency Department with a swollen painful knee and was unable to straight leg raise.
B - Haemarthrosis « YOUR ANSWER
E - Patellar fracture« CORRECT ANSWER.

Scenario 3
A footballer sustained a twisting injury to his flexed knee while playing a game yesterday. He was unable to
complete the game and by this morning his knee was very swollen. He was unable to fully flex or extend his knee.
C - Injury to the medial meniscus « CORRECT ANSWER.

Scenario 4
A 25-year-old man landed awkwardly having jumped for a ball in the line-out during a rugby match. He heard a
pop and was unable to complete the game. He noticed his knee swell immediately.
D - Medial ligament rupture« YOUR ANSWER
A - Anterior cruciate rupture « CORRECT ANSWER.

Scenario 5
A 25-year-old man was involved in a tackle during a football game today. A valgus force was applied to the knee –
he fell to the ground and noted that his knee was at a „funny angle‟. He has been unable to weight-bear and says
that his knee feels „unsafe‟. On examination, his knee is generally tender, there is significant laxity on valgus
stress but no definite effusion.
E - Patellar fracture« YOUR ANSWER
D - Medial ligament rupture« CORRECT ANSWER.

A valgus force to the knee produced by a car bumper is likely to result in bony damage (a tibial plateau
fracture); a similarly directed force during a game of football is more likely to lead to rupture of the medial
collateral ligament. Complete rupture of this ligament would lead to gross instability on weight-bearing at
the time of injury. As the joint is disrupted, no discrete effusion is seen, although the area may be swollen
and bruised. Swelling due to a medial meniscal injury usually takes some hours to become apparent.
However, an anterior cruciate ligament rupture with the classical ‘pop’ produces a rapid haemarthrosis.

18. Theme: Shoulder pain


A Supraspinatus tendonitis
B Supraspinatus rupture
C Subacromial burstis
D Biceps rupture
E Acromioclavicular joint disruptions.

Scenario 1

---------------------------------------------------------------------------------------------------------------------------------------------------------------------------------------------------------------
Dr Mohammed Shamsul Islam Khan, Medical Officer, Clinical Neuro-Surgery, National Institute of Neuro-Sciences and Hospital
Sher-E-Bangla Nagar, Dhaka-1207, Bangladesh. Mobile: +880 1713 455 662, +880 1685 811979. E-mail: drsikhan@gmail.com
MyPasTest: MRCS A Online - Jan Exam 2015
22. Surgical Specialities; Trauma & Orthopaedic Surgery (76Qs)
----------------------------------------------------------------------------------------------------------------------------------
A 52-year-old billboard poster plasterer, with a 1-year history of shoulder pain and difficulty in lifting his arm up
while performing his job. On examination he had marked tenderness over the acromion process. He is unable to
initiate abduction of the arm. He has no difficulty adducting his arm.
A - Supraspinatus tendonitis« YOUR ANSWER
B - Supraspinatus rupture« CORRECT ANSWER.

Scenario 2 Page |
A 32-year-old man presented with a 1-month history of shoulder pain especially on lifting his arm. On examination 1324
he had marked tenderness lateral to the acromion process with a painful arc of 60–120°.
B - Supraspinatus rupture« YOUR ANSWER
A - Supraspinatus tendonitis« CORRECT ANSWER.

A complete tear of the supraspinatus tendon may occur after a long period of chronic tendinitis. Active
abduction is impossible and attempting it produces a characteristic shrug, although passive abduction is
full and, once the arm has been lifted to above a right angle, the patient can keep it up using the deltoid
(abduction paradox). Supraspinatus tendinitis usually occurs in a patient aged < 40 years who develops
shoulder pain after vigorous/strenuous exercise. On active abduction scapulohumeral rhythm is
disturbed and pain is aggravated as the arm traverses an arc between 60 and 120°.

19. Theme: Soft tissue injuries of the knee joint


A Medial meniscal tear
B Lateral meniscal tear
C Supracondylar fracture of the femur
D Fracture of the fibular head
E Anterior cruciate ligament rupture
F Posterior cruciate ligament rupture
G Patellar fracture
H Medial collateral ligament
I Lateral collateral ligament.

Select the most likely site of damage for the patients described below. Match the appropriate answer for
each scenario. Each option may be used once, more than once, or not at all.
Scenario 1
A 20-year-old man present to the emergency department with an acutely swollen and painful left knee after falling
awkwardly on this knee while playing football. There is tenderness over the medial joint line. There is no joint
instability.
A - Medial meniscal tear« CORRECT ANSWER.
Scenario 2
A 66-year-old lady presents to the emergency department with a painful and swollen right kneeafter she tripped
and fell over on the pavement. There is tenderness and crepitus on palpation over the patella. Aspiration of the
knee joint reveals blood and fat globules.
B - Lateral meniscal tear« YOUR ANSWER
G - Patellar fracture« CORRECT ANSWER.
Scenario 3

---------------------------------------------------------------------------------------------------------------------------------------------------------------------------------------------------------------
Dr Mohammed Shamsul Islam Khan, Medical Officer, Clinical Neuro-Surgery, National Institute of Neuro-Sciences and Hospital
Sher-E-Bangla Nagar, Dhaka-1207, Bangladesh. Mobile: +880 1713 455 662, +880 1685 811979. E-mail: drsikhan@gmail.com
MyPasTest: MRCS A Online - Jan Exam 2015
22. Surgical Specialities; Trauma & Orthopaedic Surgery (76Qs)
----------------------------------------------------------------------------------------------------------------------------------
A 19-year-old rugby player presents to the emergency department with a painful and swollen left knee after he
sustained injury to that knee while involved in a scrum. There is severe tenderness along the lateral aspect of the
knee joint and the pain is worsened with valgus and varus force. The joint feels unstable.
C - Supracondylar fracture of the femur« YOUR ANSWER
I - Lateral collateral ligament« CORRECT ANSWER.

A history of twisting injury and pain at the antero-medial aspect of the joint is indicative of a medial Page |
meniscal injury. Cruciate ligament rupture characteristically produces swelling with 1–2 h of the injury. 1325
Aspiration of blood suggests severe intra-articular damage. The presence of fat globules usually signifies
a fracture, such as fracture of the patella in this patient. In injuries secondary to trauma, plain radiography
is needed to confirm or rule out associated fractures in patients presenting with swelling and tenderness
of the knee joint, although they may manifest classical signs and symptoms of a soft tissue injury.
Collateral ligament injuries cause pain and discomfort with straining of the knee in the valgus and varus
positions.

20. Theme: Upper limb nerve injuries


A Anterior interosseous nerve
B Injury to the lower cord of brachial plexus
C Injury to the upper cord of brachial plexus
D Long thoracic nerve
E Median nerve
F Musculocutaneous nerve
G Posterior interosseous nerve
H Radial nerve
I Suprascapular nerve
J Ulnar nerve.

For each of the following statements, select the most likely cause of nerve injury from the above list. Each
option may be used once, more than once, or not at all.

Scenario 1
A 25-year-old motorcyclist is brought to the Emergency Department after an RTA. He complains of pain in the root
of his neck; on examination, his right arm is adducted, internally rotated and extended at the elbow. There is loss
of sensation along the outer aspect of the arm and forearm.
A - Anterior interosseous nerve« YOUR ANSWER
C - Injury to the upper cord of brachial plexus« CORRECT ANSWER.

C – Injury to the upper cord of the brachial plexus:


Upper brachial plexus or supraclavicular plexus lesion (C5, C6) occurs because of excessive depression
of the shoulder or displacement of the head that opens out the angle between the shoulder and the neck
(Erb-Duchenne paralysis). In neonates, it may occur following traction on the shoulder girdle during
difficult labour or in breech delivery. In adults, it may occur as a result of a fall of weight on the shoulder
or an RTA where the head is moved away from the shoulder. There is loss of shoulder abduction, elbow
flexion and forearm supination. Consequently, the affected limb is internally rotated, extended at the
elbow and pronated. There is sensory loss over the outer aspect of the arm and forearm.

Scenario 2
---------------------------------------------------------------------------------------------------------------------------------------------------------------------------------------------------------------
Dr Mohammed Shamsul Islam Khan, Medical Officer, Clinical Neuro-Surgery, National Institute of Neuro-Sciences and Hospital
Sher-E-Bangla Nagar, Dhaka-1207, Bangladesh. Mobile: +880 1713 455 662, +880 1685 811979. E-mail: drsikhan@gmail.com
MyPasTest: MRCS A Online - Jan Exam 2015
22. Surgical Specialities; Trauma & Orthopaedic Surgery (76Qs)
----------------------------------------------------------------------------------------------------------------------------------
A 40-year-old patient presents to the Emergency Department with carpal dislocation (confirmed radiologically). On
examination, there is loss of sensation over the thumb, index and middle fingers.
B - Injury to the lower cord of brachial plexus« YOUR ANSWER
E - Median nerve« CORRECT ANSWER.

E – Median nerve:
In adults, the median nerve (C5–T1) is commonly injured near the wrist, although it can be injured Page |
anywhere along the arm or the forearm. Low lesions may be caused by lacerations in front of the wrist or 1326
by carpal dislocation. In children, supracondylar fractures of the humerus may lead to median nerve
injury at the elbow. Median nerve injuries at the wrist cause sensory loss over the thumb, index, middle
and occasionally ring finger (lateral half); motor loss includes all thenar muscles except adductor pollicis
(supplied by ulnar nerve) and the lateral two lumbricals. If the injury is at the level of the elbow, there is
paralysis of the pronators of the forearm and flexors of the wrist and fingers, with the exception of flexor
carpi ulnaris and the medial part of flexor digitorum profundus.

Scenario 3
A 65-year-old is brought to the Emergency Department with a wrist drop and sensory loss over a small patch at
the base of the thumb. X-ray shows a mid-humerus fracture.
D - Long thoracic nerve« YOUR ANSWER
H - Radial nerve« CORRECT ANSWER.

H – Radial nerve:
The radial nerve (C5–T1) is damaged at the mid-humerus level by fractures or pressure (prolonged
tourniquet). Damage is seen in patients who fall asleep with the arm dangling over the back of a chair
(‘Saturday night palsy’). Radial nerve injury causes paralysis of the brachioradialis, the wrist extensors
and extensor digitorum, leading to wrist-drop along with paralysis of extensors of the wrist, fingers and
the thumb; there may be a small patch of sensory loss over the dorsum of the thumb and the first web
space. In higher lesions, sensation is also lost on the dorsum of the forearm.

Scenario 4
A 30-year-old presents to the Emergency Department with a deep laceration to his right wrist after he was
involved in a fight in a pub. On examination, he is unable to pinch and has got loss of sensation over his little and
ring fingers.
E - Median nerve« YOUR ANSWER
J - Ulnar nerve« CORRECT ANSWER.

J – Ulnar nerve:
The ulnar nerve (C8, T1) is an important motor nerve of the hand. Pressure (from a deep ganglion) or
lacerations at the wrist may cause low lesions. Lesion of this nerve at the wrist produces hypothenar
wasting and clawing of the hand as a result of the action of unopposed long flexors; there is loss of
sensation over the little and ring fingers. Finger abduction is weak and the loss of thumb adduction
makes pinch difficult. Consequently, paralysis of the adductor pollicis and the first palmar interosseous
causes flexion of the thumb (due to flexor pollicis longus) – when the patient is asked to grasp a card
between his thumb and index finger (Froment’s sign).

21. Theme: Nerves of the upper limb


A Radial nerve
B Musculocutaneous nerve
---------------------------------------------------------------------------------------------------------------------------------------------------------------------------------------------------------------
Dr Mohammed Shamsul Islam Khan, Medical Officer, Clinical Neuro-Surgery, National Institute of Neuro-Sciences and Hospital
Sher-E-Bangla Nagar, Dhaka-1207, Bangladesh. Mobile: +880 1713 455 662, +880 1685 811979. E-mail: drsikhan@gmail.com
MyPasTest: MRCS A Online - Jan Exam 2015
22. Surgical Specialities; Trauma & Orthopaedic Surgery (76Qs)
----------------------------------------------------------------------------------------------------------------------------------
C Ulnar nerve
D Median nerve
E Axillary nerve.

For each of the following descriptions, select the most likely answer from the above list. Each option may
be used once, more than once, or not at all.
Page |
Scenario 1 1327
This nerve is formed in the axilla by the union of the lateral root from the lateral cord and the medial root from the
medial cord of the brachial plexus.
A - Radial nerve« YOUR ANSWER
D - Median nerve« CORRECT ANSWER.

The lateral root of the median nerve is the continuation of the lateral cord of the brachial plexus, and the
lateral root is joined by the medial root of the median nerve, lateral to the axillary artery to form the
median nerve.

Scenario 2
This nerve is the larger of the two terminal branches of the medial cord of the brachial plexus.
B - Musculocutaneous nerve« YOUR ANSWER
C - Ulnar nerve« CORRECT ANSWER.

The ulnar nerve is the larger of the two terminal branches of the medial cord of the brachial plexus, the
other terminal branch of the medial cord is the medial root of the median nerve.

Scenario 3
This nerve enters the arm posterior to the brachial artery, medial to the humerus, and anterior to the long head of
triceps.
C - Ulnar nerve« YOUR ANSWER
A - Radial nerve« CORRECT ANSWER.

The radial nerve is the direct continuation of the posterior cord of the brachial plexus and is the largest
branch of the brachial plexus. It enters the arm posterior to the brachial artery, medial to the humerus,
and anterior to the long head of the triceps.

Scenario 4
This nerve becomes the lateral cutaneous nerve of the forearm at the lateral border of the tendon of the biceps
brachii.
D - Median nerve« YOUR ANSWER
B - Musculocutaneous nerve« CORRECT ANSWER.

The musculocutaneous nerve supplies the muscles of the anterior aspect of the arm and is one of the two
terminal branches of the lateral cord of the brachial plexus. Just proximal to the elbow joint, the
musculocutaneous nerve pierces the deep fascia and becomes superficial, being now known as the
lateral cutaneous nerve of the forearm supplying the skin on the lateral aspect of the forearm.

22. Theme: Joints of the upper limb


A Saddle type of synovial joint
---------------------------------------------------------------------------------------------------------------------------------------------------------------------------------------------------------------
Dr Mohammed Shamsul Islam Khan, Medical Officer, Clinical Neuro-Surgery, National Institute of Neuro-Sciences and Hospital
Sher-E-Bangla Nagar, Dhaka-1207, Bangladesh. Mobile: +880 1713 455 662, +880 1685 811979. E-mail: drsikhan@gmail.com
MyPasTest: MRCS A Online - Jan Exam 2015
22. Surgical Specialities; Trauma & Orthopaedic Surgery (76Qs)
----------------------------------------------------------------------------------------------------------------------------------
B Pivot type of synovial joint
C Condyloid synovial joint
D Plane synovial joint
E Hinge type of synovial joint.

For each of the following joints, select the most likely answer from the above list. Each option may be
used once, more than once, or not at all. Page |
1328
Scenario 1
Radioulnar joint
A - Saddle type of synovial joint« YOUR ANSWER
B - Pivot type of synovial joint« CORRECT ANSWER.

The radius and ulna articulate with each other at their proximal and distal ends at synovial joints –
proximal and distal radioulnar joints. These are the pivot type of synovial joint and produce pronation and
supination of the forearm.

Scenario 2
Intercarpal joints
B - Pivot type of synovial joint« YOUR ANSWER
D - Plane synovial joint« CORRECT ANSWER.

These are the plane type of synovial joints which permit gliding and sliding movements.

Scenario 3
Metacarpophalangeal joints
C - Condyloid synovial joint« CORRECT ANSWER.

These joints are condyloid (knuckle-like) synovial joints and allow movement in two directions.

Scenario 4
Sternoclavicular joint
D - Plane synovial joint« YOUR ANSWER
A - Saddle type of synovial joint« CORRECT ANSWER.

This is a saddle type of synovial joint and is the only bony articulation between the axial skeleton and the
upper limb.

23. Theme: Shoulder joint pathologies


A Acromioclavicular joint osteoarthritis
B Acute supraspinatus tendonitis
C Calcific tendonitis
D Dislocated shoulder
E Fracture of surgical neck of humerus
F Frozen shoulder
G Painful arc syndrome
H Rotator cuff tear
I Rupture of long head of biceps
---------------------------------------------------------------------------------------------------------------------------------------------------------------------------------------------------------------
Dr Mohammed Shamsul Islam Khan, Medical Officer, Clinical Neuro-Surgery, National Institute of Neuro-Sciences and Hospital
Sher-E-Bangla Nagar, Dhaka-1207, Bangladesh. Mobile: +880 1713 455 662, +880 1685 811979. E-mail: drsikhan@gmail.com
MyPasTest: MRCS A Online - Jan Exam 2015
22. Surgical Specialities; Trauma & Orthopaedic Surgery (76Qs)
----------------------------------------------------------------------------------------------------------------------------------
J Subdeltoid bursitis.

For each of the following statements, select the most likely cause of shoulder pain from the above list.
Each option may be used once, more than once, or not at all.

Scenario 1
A 47-year-old lady presents with a 3-month history of painful shoulder. The pain is worse during the mid-phase of Page |
abduction and when bringing the hand down. There is no pain during the two extremes of movement. 1329
A - Acromioclavicular joint osteoarthritis« YOUR ANSWER
G - Painful arc syndrome« CORRECT ANSWER
G – Painful arc syndrome:
In painful arc syndrome (chronic supraspinatus tendonitis; impingement syndrome), there is pain on
abduction 60–120° (middle 1/3 of the arc), but the extremes of movements are painless. The underlying
pathology is the swelling of the tendon, and the pain is produced when it impinges on the undersurface of
the acromial process during the mid-phase of abduction. Repeating the movement with the arm in full
external rotation throughout may be much easier and relatively painless; this is virtually pathognomonic
of painful arc syndrome.

Scenario 2
A 60-year-old presents with a painful right shoulder after he fell off a tree 2 days ago. He has suffered from
chronic shoulder pain in the past. On examination, there is tenderness at the tip of his shoulder and underneath
the acromial process. He is unable to lift his arm and there is hunching of the shoulder.
B - Acute supraspinatus tendonitis« YOUR ANSWER
H - Rotator cuff tear« CORRECT ANSWER.

H – Rotator cuff tear:


Rotator cuff is a sheet of conjoint tendons (subscapularis, supraspinatus, infraspinatus and teres minor)
closely applied to the shoulder capsule and inserting into the greater and lesser tubercle of the humerus.
The differing clinical pictures stem from three basic pathological processes – degeneration, trauma and
vascular lesion. The supraspinatus tendon is liable to injury when it contracts against firm resistance;
this may occur when lifting a weight, or when the patient uses his or her arm to save themselves from
falling. This is much more likely if the cuff is already degenerate. The clinical presentation reflects the
loss of tendon function with weakness, a drop arm sign (characteristic hunching of the affected shoulder)
and even inability to lift the arm. There is often relentless night pain. On local palpation, pain is felt at the
shoulder-tip and upper arm and there is tenderness under the acromion.

Scenario 3
A 68-year-old woman presents to her GP with an acutely painful left shoulder. There is no history of trauma. On
examination, the shoulder joint is tender anterolaterally and there is restriction of all movements except external
rotation. X-ray reveals radio-opaque deposits within the supraspinatus tendon.
C - Calcific tendonitis« CORRECT ANSWER.

C – Calcific tendonitis:
Calcific tendonitis is a common disorder of unknown aetiology which results in an acutely painful
shoulder joint. Frequently, there is no history of trauma. Calcium becomes deposited within the
supraspinatus tendon and this may be part of a degenerative process. Clinical features include sudden
onset of pain with no apparent precipitating cause. Pain in usually felt over the anterolateral aspect of the
shoulder and is worse with overhead activities. On examination, the shoulder is tender anterolaterally,
---------------------------------------------------------------------------------------------------------------------------------------------------------------------------------------------------------------
Dr Mohammed Shamsul Islam Khan, Medical Officer, Clinical Neuro-Surgery, National Institute of Neuro-Sciences and Hospital
Sher-E-Bangla Nagar, Dhaka-1207, Bangladesh. Mobile: +880 1713 455 662, +880 1685 811979. E-mail: drsikhan@gmail.com
MyPasTest: MRCS A Online - Jan Exam 2015
22. Surgical Specialities; Trauma & Orthopaedic Surgery (76Qs)
----------------------------------------------------------------------------------------------------------------------------------
with some restriction of both active and passive movements. External rotation, however, is possible (this
feature differentiates the condition from frozen shoulder). X-ray reveals calcific deposits within the
supraspinatus tendon, inferior to the acromion and medial to the greater tibercle of the humerus.

Scenario 4
A 23-year-old rugby player presents with sudden shoulder pain after being involved in a tackle during the game.
On examination, there is loss of shoulder contour and a bulge is felt in the deltopectoral groove. Page |
D - Dislocated shoulder« CORRECT ANSWER. 1330
D – Dislocated shoulder:
Shoulder dislocation is common after trauma. Anterior dislocation is the commonest type (in contrast to
posterior dislocation in the hip joint). Shoulder dislocation may be associated with injury to the axillary
nerve which causes loss of sensation over the upper outer aspect of the deltoid region (‘badge’ area). The
round contour of the shoulder is lost because of the absence of the head of the humerus within the
glenoid fossa – the head of humerus may be felt in the deltopectoral groove (in anterior dislocation). Even
in clinically obvious dislocations, an X-ray should be performed to rule out an associated fracture.

24. Theme: Low back pain


A Discitis
B Facet joint arthrosis
C Metastatic disease
D Muscle strain
E Osteoporotic collapse
F Prolapsed intervertebral disc
G Spondylolisthesis.

For each of the clinical situations described below, please select the most likely diagnosis from the above
list. Each option may be used once, more than once, or not at all.

Scenario 1
A teenage boy presents to his GP with a 6-month history of increasing pain and stiffness in his lower lumbar
spine. There is no history of trauma although he is an active sportsman. His lumbar spine is stiff on examination
and there is bilateral limitation of straight leg raising with pain in the hamstring muscles.
A - Discitis« YOUR ANSWER
G - Spondylolisthesis« CORRECT ANSWER

G – Spondylolisthesis:
Spondylolisthesis commonly presents in the teenage years with pain and hamstring tightness. This is not
the same as a limited straight leg raising where the ‘strain’ is on the sciatic nerve.

Scenario 2
A young man wakes up one morning complaining of pain in his lower back. The day before he had been
redecorating his bedroom. There are no nerve root signs but his back is very stiff.
B - Facet joint arthrosis « YOUR ANSWER
D - Muscle strain« CORRECT ANSWER.

2 D – Muscle strain:
Muscle strains are the most common cause of low back pain in fit young adults, particularly if there has
---------------------------------------------------------------------------------------------------------------------------------------------------------------------------------------------------------------
Dr Mohammed Shamsul Islam Khan, Medical Officer, Clinical Neuro-Surgery, National Institute of Neuro-Sciences and Hospital
Sher-E-Bangla Nagar, Dhaka-1207, Bangladesh. Mobile: +880 1713 455 662, +880 1685 811979. E-mail: drsikhan@gmail.com
MyPasTest: MRCS A Online - Jan Exam 2015
22. Surgical Specialities; Trauma & Orthopaedic Surgery (76Qs)
----------------------------------------------------------------------------------------------------------------------------------
been some ‘unaccustomed’ exercise. A slipped disc is more likely to be associated with radiation of the
pain and symptoms of nerve root irritation.
Scenario 3
An active 88-year-old woman has had some back pain since she stumbled over her own doorstep 6 weeks ago.
Her back was initially stiff but she feels there has been some improvement over the last 2 weeks.
C - Metastatic disease « YOUR ANSWER Page |
E - Osteoporotic collapse« CORRECT ANSWER.
E – Osteoporotic collapse.
1331

Many 88-year-olds have osteoporosis and a simple fall can cause an osteoporotic fracture with collapse.
As the patient is improving, a more sinister explanation is unlikely.

Scenario 4
A 3-year-old baby girl refuses to walk. Her symptoms began last night and she is irritable and unwell. Examination
of her lower limbs is entirely normal but percussion of her lumbar spine causes her to cry.
D - Muscle strain« YOUR ANSWER
A - Discitis« CORRECT ANSWER.

A – Discitis:
In children who refuse to walk, a septic arthritis or osteomyelitis affecting the lower limbs must first be
excluded; only then may a diagnosis of discitis be considered.

Scenario 5
A 45-year-old man gives a 3-month history of low back pain, which is worse at night and has become
progressively worse such that he is now in agony and cannot get comfortable. The pain radiates to both buttocks.
He also complains of feeling generally unwell and thinks that he has lost weight and neurological examination
suggests there are problems with the sacral nerve roots.
E - Osteoporotic collapse« YOUR ANSWER
C - Metastatic disease « CORRECT ANSWER.

C – Metastatic disease:
Although the gentleman here is relatively young, the history of progressive pain radiating to the buttocks
with systemic symptoms and night pain must raise the concern of a metastatic lesion.

25. Theme: Dermatomes


A C4
B C5
C C6
D C7
E C8.

For each of the following descriptions, select the most likely answer from the above list. Each option may
be used once, more than once, or not at all.

Scenario 1
The area of the skin over the middle finger.
A - C4« YOUR ANSWER
---------------------------------------------------------------------------------------------------------------------------------------------------------------------------------------------------------------
Dr Mohammed Shamsul Islam Khan, Medical Officer, Clinical Neuro-Surgery, National Institute of Neuro-Sciences and Hospital
Sher-E-Bangla Nagar, Dhaka-1207, Bangladesh. Mobile: +880 1713 455 662, +880 1685 811979. E-mail: drsikhan@gmail.com
MyPasTest: MRCS A Online - Jan Exam 2015
22. Surgical Specialities; Trauma & Orthopaedic Surgery (76Qs)
----------------------------------------------------------------------------------------------------------------------------------
D - C7« CORRECT ANSWER.

The thumb and index finger are supplied by C6, the middle finger by C7, and the ring and little fingers by
C8.

Scenario 2
The area of the skin over the tip of the shoulder. Page |
B - C5« YOUR ANSWER 1332
A - C4« CORRECT ANSWER.

C4 supplies the infraclavicular region to the tip of the shoulder and above the scapular spine.

Scenario 3
The area of the skin over the ring finger.
C - C6« YOUR ANSWER
E - C8« CORRECT ANSWER.

C8 supplies the ring and little fingers, and distal medial forearm.

Scenario 4
The area of the skin over the little finger.
D - C7« YOUR ANSWER
E - C8« CORRECT ANSWER.

C8 supplies the ring and little fingers, and the distal medial forearm.

26. Theme: Complications of rheumatoid arthritis


A Anaemia
B Boutonnière’s deformity
C Bronchiolitis obliterans
D Carpal tunnel syndrome
E Felty’s syndrome
F Joint subluxation
G Mononeuritis
H Ocular disease
I Pericarditis
J Peripheral neuropathy
K Pleural effusion
L Renal disease
M Rheumatoid nodules
N Swan neck deformity
O Thrombocytosis
P Vascular lesions
Q Z deformity of the thumb.

Above is a list of potential complications of rheumatoid arthritis. For the following scenarios please pick
the most appropriate complication. Each item may be used once, more than once, or not at all.

---------------------------------------------------------------------------------------------------------------------------------------------------------------------------------------------------------------
Dr Mohammed Shamsul Islam Khan, Medical Officer, Clinical Neuro-Surgery, National Institute of Neuro-Sciences and Hospital
Sher-E-Bangla Nagar, Dhaka-1207, Bangladesh. Mobile: +880 1713 455 662, +880 1685 811979. E-mail: drsikhan@gmail.com
MyPasTest: MRCS A Online - Jan Exam 2015
22. Surgical Specialities; Trauma & Orthopaedic Surgery (76Qs)
----------------------------------------------------------------------------------------------------------------------------------
Scenario 1
A 60-year-old woman presents with worsening deformity of her hands. On examination you note flexion at the
proximal interphalangeal joint of the middle finger on her left hand, with hyperextension at the distal
interphalangeal joint. Her metacarpophalangeal joint is extended.
A - Anaemia « YOUR ANSWER
B - Boutonnière‟s deformity « CORRECT ANSWER.
Page |
B – Boutonnière deformity: 1333
This scenario describes the typical changes that occur with a Boutonnière deformity. These
transformations occur as a result of rupture of the middle slip of the extensor tendon. At this stage there
is no more than a failure to extend the proximal interphalangeal joint; however, if the tendon is not
repaired the lateral slips slide down towards the volar surface allowing the knuckle to ‘buttonhole’
through the extensor hood, causing the distal interphalangeal joint to be drawn into hyperextension.
Scenario 2
A 64-year-old gentleman with a chronic history of rheumatoid arthritis presents to the elderly day ward with a
history of malaise and lethargy. On examination he has splenomegaly with generalised lymphadenopathy; you
also notice some weeping leg ulcers. A full blood count shows a pancytopenia (white cell count 2.4 x 10 9/litre,
haemoglobin 10.5 g/dl, platelets 95 x 109/litre).
B - Boutonnière‟s deformity « YOUR ANSWER
E - Felty‟s syndrome « CORRECT ANSWER.

E – Felty’s syndrome:
This haematological complication of rheumatoid arthritis was named after its pioneer: Augustus Felty
(1934). The syndrome comprises seropositive (often high titres of rheumatoid factor) rheumatoid arthritis
(frequently with relatively inactive synovitis), splenomegaly and neutropenia. It usually occurs in
longstanding disease, and recurrent, severe infections are a common complication, as is vasculitis (leg
ulcers and mononeuritis), lymphadenopathy and pigmentation. Resultant hypersplenism may require
splenectomy.

Scenario 3
A 50-year-old man presents to The Emergency Department complaining of a progressive shortness of breath.
Previously fit and well, he now describes dyspnoea on mild exertion. On examination he has decreased breath
sounds at the left lung base and this region is dull to percussion.
C - Bronchiolitis obliterans « YOUR ANSWER
K - Pleural effusion« CORRECT ANSWER.
K – Pleural effusion:
Typically, pleural effusion is unilateral and arises in seropositive men over the age of 45 years. It often
precedes articular manifestations (arthritis), and unlike most other extra-articular complications, can
occur early on in the disease. The effusion can be chronic and associated with significant pleural
thickening. Other pulmonary manifestations include: rheumatoid nodules (less than 1% of cases) which
can cavitate and need to be differentiated from primary and metastatic malignancy; and bronchiolitis
obliterans, an illness characterised by rapid onset of breathlessness, progressing over a few months to
complete incapacity or death. Pulmonary function tests show considerable reduction in vital capacity,
with gross hyperinflation, although the chest X-ray is virtually normal. Interstitial disease occurs in 1–5%
of patients and generally progresses slowly, but may remain stable or even regress over the years.
Scenario 4

---------------------------------------------------------------------------------------------------------------------------------------------------------------------------------------------------------------
Dr Mohammed Shamsul Islam Khan, Medical Officer, Clinical Neuro-Surgery, National Institute of Neuro-Sciences and Hospital
Sher-E-Bangla Nagar, Dhaka-1207, Bangladesh. Mobile: +880 1713 455 662, +880 1685 811979. E-mail: drsikhan@gmail.com
MyPasTest: MRCS A Online - Jan Exam 2015
22. Surgical Specialities; Trauma & Orthopaedic Surgery (76Qs)
----------------------------------------------------------------------------------------------------------------------------------
A 48-year-old woman with known rheumatoid arthritis presents to outpatients complaining of pain and
paraesthesia to her thumb, forefinger and middle finger of her right hand. The symptoms have been progressive
and are worse at night. In clinic you are able to reproduce her symptoms using Phalen‟s test.
D - Carpal tunnel syndrome « CORRECT ANSWER.
D – Carpal tunnel syndrome:
Carpal tunnel syndrome is a nerve compression syndrome that is frequently associated with rheumatoid Page |
arthritis. Swelling, inflammation and tenosynovitis of the anterior tendons as they pass under the flexor
retinaculum cause compression of the median nerve with resultant tingling and pain in its region of 1334
distribution. Chronic pressure leads to wasting of the muscles supplied by the median nerve at this level
(LOAF – ie Lumbricals (first two), Opponens pollicis, Abductor pollicis brevis and Flexor pollicis brevis).
The causes should also be memorised.

27. Theme: Swollen painful joints


A Osteoarthritis
B Gout
C Rheumatoid arthritis
D Tuberculous arthritis
E Neuropathic joint disease.

For each of the patients described below, select the single most likely diagnosis from the options listed
above each option may be used once more, than once or not at all.

Scenario 1
A 35-year-old woman progressively develops pain, swelling and stiffness of her hands. On examination, 2 years
after the onset of her joint complaints, she is found to have swelling and tenderness in relation to the
metacarpophalangeal joints. X-rays of the affected joints show diminution of joint space, as well as osteoporosis
and marginal erosions of the articulating bones.
A - Osteoarthritis« YOUR ANSWER
C - Rheumatoid arthritis« CORRECT ANSWER.

Rheumatoid arthritis is associated with joint swelling, pain and stiffness. Common X-ray changes include
narrowing of joint space, marginal “mouse bite” erosions and cysts, as well as osteoporosis.

Scenario 2
A 60-year old woman complains of pain and swelling in both her knees of gradual onset over a period of 2 years.
On examination there is evidence of excess synovial fluid and synovial thickening in both knee joints and local
tenderness. Standing X-rays of her knees show diminution of joint space, sclerosis and cysts in the adjacent
bones. Osteophytes are also seen at the articular margins.
B - Gout« YOUR ANSWER
A - Osteoarthritis« CORRECT ANSWER.

This is typical of osteoarthritis, where you have joint tenderness, poor range of movements and effusions.
Typically on X-rays you will have loss of joint space, subchondral sclerosis and cysts and marginal
osteophytes.

28. A 2-month-old boy presents with a 12-hour history of fever, vomiting and pain on movement s of the
left hip. The ESR is 60, the CRP 120 and the WCC 30 x109/L. What is the most likely causative organism?
Select one answer only.
---------------------------------------------------------------------------------------------------------------------------------------------------------------------------------------------------------------
Dr Mohammed Shamsul Islam Khan, Medical Officer, Clinical Neuro-Surgery, National Institute of Neuro-Sciences and Hospital
Sher-E-Bangla Nagar, Dhaka-1207, Bangladesh. Mobile: +880 1713 455 662, +880 1685 811979. E-mail: drsikhan@gmail.com
MyPasTest: MRCS A Online - Jan Exam 2015
22. Surgical Specialities; Trauma & Orthopaedic Surgery (76Qs)
----------------------------------------------------------------------------------------------------------------------------------
Group A streptococcus« YOUR ANSWER
Enterobacteriaceae
Neisseria gonorrhoeae
Staphylococcus aureus« CORRECT ANSWER
Page |
Streptococcus pneumonia.
1335
The neonate in this scenario has septic arthritis of the hip, with raised inflammatory markers, white cell
count, and a pyrexia. A child of this age would not be weight bearing. The commonest organism causing
septic arthritis is staphylococcus aureus. There is some variation in causative organisms with age.
Neisseria gonorrhoeae is more common in sexually active adults, and streptococcus pneumoniae and
steptococcuspyogenes are more common in children between 3-months and 14-years, however, Staph
aureus still remains the most common cause in these groups.
29. Theme: Finger problems
A Trigger finger
B Volkmann's contracture
C Dupuytren's contracture
D Extensor pollicis longus (EPL) tendon rupture
E Abductor pollicis longus rupture
F Flexor pollicis tendon rupture
G de Quervain's disease.

Select the most appropriate diagnosis for each of the patients below. Each option may be used once,
more than once, or not at all.

Scenario 1
A 63-year-old man presented with painless inability to extend his thumb. He does not recall sustaining an injury to
his thumb; however, he does recall having a Colles‟ fracture treated 4 months ago.
A - Trigger finger« YOUR ANSWER
D - Extensor pollicis longus (EPL) tendon rupture« CORRECT ANSWER.

Delayed rupture of the extensor pollicis longus (EPL) tendon is a recognised complication after Colles’
fracture due to attrition at the radial (lister) tubercle.

Scenario 2
A 35-year-old lady presented to the outpatient department complaining of pain and swelling in the anatomical
snuff box with no history of trauma. On examination, flexion of her thumb is extremely painful and there is
thickening of the tendons in the anatomical snuff box.
B - Volkmann's contracture« YOUR ANSWER
G - de Quervain's disease« CORRECT ANSWER.
De Quervain's disease is a tenosynovitis of the extensor pollicis brevis and abductor pollicis longus
which commonly affects middle-aged women, causing pain on flexion of the thumb (and ulnar deviation at
the writs i.e. Finkelstein's test).
Scenario 3

---------------------------------------------------------------------------------------------------------------------------------------------------------------------------------------------------------------
Dr Mohammed Shamsul Islam Khan, Medical Officer, Clinical Neuro-Surgery, National Institute of Neuro-Sciences and Hospital
Sher-E-Bangla Nagar, Dhaka-1207, Bangladesh. Mobile: +880 1713 455 662, +880 1685 811979. E-mail: drsikhan@gmail.com
MyPasTest: MRCS A Online - Jan Exam 2015
22. Surgical Specialities; Trauma & Orthopaedic Surgery (76Qs)
----------------------------------------------------------------------------------------------------------------------------------
A 55-year-old gentleman with a known history of alcohol abuse presented to you complaining of inability to extend
his right ring and little fingers for the last 4 months that is progressively worsening. On examination you find
painless nodules on the palmar aspect of the ring and little finger bases.
C - Dupuytren's contracture« CORRECT ANSWER.
This is commonly idiopathic but may be attributed to alcohol abuse, trauma, diabetes and drugs.
Scenario 4
Page |
A 61-year-old lady known to have rheumatoid arthritis presented to the outpatient department complaining of pain 1336
and difficulty straightening her ring finger after making a fist; however, she manages to pull the finger straight but
with pain. On examination you find a tender nodule on the palmar aspect of the ring finger.
D - Extensor pollicis longus (EPL) tendon rupture« YOUR ANSWER
A - Trigger finger« CORRECT ANSWER.

This is tenosynovitis of the flexor tendon sheath which is trapped after flexion in the membranous sheath
at the proximal phalanx (A1 pulley). When the patient attempts to straighten it, it is painful and snaps back
once it passes the constrictive narrowing. Tenosynovitis is commonly found in rheumatoid arthritis;
however, the trigger finger can also be associated with gout and diabetes.

30. THEME: UPPER-LIMB NERVE INJURIES


A Posterior interosseous nerve
B Upper roots of the brachial plexus
C Musculocutaneous nerve
D Lower roots of the brachial plexus
E Anterior interosseous nerve
F Radial nerve
G Median nerve
H Suprascapular nerve
I Ulnar nerve
J Long thoracic nerve.

For each of the case descriptions below, select from the above list the most likely upper-limb nerve injury.
Each option may be used once, more than once, or not at all.

Scenario 1
A 25-year-old motorcyclist is brought to the Accident and Emergency Department following a road traffic accident
(RTA). He complains of pain in the root of his neck. On examination, his right arm is adducted, internally rotated
and extended at the elbow. He has loss of sensation along the outer aspects of the arm and the forearm.
A - Posterior interosseous nerve« YOUR ANSWER
B - Upper roots of the brachial plexus« CORRECT ANSWER.

Upper brachial plexus or supraclavicular plexus lesion (C5, C6) occurs due to excessive depression of the
shoulder or displacement of the head that opens out the angle between the shoulder and the neck (Erb-
Duchenne paralysis). In neonates, it may occur following traction on the shoulder girdle during difficult
labour or in breech delivery. In adults, it may occur due to a fall exerting weight on the shoulder or to an
RTA in which the head has been forcibly moved away from the shoulder. There is loss of shoulder
abduction, elbow flexion and forearm supination. Consequently, the affected limb is internally rotated,
extended at the elbow and pronated (porter’s tip position). There is sensory loss over the outer aspect of
the arm and the outer aspect of the forearm.
---------------------------------------------------------------------------------------------------------------------------------------------------------------------------------------------------------------
Dr Mohammed Shamsul Islam Khan, Medical Officer, Clinical Neuro-Surgery, National Institute of Neuro-Sciences and Hospital
Sher-E-Bangla Nagar, Dhaka-1207, Bangladesh. Mobile: +880 1713 455 662, +880 1685 811979. E-mail: drsikhan@gmail.com
MyPasTest: MRCS A Online - Jan Exam 2015
22. Surgical Specialities; Trauma & Orthopaedic Surgery (76Qs)
----------------------------------------------------------------------------------------------------------------------------------
Scenario 2
A 40-year-old woman presents to the Accident and Emergency Department with carpal dislocation (confirmed
radiologically). On examination, she has loss of sensation over the thumb and the index and middle fingers.
B - Upper roots of the brachial plexus« YOUR ANSWER
G - Median nerve« CORRECT ANSWER.
Page |
In adults, the median nerve (C5–T1) is commonly injured near the wrist, although it can be injured 1337
anywhere along the upper arm or the forearm. Distal lesions may be caused by lacerations to the front of
the wrist or by carpal dislocation. In children, supracondylar fractures of the humerus may lead to median
nerve injury at the elbow. Median nerve injuries at the wrist causes sensory loss over the thumb and the
index and middle fingers and occasionally the lateral half of the ring finger; motor loss includes all thenar
muscles except the adductor pollicis (which is supplied by the ulnar nerve) and the lateral two lumbricals.
If the injury is at the level of the elbow, there is paralysis of the pronators of the forearm and the flexors of
the wrist and fingers, with the exception of the flexor carpi ulnaris and the medial part of the flexor
digitorum profundus.

Scenario 3
A 65-year-old man is brought to the Accident and Emergency Department with a wrist drop and sensory loss over
a small patch at the base of the thumb. X-rays shows a mid-humerus fracture.
C - Musculocutaneous nerve« YOUR ANSWER
F - Radial nerve« CORRECT ANSWER.

The radial nerve (C5–T1) is damaged at the mid-humerus level by fractures or pressure (eg prolonged
tourniquet). Radial nerve damage is also seen in patients who fall asleep with an arm dangling over the
back of a chair (Saturday night palsy). Radial nerve injury causes paralysis of the brachioradialis, the
wrist extensors and the extensor digitorum, leading to wrist drop; there may be a small patch of sensory
loss over the dorsum of the thumb and the first web space. With more proximal lesions, sensation is also
lost over the dorsum of the forearm.

Scenario 4
A 30-year-old man presents to the Accident and Emergency Department with a deep laceration to his right wrist
after he was involved in a fight in his local pub. On examination, he is unable to pinch and has loss of sensation
over his little and ring fingers.
D - Lower roots of the brachial plexus« YOUR ANSWER
I - Ulnar nerve« CORRECT ANSWER.

The ulnar nerve (C8, T1) is an important motor nerve of the hand. Pressure (eg from a deep ganglion) or
laceration at the wrist may cause distal lesions. Injury to this nerve at the level of the wrist produces
hypothenar wasting and clawing of the hand due to unopposed action of the long flexors; there is loss of
sensation over the little and ring fingers. Finger abduction is weak and the loss of thumb adduction
makes pinching difficult. This is due to paralysis of the adductor pollicis and the first palmar interosseous
muscle causing flexion of the thumb (due to the flexor pollicis longus) when the patient is asked to grasp
a card between their thumb and index finger (Froment’s paper sign).
31. Theme: Shoulder pain
A Clavicle fracture
B Frozen shoulder
C Septic arthritis
---------------------------------------------------------------------------------------------------------------------------------------------------------------------------------------------------------------
Dr Mohammed Shamsul Islam Khan, Medical Officer, Clinical Neuro-Surgery, National Institute of Neuro-Sciences and Hospital
Sher-E-Bangla Nagar, Dhaka-1207, Bangladesh. Mobile: +880 1713 455 662, +880 1685 811979. E-mail: drsikhan@gmail.com
MyPasTest: MRCS A Online - Jan Exam 2015
22. Surgical Specialities; Trauma & Orthopaedic Surgery (76Qs)
----------------------------------------------------------------------------------------------------------------------------------
D Anterior dislocation
E Posterior dislocation
F Acromioclavicular joint osteoarthritis
G Osteoarthritis
H Supraspinatous tendon rupture
I Supraspinatous tendonitis.
Page |
Select the most appropriate diagnosis for each of the patients below. Each option may be used once,
more than once, or not at all. 1338
Scenario 1
A 65-year-old joiner presented with left shoulder pain especially at night, usually relieved by paracetamol.
Movement is moderately restricted in all directions, however when he raises his arm above his head the shoulder
is mainly painful during the last degrees of abduction. X-ray shows new bone formation at the glenoid edges.
A - Clavicle fracture« YOUR ANSWER
G - Osteoarthritis« CORRECT ANSWER.

Pain during the last degrees of shoulder abduction is typical of osteoarthritis, particularly in the presence
of osteophytes on X-rays.

Scenario 2
A 45-year-old joiner presented with pain in his left shoulder which occasionally responds to simple analgesia. The
pain bothers him mainly during work. On examination, abduction of the shoulder is painful during 60–120 degrees.
When lowering the arm it suddenly drops halfway during adduction. X-rays are normal.
B - Frozen shoulder« YOUR ANSWER
I - Supraspinatous tendonitis« CORRECT ANSWER.

Supraspinatous tendonitis, usually resulting occupations with shoulder overuse, is characterised by pain
and impingement during 60–120 degrees of abduction.

Scenario 3
A 53-year-old lady suffering from NIDDM ( non-insulin-dependent diabetes mellitus) for the last 15 years
presented with pain and stiffness in her right shoulder of few months duration. Examination reveals restriction of
movement in all directions. Blood investigations show normal inflammatory markers. X-ray of the shoulder does
not reveal any abnormality. Culture of joint aspirate did not grow any organisms.
C - Septic arthritis« YOUR ANSWER
B - Frozen shoulder« CORRECT ANSWER.

Frozen shoulder is common in middle-aged women especially diabetics. X-rays do not show any
abnormality as the main pathology is adhesive capsulitis which can be released by arthroscopy. As septic
arthritis still need to be excluded, where blood results can be unreliable then the gold standard is culture
of joint aspirate.

Scenario 4
A 36-year-old electrician is an inpatient after sustaining an electric shock at work. He is complaining of left
shoulder pain and stiffness. On examination you find any movement of the shoulder is not possible, and it is is
held in adduction. AP view X-ray of the shoulder does not reveal any abnormality.
D - Anterior dislocation« YOUR ANSWER
E - Posterior dislocation« CORRECT ANSWER.
---------------------------------------------------------------------------------------------------------------------------------------------------------------------------------------------------------------
Dr Mohammed Shamsul Islam Khan, Medical Officer, Clinical Neuro-Surgery, National Institute of Neuro-Sciences and Hospital
Sher-E-Bangla Nagar, Dhaka-1207, Bangladesh. Mobile: +880 1713 455 662, +880 1685 811979. E-mail: drsikhan@gmail.com
MyPasTest: MRCS A Online - Jan Exam 2015
22. Surgical Specialities; Trauma & Orthopaedic Surgery (76Qs)
----------------------------------------------------------------------------------------------------------------------------------
Less common than anterior dislocation, posterior dislocation common takes place after seizures and
electrocution. Anteroposterior view may not show any abnormality. A scapulohumeral view will show the
posterior dislocation.

32. Theme: Upper limb neurological injuries


A Ulnar nerve
B Median nerve Page |
C Radial nerve 1339
D Axillary nerve.

Match the nerve from the option list above which when injured would result in the clinical presentations
described below. Each option may be used once only, more than once or not at all.
Scenario 1
Paraesthesia over first dorsal webspace of hand
A - Ulnar nerve« YOUR ANSWER
C - Radial nerve« CORRECT ANSWER.

Scenario 2
Finger adduction weakened
B - Median nerve« YOUR ANSWER
A - Ulnar nerve« CORRECT ANSWER.

Scenario 3
Paraesthesia over the deltoid muscle
C - Radial nerve« YOUR ANSWER
D - Axillary nerve« CORRECT ANSWER.

Scenario 4
Anaesthesia of the little finger
D - Axillary nerve« YOUR ANSWER
A - Ulnar nerve« CORRECT ANSWER.

Scenario 5
At risk from a fracture of the medial epicondyle of the humerus
A - Ulnar nerve« CORRECT ANSWER.

Scenario 6
Paralysis of the thenar muscles
B - Median nerve« CORRECT ANSWER.

Scenario 7
At risk from cannulation of blood vessels in the antecubital fossa
C - Radial nerve« YOUR ANSWER
B - Median nerve« CORRECT ANSWER.

The ulnar nerve innervates the intrinsic muscles of the hand and provides sensory innervation to the
medial 1½ fingers.The median nerve provides sensory innervation to the lateral 2½ fingers (and thumb) of
the hand and supplies most of the forearm flexor muscles and muscles of the thumb. The ulnar nerve
---------------------------------------------------------------------------------------------------------------------------------------------------------------------------------------------------------------
Dr Mohammed Shamsul Islam Khan, Medical Officer, Clinical Neuro-Surgery, National Institute of Neuro-Sciences and Hospital
Sher-E-Bangla Nagar, Dhaka-1207, Bangladesh. Mobile: +880 1713 455 662, +880 1685 811979. E-mail: drsikhan@gmail.com
MyPasTest: MRCS A Online - Jan Exam 2015
22. Surgical Specialities; Trauma & Orthopaedic Surgery (76Qs)
----------------------------------------------------------------------------------------------------------------------------------
passes over the medial epicondyle of the humerus and the median nerve in close proximity to the brachial
artery.

33. THEME: SHOCK


A Fat embolism
B Thromboembolism Page |
C Cardiogenic shock
D Hypovolaemic shock. 1340
For each of the patients described below, select the single most likely diagnosis from the options listed
above. Each option may be used once, more than once, or not at all.

Scenario 1
A 26-year-old man with a comminuted closed fracture of the femur shaft undergoes intra-medullary nail fixation.
Two days after the operation, he develops a pyrexia, shortness of breath, petechial haemorrhages on his chest
and tachycardia.
A - Fat embolism« CORRECT ANSWER.

Fat embolism occurs in patients with multiple closed fractures, but has been reported with other skeletal
trauma, including burns. Fat embolism causes a temperature, tachycardia, shortness of breath, confusion
and petechial haemorrhages, especially on the chest.

Scenario 2
A 72-year-old man with an underlying prostate carcinoma and a fractured femoral shaft undergoes intra-medullary
nail fixation. Seven days after the operation, he develops shortness of breath, hypotension and tachycardia.
B - Thromboembolism« CORRECT ANSWER.

Seven days post-operation is the characteristic time for a thromboembolism.

Scenario 3
A 60-year-old man develops sudden back pain. He is brought to Accident and Emergency (A&E) with a swollen
tense abdomen, tachycardia, low volume pulse and low BP.
C - Cardiogenic shock« YOUR ANSWER
D - Hypovolaemic shock« CORRECT ANSWER.

In a patient with a sudden onset of back pain, a swollen tense abdomen and low BP, a ruptured aortic
aneurysm is the most common diagnosis. This leads to hypovolaemic shock.

34. Theme: Leg Pain


A Acute limb ischaemia
B Superficial thrombophlebitis
C Fractured tibia
D Ruptured Baker's cyst
E Gastrocnemius tears
F Complete Achilles' tendon rupture
G Chronic venous insufficiency
H Deep venous thrombosis (DVT)
I Intervertebral disc (IVD) prolapsed.
---------------------------------------------------------------------------------------------------------------------------------------------------------------------------------------------------------------
Dr Mohammed Shamsul Islam Khan, Medical Officer, Clinical Neuro-Surgery, National Institute of Neuro-Sciences and Hospital
Sher-E-Bangla Nagar, Dhaka-1207, Bangladesh. Mobile: +880 1713 455 662, +880 1685 811979. E-mail: drsikhan@gmail.com
MyPasTest: MRCS A Online - Jan Exam 2015
22. Surgical Specialities; Trauma & Orthopaedic Surgery (76Qs)
----------------------------------------------------------------------------------------------------------------------------------
Select the most appropriate diagnosis for each of the patients below. Each option may be used once,
more than once, or not at all.

Scenario 1
A 65-year-old presented with a 3-day history of painful left leg and foot which is gradually worsening. On
examination you see erythema over the dorsum of the foot extending to the shin of the leg; you also feel a firm Page |
tender cord-like vessel. There is an ulcer between the 3rd and 4th toes. The calf feels soft and not tender. 1341
A - Acute limb ischaemia« YOUR ANSWER
B - Superficial thrombophlebitis« CORRECT ANSWER.

Superficial thrombophlebitis can occur spontaneously, but can also be associated with trauma, ulcers or
even associated with a silent deep vein thrombosis (DVT.
Scenario 2
A 25-year-old male sustained a sudden painful left leg particularly on the back of the calf while playing badminton.
On examination you find that plantar flexion power of the foot is only mildly affected; however it aggravates the
pain. There is also a localised tenderness on the back of the leg approximately 5 cm below the knee joint.
B - Superficial thrombophlebitis« YOUR ANSWER
E - Gastrocnemius tears« CORRECT ANSWER.

Gastrocnemius tears are common sport injuries. Here plantar flexion is preserved so Achilles’ tendon
rupture is ruled out (not partial ones though), and the localised muscular tenderness up the calf favours
the diagnosis.

Scenario 3
A 76-year-old demented lady was referred from a residential home complaining of increasingly painful right leg
over the last 3 days. She had bilateral hemiarthroplasty and her mobility is restricted to bed and chair. On
examination her right leg is swollen, with shiny skin, erythematous, and tender, and dorsiflexion aggravates the
pain. She also has a low-grade fever.
C - Fractured tibia« YOUR ANSWER
H - Deep venous thrombosis (DVT)« CORRECT ANSWER.

Immobility (bilateral hemiarthroplasty) is a major factor contributing to this lady’s DVT. Homan’s sign
(pain on foot dorsiflexion) is said to be detrimental as it carries a theoretical risk of dislodging thrombi.

Scenario 4
An 83-year-old gentleman presented with an increasingly painful left leg over the last 5 hours and feeling
weakness in that leg. On examination you find a pale, cold left leg, power 2/5 MRC (Medical Research Council
grading system) but distal pulses are present.
D - Ruptured Baker's cyst« YOUR ANSWER
A - Acute limb ischaemia« CORRECT ANSWER.

You do not have to have all the Ps (pallor, paralysis, parasthesia, pain, pulsles) to diagnose acute limb
ischaemia. Absent pulses is a very late sign. Muscle weakness is an important sign of critical ischaemia
that needs urgent intervention.

35. Theme: Nerve injury


A Axillary nerve
---------------------------------------------------------------------------------------------------------------------------------------------------------------------------------------------------------------
Dr Mohammed Shamsul Islam Khan, Medical Officer, Clinical Neuro-Surgery, National Institute of Neuro-Sciences and Hospital
Sher-E-Bangla Nagar, Dhaka-1207, Bangladesh. Mobile: +880 1713 455 662, +880 1685 811979. E-mail: drsikhan@gmail.com
MyPasTest: MRCS A Online - Jan Exam 2015
22. Surgical Specialities; Trauma & Orthopaedic Surgery (76Qs)
----------------------------------------------------------------------------------------------------------------------------------
B Intercostal nerve
C Median nerve
D Radial nerve
E Ulnar nerve.

Match the most appropriate site of damage from the list above to each clinical situation described below.
Each option may be used once only, more than once or not at all. Page |
1342
Scenario 1
A rugby player who has some loss of sensation over the deltoid muscle following dislocation of his shoulder joint.
A - Axillary nerve« CORRECT ANSWER.

Scenario 2
A teenager who has a fractured tibia and is experiencing increasing difficulty using her axilla crutch.
B - Intercostal nerve« YOUR ANSWER
D - Radial nerve« CORRECT ANSWER.

Scenario 3
A child who has a weak grip and impaired sensation involving the ring and little fingers following dislocation of his
elbow joint which was treated by manipulation.
C - Median nerve« YOUR ANSWER
E - Ulnar nerve« CORRECT ANSWER.

Scenario 4
A child with a supracondylar fracture of his humerus who cannot oppose his thumb.
D - Radial nerve« YOUR ANSWER
C - Median nerve« CORRECT ANSWER.

The axillary nerve lies close to the surgical neck of the neck of the humerus and may be damaged by
dislocation of the shoulder joint. The nerve supplies skin over the lower part of the deltoid (the regimental
badge area) as well as the deltoid muscle itself. Improper use of acilla crutches may damage the radial
nerve in the axilla resulting in weakness of the triceps and of the muscles in the posterior compartment of
the forearm, causing reduced power of extension at the elbow and instability at the wrist with impaired
grip. Both the ulnar and median nerves may be damaged by a fracture or dislocation at the elbow joint.
36. A 25-year-old man who was playing football yesterday attends the emergency department, he
sustained a blow to the lateral aspect of the tibia whilst his foot was planted and the knee flexed to about
20o. He was unable to play on or weight bear, and he developed a tense effusion in his knee within an
hour. What is the most likely pattern of injury? Select one answer only.
Lateral collateral ligament rupture, lateral meniscal tear, and anterior cruciate ligament rupture« YOUR
ANSWER
Lateral collateral ligament rupture, medial meniscal tear, and anterior cruciate ligament rupture
Lateral meniscal tear, anterior cruciate ligament rupture and posterior cruciate ligament rupture
Medial collateral ligament rupture, lateral meniscal tear, and anterior cruciate ligament rupture

---------------------------------------------------------------------------------------------------------------------------------------------------------------------------------------------------------------
Dr Mohammed Shamsul Islam Khan, Medical Officer, Clinical Neuro-Surgery, National Institute of Neuro-Sciences and Hospital
Sher-E-Bangla Nagar, Dhaka-1207, Bangladesh. Mobile: +880 1713 455 662, +880 1685 811979. E-mail: drsikhan@gmail.com
MyPasTest: MRCS A Online - Jan Exam 2015
22. Surgical Specialities; Trauma & Orthopaedic Surgery (76Qs)
----------------------------------------------------------------------------------------------------------------------------------
Medial collateral ligament rupture, medial meniscal tear, and anterior cruciate ligament rupture« CORRECT
ANSWER.

The patient in this scenario has suffered a significant ligamentous injury to the knee as is evident from
the rapid large haemarthrosis, and inability to weight bear immediately post-injury. An immediate tense
haemarthrosis is most likely with anterior cruciate ligament rupture or fracture/dislocation. The medial Page |
collateral ligament is more commonly injured than the lateral, as lateral blows to the knee are more
frequent than medial. When the medial collateral ligament ruptures, injury to the medial meniscus is 1343
commonly associated as the meniscus is attached to the medial collateral ligament. If the blow to the leg
is severe the medial collateral ligament may completely rupture, the meniscus tear and the force continue
to rupture the anterior cruciate ligament.

37. A 37-year-old man attends the emergency department having sustained a laceration to the left hand on
a piece of broken glass. On examination there is a deep laceration on the volar surface lying 1cm
proximal to the distal palmar crease. Which flexor tendon injury zone does this injury lie in? Select one
answer only.
Zone 1« YOUR ANSWER
Zone 2
Zone 3« CORRECT ANSWER
Zone 4
Zone 5.

There are five zones of injury described by Verden for flexor tendons, based on anatomic factors and
prognosis.

The five zones are:


Zone 1 – Between the DIP and PIP joint creases
Zone 2 – Between the mid point of the middle phalanx and distal palmar crease
Zone 3 – Between the distal palmar crease and the distal margin of the carpal tunnel
Zone 4 – Overlying the carpal tunnel
Zone 5 – The forearm and wrist up to the proximal border of the carpal tunnel.

38. Theme: Glasgow Coma Score (GCS)


A 10
B 11
C 12
D9
E 8.

Pick the correct GCS for the patients described below. Each option may be used once only, more than
once or not at all.

Scenario 1
A young child has fallen from a swing and hit his head. His eyes are opening to speech, localising to pain and he
is mumbling incomprehensible sounds.
---------------------------------------------------------------------------------------------------------------------------------------------------------------------------------------------------------------
Dr Mohammed Shamsul Islam Khan, Medical Officer, Clinical Neuro-Surgery, National Institute of Neuro-Sciences and Hospital
Sher-E-Bangla Nagar, Dhaka-1207, Bangladesh. Mobile: +880 1713 455 662, +880 1685 811979. E-mail: drsikhan@gmail.com
MyPasTest: MRCS A Online - Jan Exam 2015
22. Surgical Specialities; Trauma & Orthopaedic Surgery (76Qs)
----------------------------------------------------------------------------------------------------------------------------------
A - 10« CORRECT ANSWER.

Scenario 2
A young man has been attacked with a baseball bat and has a palpable fracture through a wound in his scalp. His
eyes are opening to pain, he is withdrawing to pain and saying inappropriate words.
B - 11« YOUR ANSWER
D - 9« CORRECT ANSWER. Page |
1344
The Glasgow Coma Score (GCS) documents the depth of coma by assessment of the best verbal, motor
and eye responses to stimulation.Eye opening: spontaneous (4), to speech (3), to pain (2) absent (1).Best
motor response: obeys commands (6), localises to pain (5), withdraws to pain (4), abnormal flexion (3),
extension (2), no response (1).Best verbal response: oriented (5) confused (4), inappropriate words (3),
incomprehensible sounds (2), no verbal response (1).

39. Theme: Shock


A Cardiogenic shock
B Septic shock
C Neurogenic shock
D Hypovolaemic shock <15% volume loss
E Hypovolaemic shock 15–30% volume loss
F Hypovolaemic shock >40% volume loss.

For each of the patients described below choose the most appropriate variety of shock from the list
above. Each may be used once, more than once or not at all.

Scenario 1
A motorcyclist was admitted to the emergency department having been thrown a distance of 30 feet. He was
wearing a helmet and had recovered consciousness at the scene. He remained confused. Pulse 40 bpm,
respiratory rate 30/min, blood pressure (BP) 75/60 mmHg. Lateral cervical spine X-rays were suggestive of a
fracture of C6.
A - Cardiogenic shock« YOUR ANSWER
C - Neurogenic shock« CORRECT ANSWER.

C-Neurogenic shock:
Neurogenic shock is due to the loss of sympathetic tone and combines the symptoms characteristic of
hypovolaemic shock with a profound bradycardia.

Scenario 2
A 25-year-old marathon runner was involved in a road traffic accident while out training. She was admitted to the
emergency department with a pulse of 100 bpm, BP 75/60 mmHg, respiratory rate 30/min. Her abdomen was
generally tender, peritoneal lavage was positive. There was no urine output.
B - Septic shock« YOUR ANSWER
F - Hypovolaemic shock >40% volume loss« CORRECT ANSWER.

Scenario 3
A homeless gentleman was found collapsed in a dark alleyway at 5am on New Year‟s Day. He was unconscious
and smelt of alcohol. On arrival in the emergency department his pulse was 110/min, BP 115/50 mmHg, he was
---------------------------------------------------------------------------------------------------------------------------------------------------------------------------------------------------------------
Dr Mohammed Shamsul Islam Khan, Medical Officer, Clinical Neuro-Surgery, National Institute of Neuro-Sciences and Hospital
Sher-E-Bangla Nagar, Dhaka-1207, Bangladesh. Mobile: +880 1713 455 662, +880 1685 811979. E-mail: drsikhan@gmail.com
MyPasTest: MRCS A Online - Jan Exam 2015
22. Surgical Specialities; Trauma & Orthopaedic Surgery (76Qs)
----------------------------------------------------------------------------------------------------------------------------------
apyrexial on admission, but his skin was noted to be flushed. During the secondary survey he was found to have
sustained a penetrating abdominal injury, which looked a few days old.
C - Neurogenic shock« YOUR ANSWER
B - Septic shock« CORRECT ANSWER.

B-Septic shock:
Generally in hypovolaemic shock <15% loss leads to anxiety, no change in pulse, blood pressure (BP) or Page |
respiratory rate, urine output is maintained; 15–30% loss leads to anxiety. The BP is maintained, the pulse 1345
generally 100 bpm and the pulse pressure reduced, urine output is maintained at 20–30 ml/h.; 40% loss
causes confusion. The pulse is generally greater than 140/min, the BP and pulse pressure fall. There is no
urine output. Fit athletic individuals may initially compensate for massive blood loss causing
hypovolaemia maintaining pulse and BP, then rapidly decompensate. In cases of delayed presentation
septic shock should be considered. This is similar in presentation to hypovolaemic shock but
characterised by a wide pulse pressure. Patients who are hypothermic from exposure may initially appear
to be apyrexial.

40. Theme: Cutaneous sensation


A The median nerve
B The radial nerve
C The dorsal cutaneous branch of the ulnar nerve
D The superficial branch of the ulnar nerve
E The palmar cutaneous branch of the ulnar nerve.

For each of the following descriptions, select the most likely answer from the above list. Each option may
be used once, more than once, or not at all.

Scenario 1
The sensory innervation of the nail bed of the index finger.
A - The median nerve« CORRECT ANSWER.

The median nerve supplies the lateral 3½ digits of the hand on the palmar aspect, and the tips of the
lateral 3 and a half digits on the dorsal aspect.

Scenario 2
The sensory innervation of the medial side of the palm.
B - The radial nerve« YOUR ANSWER
E - The palmar cutaneous branch of the ulnar nerve« CORRECT ANSWER.

The ulnar nerve enters the palm anterior to the flexor retinaculum, and as it crosses the flexor retinaculum
it divides into a superficial and a deep terminal branch. The palmar cutaneous branch of the ulnar nerve is
given off in the front of the forearm anterior to the flexor retinaculum, and supplies the skin over the
medial aspect of the palm.

Scenario 3
The sensory innervation of the dorsal surface of the base of the thumb.
C - The dorsal cutaneous branch of the ulnar nerve« YOUR ANSWER
B - The radial nerve« CORRECT ANSWER.

---------------------------------------------------------------------------------------------------------------------------------------------------------------------------------------------------------------
Dr Mohammed Shamsul Islam Khan, Medical Officer, Clinical Neuro-Surgery, National Institute of Neuro-Sciences and Hospital
Sher-E-Bangla Nagar, Dhaka-1207, Bangladesh. Mobile: +880 1713 455 662, +880 1685 811979. E-mail: drsikhan@gmail.com
MyPasTest: MRCS A Online - Jan Exam 2015
22. Surgical Specialities; Trauma & Orthopaedic Surgery (76Qs)
----------------------------------------------------------------------------------------------------------------------------------
The superficial branch of the radial nerve descends over the extensor retinaculum and supplies the lateral
two-thirds of the dorsum of the hand. It divides into a number of dorsal digital nerves which supply the
dorsal surface of the thumb, the dorsal surface of the index finger and dorsal surface of the lateral side of
the middle finger.

Scenario 4
The sensory innervation of the medial side of the palmar surface of the ring finger. Page |
D - The superficial branch of the ulnar nerve« CORRECT ANSWER. 1346
The superficial branch of the ulnar nerve descends into the palm and gives off a muscular branch to the
palmaris brevis, and cutaneous branches to the palmar aspect of the medial side of the little finger and
the adjacent sides of the little and ring fingers.

41. An 80-year-old lady is brought to the emergency department following a fall, her right leg is shortened
and externally rotated. An AP x-ray shows an intracapsular fractured neck of femur which extends across
the neck completely, the trabecular lines are interrupted, but they are not angulated. How would you
classify this fracture? Select one answer only.
Garden I« YOUR ANSWER
Garden II« CORRECT ANSWER
Garden III
Garden IV
Garden V.

Garden classified intracapsular hip fractures based on the trabecular lines on the AP x-ray. The
classification can be used to assist in guiding treatment, along with the patients’ age and physiological
status.

Garden described 4 types, I-IV, which are:


Garden I – Incomplete fracture, impacted, medial cortex intact, trabecular lines angulated
Garden II – Complete fracture, undisplaced, trabecular lines interrupted but not angulated
Garden III – Complete fracture, partially displaced, trabeculae angulated
Garden IV – Complete fracture with complete displacement.

42. Theme: Pelvic fracture


A Rotationally and vertically stable
B Rotationally unstable, vertically stable pelvic fracture
C Rotationally unstable, vertically unstable pelvic fracture.

For each of the pelvic injuries below, select the correct classification of pelvic fracture from the above list.
Each option may be used once, more than once, or not at all.

Scenario 1
Lateral compression fracture
A - Rotationally and vertically stable« YOUR ANSWER
B - Rotationally unstable, vertically stable pelvic fracture« CORRECT ANSWER.

---------------------------------------------------------------------------------------------------------------------------------------------------------------------------------------------------------------
Dr Mohammed Shamsul Islam Khan, Medical Officer, Clinical Neuro-Surgery, National Institute of Neuro-Sciences and Hospital
Sher-E-Bangla Nagar, Dhaka-1207, Bangladesh. Mobile: +880 1713 455 662, +880 1685 811979. E-mail: drsikhan@gmail.com
MyPasTest: MRCS A Online - Jan Exam 2015
22. Surgical Specialities; Trauma & Orthopaedic Surgery (76Qs)
----------------------------------------------------------------------------------------------------------------------------------
B – Rotationally unstable, vertically stable pelvic fracture:
Rotationally unstable, vertically stable pelvic fractures include open-book fractures (pubic diastasis > 2.5
cm) and lateral compression fractures.

Scenario 2
Open-book fracture
B - Rotationally unstable, vertically stable pelvic fracture« CORRECT ANSWER. Page |
1347
B – Rotationally unstable, vertically stable pelvic fracture
Rotationally unstable, vertically stable pelvic fractures include open-book fractures (pubic diastasis >2.5
cm) and lateral compression fractures.

Scenario 3
Vertical shear injuries
C - Rotationally unstable, vertically unstable pelvic fracture« CORRECT ANSWER
C – Rotationally unstable, vertically unstable pelvic fracture:
Rotationally and vertically unstable pelvic fractures usually result from vertical shear injuries.

Scenario 4
Isolated iliac wing fracture
D - « YOUR ANSWER
A - Rotationally and vertically stable« CORRECT ANSWER.

A – Rotationally and vertically stable:


Stable pelvic fractures include fractures not displacing the pelvic ring (such as avulsion fractures and
isolated fractures of the iliac wing or pubic ramus) or minimally displaced fractures of the pelvic ring.

Scenario 5
Isolated pubic ramus fracture
A - Rotationally and vertically stable« CORRECT ANSWER.

A – Rotationally and vertically stable:


Stable pelvic fractures include fractures not displacing the pelvic ring (such as avulsion fractures and
isolated fractures of the iliac wing or pubic ramus) or minimally displaced fractures of the pelvic ring.

43. Theme: The childhood limp


A Acquired dislocation of the hip
B Congenital dislocation of the hip
C Congenital subluxation
D Non-specific transient synovitis
E Perthes’ disease
F Pyogenic arthritis
G Slipped upper femoral epiphysis
H Still’s disease
I Tuberculous synovitis.

The following are descriptions of children who have presented with a limp. Please select the most
appropriate diagnosis from the above list. The items may be used once, more than once, or not at all.
---------------------------------------------------------------------------------------------------------------------------------------------------------------------------------------------------------------
Dr Mohammed Shamsul Islam Khan, Medical Officer, Clinical Neuro-Surgery, National Institute of Neuro-Sciences and Hospital
Sher-E-Bangla Nagar, Dhaka-1207, Bangladesh. Mobile: +880 1713 455 662, +880 1685 811979. E-mail: drsikhan@gmail.com
MyPasTest: MRCS A Online - Jan Exam 2015
22. Surgical Specialities; Trauma & Orthopaedic Surgery (76Qs)
----------------------------------------------------------------------------------------------------------------------------------
Scenario 1
A 14-year-old boy is brought to The Emergency Department by his mother. She is worried as he fell 4 days ago
and has continued to limp since. He gives a 6-month history of pain in his right knee, which has worsened since
falling over. On examination he is considerably overweight and has evidence of delayed puberty. His right leg is
externally rotated and appears to be shorter than the left. Assessment of limb movements is not possible because
of excessive pain in the limb. Page |
A - Acquired dislocation of the hip« YOUR ANSWER 1348
G - Slipped upper femoral epiphysis« CORRECT ANSWER.

G – Slipped upper femoral epiphysis:


This condition occurs during the pubertal growth spurt and the description of the patient is typical; an
overweight boy, aged 13–15 who is sexually underdeveloped. The condition also occurs less commonly
in unusually tall and thin children. The onset of pain may be either acute, chronic, or acute on chronic.
Examination findings are as described. Treatment is by internal fixation of the slipped epiphysis with or
without a corrective osteotomy.

Scenario 2
A 7-year-old boy is seen in clinic with a history of a limp and pain in the left groin. Initially the pain was severe and
he was unable to move his left leg. The pain resolved spontaneously after bed rest. One month later his
symptoms have now recurred with a similar, less severe pain, which causes him to limp. Examination of the left
lower limb is normal except for limited abduction and internal rotation of the left hip.
B - Congenital dislocation of the hip« YOUR ANSWER
E - Perthes‟ disease« CORRECT ANSWER.

E – Perthes’ disease:
The condition tends to occur between the ages of 4 and 7 when the blood supply is dependent on the
lateral epiphyseal vessels, which run in the retinaculum. Trauma to the joint with a secondary effusion
may compromise these vessels with consequent necrosis of the femoral head. Initial presentation is with
symptoms and signs of an irritable hip; pain in the affected joint and a reduced range of movement. With
time the pain subsides and movement returns; however, abduction and internal rotation remain limited.
Changes may only become apparent on X-ray after several years and as such these patients need to be
followed up carefully. Treatment is directed at ‘containment’, with the aim of ensuring the femoral head
remains in the acetabulum by preventing lateral displacement of the femoral head. This is usually
achieved by the use of an abduction splint.

Scenario 3
A 7-year-old girl is seen in the paediatric Emergency Department with a 1-day history of sudden onset of pain in
the left hip and difficulty in ambulation. On examination she is apyrexial, appears well and is lying with her hip
flexed and externally rotated. Palpation of her hip reveals tenderness over the anterior aspect. Extension,
abduction and internal rotation lead to severe pain. Initial investigations include a full blood count, erythrocyte
sedimentation rate and plain radiographs, all of which are normal.
C - Congenital subluxation« YOUR ANSWER
D - Non-specific transient synovitis« CORRECT ANSWER.

D – Non-specific transient synovitis:


This is the commonest cause of a limp with hip pain in a child and is self limiting in its natural history.
Onset of pain tends to be sudden and of variable intensity. Examination findings are as described, and
---------------------------------------------------------------------------------------------------------------------------------------------------------------------------------------------------------------
Dr Mohammed Shamsul Islam Khan, Medical Officer, Clinical Neuro-Surgery, National Institute of Neuro-Sciences and Hospital
Sher-E-Bangla Nagar, Dhaka-1207, Bangladesh. Mobile: +880 1713 455 662, +880 1685 811979. E-mail: drsikhan@gmail.com
MyPasTest: MRCS A Online - Jan Exam 2015
22. Surgical Specialities; Trauma & Orthopaedic Surgery (76Qs)
----------------------------------------------------------------------------------------------------------------------------------
initial investigations tend to be normal. The condition tends to resolve after a few days of bed rest with
analgesia and gentle traction applied to the affected limb.

Scenario 4
A 5-year-old girl is seen with a history of a limp. Her father says that she has been unwell recently with an
intermittent fever. On examination she has an obvious limp when walking and is pyrexial (temperature 37.9°C).
Examination reveals mild tenderness over the right hip, with a generalised reduction in movement. Her erythrocyte Page |
sedimentation rate is raised and her serum is positive for antinuclear antibodies. 1349
D - Non-specific transient synovitis« YOUR ANSWER
H - Still‟s disease« CORRECT ANSWER.

H – Still’s disease:
The history is characteristic with an episodic fever associated with arthritis. Despite the presentation with
a fever there are no other features to suggest a septic arthritis; pyogenic arthritis tends to occur at a
younger age (usually less than 2), the child appears unwell and the affected joint is hot, swollen and held
absolutely still. Rheumatoid factor is negative and antinuclear antibodies may be present. The erythrocyte
sedimentation rate tends to be raised. Three types have been described, systemic, pauciarticular and
polyarticular. Initial treatment is with non-steroidal anti-inflammatory drugs.

44. Theme: Back pain


A Ankylosing spondylitis
B Intervertebral disc herniation
C Metastatic carcinoma
D Multiple myeloma
E Osteoporosis
F Osteomyelitis
G Scheuermann’s disease
H Spinal stenosis
I Spinal trauma
J Spondylolisthesis
K Spondylosis
L Tuberculosis.

From the list above, select the most likely diagnosis for the following patients who all present with back
pain. The items may be used once, more than once, or not at all.

Scenario 1
A 73-year-old diabetic man complains of severe back pain following an anterior resection. On examination, his
temperature is 38.1°C, there is a limited range of spinal movements and marked lumbar muscle spasm.
Neurological examination is normal.
B - Intervertebral disc herniation « YOUR ANSWER
F - Osteomyelitis« CORRECT ANSWER.

F – Osteomyelitis:
In approximately half of cases of osteomyelitis of the spine there is a history of recent pelvic surgery,
urinary tract infection, cutaneous sepsis, or diabetes mellitus. It is likely that haematogenous spread
occurs via the vertebral venous plexus (Batson’s plexus) to the spine. The most common infecting
organism is Staphylococcus aureus, although Gram-negative organisms occur in association with urinary
---------------------------------------------------------------------------------------------------------------------------------------------------------------------------------------------------------------
Dr Mohammed Shamsul Islam Khan, Medical Officer, Clinical Neuro-Surgery, National Institute of Neuro-Sciences and Hospital
Sher-E-Bangla Nagar, Dhaka-1207, Bangladesh. Mobile: +880 1713 455 662, +880 1685 811979. E-mail: drsikhan@gmail.com
MyPasTest: MRCS A Online - Jan Exam 2015
22. Surgical Specialities; Trauma & Orthopaedic Surgery (76Qs)
----------------------------------------------------------------------------------------------------------------------------------
tract infection. The pain is usually worse in the recumbent position, and the patient is frequently pyrexial.
White cell count, erythrocyte sedimentation rate and C-reactive protein are raised. Blood cultures,
computed tomography, radioisotope or magnetic resonance scanning may all be helpful in making the
diagnosis. Treatment is with rest, analgesia and appropriate antibiotics. Occasionally, surgery is required
to ‘decompress’ the spine.

Scenario 2 Page |
A 57-year-old woman with a past history of breast cancer presents to The Emergency Department with 1350
progressive severe thoracic back pain that has not responded to simple analgesics. The pain is now constant, and
interrupts her sleep.
A - Ankylosing spondylitis « YOUR ANSWER
C - Metastatic carcinoma « CORRECT ANSWER.

C – Metastatic carcinoma:
This case describes the disseminated presentation of carcinoma of the bronchus with secondary
involvement of the lumbar spine. The majority of extradural spinal tumours are metastatic. The most
common primary sites include the breast, bronchus, prostate, kidney, thyroid gland, or haemopoietic
malignancies. The pain is non-mechanical (unrelated to physical activity), and is aggravated by
recumbency. Occasionally, vertebral collapse may lead to presentation with profound neurological deficit.
Plain radiographs may be normal early in the disease, and so isotope and magnetic resonance imaging
may be more useful. Metastatic disease without clinical or radiological evidence of neurological
compression is treated with radiotherapy, chemotherapy, or hormone manipulation. Occasionally,
anterior or posterior decompression surgery +/- fixation is required.

Scenario 3
A 65-year-old woman is referred for assessment of chronic lower back pain. Past history includes rheumatoid
arthritis, for which she uses long-term steroids. She underwent a total abdominal hysterectomy and bilateral
salpingo-oophorectomy at the age of 35 years. There is no neurological deficit on examination.
C - Metastatic carcinoma « YOUR ANSWER
E - Osteoporosis « CORRECT ANSWER.

E – Osteoporosis:
This condition is defined by the World Health Organisation as ‘a progressive systemic skeletal disease
characterised by low bone mass and microarchitectural deterioration of bone tissue with increased bone
fragility and susceptibility to fracture’. Osteoporosis results in 200, 000 fractures per year (one-third of all
women sustain one or more osteoporotic fractures), and leads to significant pain and individual disability.
Bone density decreases with increasing age, immobility and postmenopausally in women. Conditions
leading to secondary osteoporosis include: thyroid dysfunction, Cushing’s syndrome, premature (< 51
years)/iatrogenic (total abdominal hysterectomy with bilateral salpingo-oophorectomy) menopause,
rheumatoid arthritis, drugs (corticosteroids, alcohol, anticonvulsants). Measurement of bone mineral
density can be performed using dual energy X-ray absorptiometry scanning, and the hip is the site with
the highest predictive value. Hormone replacement therapy, and bisphosphonates
(etidronate/alendronate) prevent bone loss and decrease the risk of fractures; vitamin D and calcium
supplements decrease the risk of hip and other fractures in the frail elderly.
45. Theme: Complications of fractures
A Associated injury: nerve
B Associated injury: vascular
C Associated injury: visceral
---------------------------------------------------------------------------------------------------------------------------------------------------------------------------------------------------------------
Dr Mohammed Shamsul Islam Khan, Medical Officer, Clinical Neuro-Surgery, National Institute of Neuro-Sciences and Hospital
Sher-E-Bangla Nagar, Dhaka-1207, Bangladesh. Mobile: +880 1713 455 662, +880 1685 811979. E-mail: drsikhan@gmail.com
MyPasTest: MRCS A Online - Jan Exam 2015
22. Surgical Specialities; Trauma & Orthopaedic Surgery (76Qs)
----------------------------------------------------------------------------------------------------------------------------------
D Avascular necrosis (AVN)
E Compartment syndrome
F Complex regional pain syndrome (Sudeck’s atrophy)
G Crush syndrome
H Deep venous thrombosis
I Delayed union
J Fat embolism Page |
K Hypovolaemic shock 1351
L Malunion
M Myositis ossificans
N Non-union
O Osteomyelitis
P Pulmonary embolus.
The above list documents complications of fractures. Please pick the most appropriate complication for
the following clinical scenarios. Each item may be used once, more than once, or not at all.

Scenario 1
A 35-year-old footballer has been admitted to the ward having sustained a spiral fracture to his right tibia. He is
awaiting surgery and currently has a plaster backslab protecting his leg. The nurses on the ward have called you
multiple times to ask for stronger analgesia, which apparently is still not settling his pain. When you assess him he
is fidgeting in the bed. He complains of significant pain at the site of his fracture and some numbness between his
great and second toe. He has warm, pink toes, and a dorsalis pedis pulse can just be palpated beneath the
plaster and bandaging.
A - Associated injury: nerve « YOUR ANSWER
E - Compartment syndrome « CORRECT ANSWER.

E – Compartment syndrome:
This is the typical description of compartment syndrome. This term refers to increased soft tissue
pressure within an enclosed soft tissue compartment usually of the extremities (although it is possible to
see abdominal compartment syndrome). Untreated it can lead to devastating muscle necrosis,
contracture, nerve damage and ultimately severe functional impairment. Common causes of compartment
syndrome include fractures (commonly tibial), soft tissue crush injuries, burns, gunshot wounds, surgery
and vascular impairment. Clinical presentation, in a conscious patient, is severe pain, particularly on
passive stretching of the muscle group involved. As time progresses paraesthesia, pallor, pulselessness
and paralysis all begin to develop (the latter three at a stage where irreversible damage may already have
occurred). Clinical suspicion with a relevant history should be enough to warrant surgical exploration and
decompression via fasciotomy.

Scenario 2
A 29-year-old woman who dislocated her left elbow 4 weeks ago, during a strenuous martial arts training session,
is being followed up in fracture clinic. (The elbow had been successfully re-located in a closed manoeuvre in
Casualty). She tells you that after an initially good recovery, movement of the joint has suddenly become
restricted and very painful. On closer questioning you determine that she has been rather actively mobilising the
joint from an early stage to attempt to get back to her martial arts. An X-ray shows some calcification anterior to
the joint.
B - Associated injury: vascular « YOUR ANSWER
M - Myositis ossificans« CORRECT ANSWER.

---------------------------------------------------------------------------------------------------------------------------------------------------------------------------------------------------------------
Dr Mohammed Shamsul Islam Khan, Medical Officer, Clinical Neuro-Surgery, National Institute of Neuro-Sciences and Hospital
Sher-E-Bangla Nagar, Dhaka-1207, Bangladesh. Mobile: +880 1713 455 662, +880 1685 811979. E-mail: drsikhan@gmail.com
MyPasTest: MRCS A Online - Jan Exam 2015
22. Surgical Specialities; Trauma & Orthopaedic Surgery (76Qs)
----------------------------------------------------------------------------------------------------------------------------------
M – Myositis ossificans:
This is a process whereby extraskeletal ossification occurs in muscle and soft tissue. Usually it arises
after injury: commonly elbow fractures, elbow dislocations and injuries leading to large haematoma
formation (eg soft tissue injuries of the thigh). It is thought to be related to joint mobilisation that is too
early after injury, or too rigorous. Symptoms are as those described in the clinical scenario and treatment
is rest, usually with immobilisation of the affected joint in a cast for a few weeks. In this way the
accumulated calcification gradually reduces, and movement increases. Late excision may ultimately be Page |
required for non-resolution. 1352
Scenario 3
A 45-year-old woman, who fell from her horse 3 months ago, returns for follow-up in orthopaedic outpatients. At
the time her notes documented a dorsiflexion injury to her left wrist causing a scaphoid fracture that was treated in
a scaphoid cast for 8 weeks. She continues to have pain and weakness in her left wrist despite a course of
physiotherapy. The X-ray that you request shows increased density at the proximal pole of the scaphoid.
C - Associated injury: visceral « YOUR ANSWER
D - Avascular necrosis (AVN) « CORRECT ANSWER.

D – Avascular necrosis (AVN)


Bone death (osteonecrosis) is usually the result of impaired blood supply. This can occur by two
mechanisms: interruption of arterial inflow (eg after a fracture), or obstruction of venous outflow (eg by
lesions that infiltrate and block the venous sinusoids). In this case a fracture to the scaphoid has
disrupted the local blood supply. Other common sites that have a propensity to become ischaemic, and
hence develop AVN, include the femoral head and the body of the talus. On X-ray the distinctive feature of
AVN is increased bone density (as a result of new bone ingrowth in the necrotic segment with disuse
osteoporosis in the surrounding regions).

Scenario 4
A 27-year-old rugby player presents to Casualty after being tackled to the ground during a match. His team-mates
give an account of a forceful shoulder blow to his left chest. On examination he has significant bruising over his
lower left chest and this region feels boggy. Clinically two ribs appear fractured. As you assess him further he
starts to complain of severe shortness of breath and his breath sounds are diminished on the left side. Percussion
note is hyper-resonant. SaO2: 89% on air, respiratory rate 35 breaths/min.
D - Avascular necrosis (AVN) « YOUR ANSWER
C - Associated injury: visceral « CORRECT ANSWER.

C – Associated injury: visceral:-


Fractures around the trunk can often be complicated by injuries to the underlying viscera. In this scenario
blunt trauma to the chest with resultant rib fractures have led to a pneumothorax.

46. Theme: Lower limb pathologies


A Ascending lymphangitis
B Common peroneal nerve palsy
C Compartment syndrome
D Deep-vein thrombosis
E Elephantiasis
F Haemarthrosis of the knee joint
G Milroy’s syndrome
H Ruptured Achilles tendon
---------------------------------------------------------------------------------------------------------------------------------------------------------------------------------------------------------------
Dr Mohammed Shamsul Islam Khan, Medical Officer, Clinical Neuro-Surgery, National Institute of Neuro-Sciences and Hospital
Sher-E-Bangla Nagar, Dhaka-1207, Bangladesh. Mobile: +880 1713 455 662, +880 1685 811979. E-mail: drsikhan@gmail.com
MyPasTest: MRCS A Online - Jan Exam 2015
22. Surgical Specialities; Trauma & Orthopaedic Surgery (76Qs)
----------------------------------------------------------------------------------------------------------------------------------
I Superficial thrombophlebitis
J Torn calf muscle.

For each of the following statements, select the most likely cause of limb pain from the above list. Each
option may be used once, more than once, or not at all.

Scenario 1 Page |
A 37-year-old motorcyclist is brought to the Emergency Department following an RTA. He complains of numbness 1353
in his foot and unremitting pain in his right lower leg which is made worse by passive dorsiflexion of his ankle. X-
ray reveals a closed fracture of the right tibia.
A - Ascending lymphangitis« YOUR ANSWER
C - Compartment syndrome« CORRECT ANSWER.

C – Compartment syndrome:
Compartment syndrome may develop after trauma (accident) or surgery. It results from an increase in
pressure within an osseofacial compartment, leading to compromise of the microcirculation and nerve
and muscle damage. The microcirculation is compromised if compartment pressures rise above 30–40
mmHg (or the difference between the diastolic pressure and the compartment pressure is < 30 mmHg).
With increasing compartment pressures, the capillary perfusion is inadequate to meet the metabolic
demands of the intracompartmental tissues. The patient complains of unremitting pain that is not relieved
by morphine. Loss of sensation in the early stages precedes motor loss – loss of peripheral pulses is
usually a late, but sinister sign. Irreversible injury occurs with pressures as low as 30 mmHg for 6–8
hours.

Scenario 2
A 73-year-old patient presents with mild pyrexia, and pain and swelling in her left calf. She underwent total hip
replacement 8 days ago.
B - Common peroneal nerve palsy« YOUR ANSWER
D - Deep-vein thrombosis« CORRECT ANSWER.

D – Deep-vein thrombosis:
Hospitalisation and immobility increase the incidence of deep-vein thrombosis. The other risk factors
include: increasing age, pregnancy, oral contraceptive pill, surgery especially orthopaedic or pelvic,
malignancy, past history of DVT and thrombophilia. In addition, any factor contributing to venous stasis,
vein wall damage and increased coagulation of blood (known as Virchow’s triad) predisposes to
formation of a clot. Clinical features of DVT include redness, swelling, pain, calf tenderness, dilated
superficial veins (sometimes), and low-grade pyrexia. These signs and symptoms usually develop after 7–
10 days post-operatively.

Scenario 3
A 13-year-old adolescent is brought to her GP with a swollen right leg; the swelling is extending up to the knee. It
is painless and the parents say that the swelling was first noticed shortly after menarche. She is otherwise
systemically well.
C - Compartment syndrome« YOUR ANSWER
G - Milroy‟s syndrome« CORRECT ANSWER.

G – Milroy’s syndrome:

---------------------------------------------------------------------------------------------------------------------------------------------------------------------------------------------------------------
Dr Mohammed Shamsul Islam Khan, Medical Officer, Clinical Neuro-Surgery, National Institute of Neuro-Sciences and Hospital
Sher-E-Bangla Nagar, Dhaka-1207, Bangladesh. Mobile: +880 1713 455 662, +880 1685 811979. E-mail: drsikhan@gmail.com
MyPasTest: MRCS A Online - Jan Exam 2015
22. Surgical Specialities; Trauma & Orthopaedic Surgery (76Qs)
----------------------------------------------------------------------------------------------------------------------------------
The peak age of onset of Milroy’s disease (primary lymphoedema; lymphoedema praecox) is 10–30 years,
and it has a clear genetic predisposition in some patients. This condition is the result of congenital
absence of the lymphatics. It is more common in females and is seen shortly after menarche. It is more
likely to be unilateral. The swelling usually develops around the ankle and the dorsum of the foot which
soon spreads proximally extending up to the knee. Occasionally, patients develop numerous vesicles in
the skin which may leak clear lymph or, occasionally chyle. The swelling is worse during the day and
decreases at night. Page |
1354
Scenario 4
A 42-year-old teacher is persuaded to take part in a staff–student tennis match. She has not done any exercise in
the last 10 years. While running for a ball, she experiences sudden, severe pain in her right calf and is unable to
play any longer. On examination, there is swelling and tenderness just above the heel, and she is unable to walk
tiptoe.
D - Deep-vein thrombosis« YOUR ANSWER
H - Ruptured Achilles tendon« CORRECT ANSWER.

H– Ruptured Achilles tendon:


Rupture of the Achilles tendon is common in patients over the age of 40, probably because the tendon is
frayed. It usually occurs when the patient is involved in unaccustomed activities, such as running or
jumping; sudden contraction of the calf muscle is resisted by the body weight and the tendon ruptures.
The patient feels as if he or she has been struck just above the heel and is usually unable to continue with
the involved activity. Soon after the tear, a gap can be felt about 5 cm above the insertion of the tendon.
The patient is unable to tiptoe and plantar flexion of the ankle (or foot) is weak or lost. Simmond’s test is
positive (reduced plantar flexion of the ankle on squeezing the calf muscle). The management is either
conservative (using plaster casts) or surgical (with the foot plantar flexed, the cut ends are approximated
and held in place using non-absorbable sutures 2/0 nylon). Following this, the ankle and the foot are
immobilised in a below-knee plaster cast in equinus position for 6–8 weeks.

47. Theme: Femoral neck fracture management


A Unipolar Austin Moore hemiarthroplasty
B Bipolar Hastings hemiarthroplasty
C Cannulated hip screws
D Traction
E Intramedullary nail
F Total hip replacement
G Dynamic hip screw.

Select the most appropriate treatment for each of the patients below. Each option may be used once,
more than once, or not at all.

Scenario 1
A 66-year-old actively mobile lady fell whilst crossing the road sustaining an intracapsular right femoral neck
fracture which appears to be completely displaced and detached from the neck (Garden IV).
A - Unipolar Austin Moore hemiarthroplasty« YOUR ANSWER
F - Total hip replacement« CORRECT ANSWER.

Garden IV intracapsular neck of femur fractures will inevitably need removal of the head and arthroplasty.
Scottish Intercollegiate Guidelines Network (SIGN) guidelines suggest that patients with displaced
---------------------------------------------------------------------------------------------------------------------------------------------------------------------------------------------------------------
Dr Mohammed Shamsul Islam Khan, Medical Officer, Clinical Neuro-Surgery, National Institute of Neuro-Sciences and Hospital
Sher-E-Bangla Nagar, Dhaka-1207, Bangladesh. Mobile: +880 1713 455 662, +880 1685 811979. E-mail: drsikhan@gmail.com
MyPasTest: MRCS A Online - Jan Exam 2015
22. Surgical Specialities; Trauma & Orthopaedic Surgery (76Qs)
----------------------------------------------------------------------------------------------------------------------------------
intracapsular femoral neck fractures who are relatively fit and mobile with reasonable life expectancy then
total hip replacement is the option of choice.

Scenario 2
A 70-year-old demented lady in a nursing home falls out of bed complaining right hip pain. X-ray of the right hip
shows an intracapsular completely detached and displaced fracture of the femoral neck (Garden IV).
B - Bipolar Hastings hemiarthroplasty« YOUR ANSWER Page |
A - Unipolar Austin Moore hemiarthroplasty« CORRECT ANSWER. 1355
Here mobility is not an issue, so a hemiarthroplasty that will not erode into the acetabulum is justified. No
benefit of bipolar has been shown over unipolar (as movement has been shown to occur at the outer
articulation); although some surgeons prefer one to the other, there does not appear to be any good
evidence to show any significant advantage from using bipolar hemiarthroplasty in favour of unipolar
hemiarthroplasty (SIGN guidelines: Prevention and management of hip fracture on older people; section
7: surgical management).

Scenario 3
A 61-year-old lady fell on her right hip while watering the plants in the garden, sustaining pain and inability to
weight bear. X-ray of the right hip shows a two-part intertrochanteric fracture of reverse obliquity.
C - Cannulated hip screws« YOUR ANSWER
E - Intramedullary nail« CORRECT ANSWER
Intertrochanteric fractures with reverse obliquity will need intramedullary nailing as a dynamic hip screw
(DHS) will fail because most of the weight will be transmitted through the DHS pin.
Scenario 4
A 77-year-old lady trips while walking and falls on her right hip sustaining a two-part intertrochanteric fracture of
usual obliquity.
D - Traction« YOUR ANSWER
G - Dynamic hip screw« CORRECT ANSWER.
Here a dynamic hip screw to hold the two parts together will be justified (cf previous case).
48. Theme: Knee injury
A Anterior cruciate ligament (ACL) rupture
B Posterior cruciate ligament (PCL) rupture
C Medial meniscal injury
D Lateral meniscal injury
E Medial collateral ligament (MCL) injury
F Lateral collateral ligament (LCL) injury
G Patellar ligament tendonitis
H Patellar tendon rupture.

Select the most appropriate diagnosis for each of the patients below. Each option may be used once,
more than once, or not at all.
Scenario 1
A 29-year-old football player complains of anterior knee pain especially on extending the knee, relieved by rest.
On examination there is tenderness below the patella and on the tibial tuberosity; however there are no other
signs of knee injury.
A - Anterior cruciate ligament (ACL) rupture« YOUR ANSWER
---------------------------------------------------------------------------------------------------------------------------------------------------------------------------------------------------------------
Dr Mohammed Shamsul Islam Khan, Medical Officer, Clinical Neuro-Surgery, National Institute of Neuro-Sciences and Hospital
Sher-E-Bangla Nagar, Dhaka-1207, Bangladesh. Mobile: +880 1713 455 662, +880 1685 811979. E-mail: drsikhan@gmail.com
MyPasTest: MRCS A Online - Jan Exam 2015
22. Surgical Specialities; Trauma & Orthopaedic Surgery (76Qs)
----------------------------------------------------------------------------------------------------------------------------------
G - Patellar ligament tendonitis« CORRECT ANSWER.
Patellar ligament tendonitis (so called jumper’s knee) is common in athletes due to repetitive microtrauma
to the ligament. It is treated by non-steroidal anti-inflammatory drugs (NSAID) and rest, or steroid
injection. The extensor mechanism is painful but NOT lost.
Scenario 2
A 28-year-old footballer presented to outpatients with a history of recurrent right knee pain and inability to extend Page |
the knee (he describes as if the knee locks). He sustained an injury to that knee 2 months ago with no mention of
a significant swelling. On examination there is click and tenderness in the medial joint line when you extend the 1356
knee.
B - Posterior cruciate ligament (PCL) rupture« YOUR ANSWER
C - Medial meniscal injury« CORRECT ANSWER.
Due to relative avascularity of the menisci there is seldom a mention of significant swelling that
classically accompanies anterior cruciate ligament (ACL) rupture (haemarthrosis); locking (inability to
extend) of the knee and medial joint line tenderness all point to medial meniscal injury. Arthroscopy is
indicated here.
Scenario 3
A 45-year-old man presented with his knee giving way when he climbs upstairs. He had sustained an injury to that
knee few months ago when he was skiing in France where he describes a hyperextension injury of the knee which
was followed rapidly by a tense knee effusion. On examination you can displace the tibia forward on the femur.
C - Medial meniscal injury« YOUR ANSWER
A - Anterior cruciate ligament (ACL) rupture« CORRECT ANSWER.

Hyperextension injury and rapid swelling (haemarthrosis) point to cruciate injury. The fact that you can
displace the tibia forward (Lachman test) favours ACL rupture.

Scenario 4
A 25-year-old rugby player presented with recurrent pain and giving way of his left knee. His knee was hit 2
months ago from the side in a tackle during a game sustaining swelling and pain. On examination you find that the
joint opens on valgus stress.
D - Lateral meniscal injury« YOUR ANSWER
E - Medial collateral ligament (MCL) injury« CORRECT ANSWER.

This is a typical mechanism of sport injury to the medial collateral ligament MCL (lateral blow to the knee).
Valgus stress confirms rupture of the ligament.

Scenario 5
46-year-old male presented with inability to walk after he tripped on climbing the stairs. He suffers from brittle
asthma for which he is on steroids. On examination of the knee there is mild swelling but no effusion. He also
can‟t extend his knee, and a gap is felt below the patella compared to the other side.
E - Medial collateral ligament (MCL) injury« YOUR ANSWER
H - Patellar tendon rupture« CORRECT ANSWER.

This is common in patients on steroid as it causes breakdown of collagen strength and organisation,
thereby weakening tendons, ligaments, bones, skin etc.

49. Theme: Nerve damage


A Horner syndrome
---------------------------------------------------------------------------------------------------------------------------------------------------------------------------------------------------------------
Dr Mohammed Shamsul Islam Khan, Medical Officer, Clinical Neuro-Surgery, National Institute of Neuro-Sciences and Hospital
Sher-E-Bangla Nagar, Dhaka-1207, Bangladesh. Mobile: +880 1713 455 662, +880 1685 811979. E-mail: drsikhan@gmail.com
MyPasTest: MRCS A Online - Jan Exam 2015
22. Surgical Specialities; Trauma & Orthopaedic Surgery (76Qs)
----------------------------------------------------------------------------------------------------------------------------------
B Neurapraxia of the common peroneal nerve
C Neurapraxia of the median nerve
D Neurapraxia of the radial nerve
E Neurotmesis of the common peroneal nerve
F Neurotmesis of the median nerve
G Posterior interosseus nerve lesion
H Sciatic nerve injury. Page |
1357
For each of the patients listed below, select the site and type of nerve damage that best explains the
clinical situation from the above list. Each option may be used once, more than once, or not at all.

Scenario 1
A 21-year-old man sustained a comminuted fracture of the right femur and a fracture of the ipsilateral tibia and
fibula. He was treated with skeletal traction and a below-knee plaster overnight. On review it was noticed that he
could not dorsiflex his right toes.
A - Horner syndrome« YOUR ANSWER
B - Neurapraxia of the common peroneal nerve« CORRECT ANSWER
The peroneal nerve is very susceptible to pressure, and thus may result in a neurapraxia.

Scenario 2
A child falls on an outstretched hand and sustains a severely displaced supracondylar fracture of the humerus.
B - Neurapraxia of the common peroneal nerve« YOUR ANSWER
C - Neurapraxia of the median nerve« CORRECT ANSWER
Following a supracondylar fracture of a child‟s humerus, a neurapraxia of the median nerve (specifically, the
anterior interosseous branch of the median nerve) is the most common neurological lesion but damage to the
ulnar nerve is also not uncommon.

Scenario 3
Following a difficult elective plating of a non-union fracture of the humeral shaft, the patient was unable to extend
his fingers and wrist. No nerves were visualised during the procedure.
C - Neurapraxia of the median nerve« YOUR ANSWER
D - Neurapraxia of the radial nerve« CORRECT ANSWER.

If a surgical procedure is complicated by nerve injury, a transection (neurotmesis) must be considered –


especially if the nerve has not been visualized.

Scenario 4
A motorcyclist came off his bike at considerable speed. Both he and his bike were then dragged down the road by
a car. On examination he had a flail left upper limb. Secondary survey also reveals ocular ptosis and pupillary
miosis on the ipsilateral side.
D - Neurapraxia of the radial nerve« YOUR ANSWER
A - Horner syndrome« CORRECT ANSWER.

A flail upper limb suggests a brachial plexus lesion, which might well be associated with Horner
syndrome if the sympathetic chain is involved.

Scenario 5
Following a total hip replacement performed via a posterior approach, the patient was noted to have a foot drop.
---------------------------------------------------------------------------------------------------------------------------------------------------------------------------------------------------------------
Dr Mohammed Shamsul Islam Khan, Medical Officer, Clinical Neuro-Surgery, National Institute of Neuro-Sciences and Hospital
Sher-E-Bangla Nagar, Dhaka-1207, Bangladesh. Mobile: +880 1713 455 662, +880 1685 811979. E-mail: drsikhan@gmail.com
MyPasTest: MRCS A Online - Jan Exam 2015
22. Surgical Specialities; Trauma & Orthopaedic Surgery (76Qs)
----------------------------------------------------------------------------------------------------------------------------------
E - Neurotmesis of the common peroneal nerve« YOUR ANSWER
H - Sciatic nerve injury« CORRECT ANSWER.

During a hip replacement, two forms of nerve injury are well documented: direct damage to the sciatic
nerve at the level of the hip joint (more common); and pressure on the peroneal nerve at the neck of the
fibula. During a posterior approach to the hip, the sciatic nerve is in particular danger.
Page |
50. Theme: Knee injuries 1358
A Anterior cruciate ligament (ACL) injury
B Extensor mechanism disruption
C Knee dislocation
D Lateral collateral ligament injury
E Medial collateral ligament injury
F Meniscal tear
G Neck of fibular fracture
H Patellar dislocation
I Patellar fracture
J Posterior cruciate ligament (PCL) injury
K Proximal tibio-fibular dislocation
L Supracondylar fracture
M Tibial plateau fracture.
The above are all descriptions of injuries to the knee. For the following scenarios please select the most
appropriate answer from the list. The items may be used once, more than once, or not at all.

Scenario 1
A 30-year-old footballer, playing in the Sunday league, presents to The Emergency Department minors on
Monday evening. He is complaining of a swollen right knee with pain, and an inability to completely straighten the
joint. He describes the injury occurring as he tried to break into a run on the muddy pitch. His right thigh twisted
but his leg remained fixed in the mud. At the time, although his knee was painful, he had not noticed any swelling
and had continued to play. This morning he had observed that his right knee was much larger in size than the
other. He has tenderness along the lateral joint line.
A - Anterior cruciate ligament (ACL) injury« YOUR ANSWER
F - Meniscal tear« CORRECT ANSWER.

F – Meniscal tear:
Meniscal tears in the young adult typically result from a rotational stress upon the flexed weight-bearing
knee joint. This mechanism of injury is often the same for anterior cruciate ligament (ACL) rupture (and
frequently these injuries co-exist). The clues in this clinical scenario that point to meniscal damage, over
ACL rupture, include: the symptom of locking (true locking is the inability to fully straighten the knee),
which indicates a mechanical blockage within the knee; and the delayed onset of swelling
(characteristically > 6 h after injury). The swelling is an effusion as a result of the synovial reaction, in
contrast to the haemarthrosis seen in ACL rupture, which is an immediate phenomenon. Symptomatic
meniscal tears warrant arthroscopic intervention.

Scenario 2
A 60-year-old grandfather presents acutely to the Casualty department complaining of severe pain in his left knee.
He tells you that he was playing with his grandson who had unexpectedly jumped up on him from behind, causing
him to stumble and lunge forward. As he lurched forward he noticed a sudden searing pain in his thigh and knee
---------------------------------------------------------------------------------------------------------------------------------------------------------------------------------------------------------------
Dr Mohammed Shamsul Islam Khan, Medical Officer, Clinical Neuro-Surgery, National Institute of Neuro-Sciences and Hospital
Sher-E-Bangla Nagar, Dhaka-1207, Bangladesh. Mobile: +880 1713 455 662, +880 1685 811979. E-mail: drsikhan@gmail.com
MyPasTest: MRCS A Online - Jan Exam 2015
22. Surgical Specialities; Trauma & Orthopaedic Surgery (76Qs)
----------------------------------------------------------------------------------------------------------------------------------
and was immediately unable to walk on the affected leg. On examination he is holding his knee in slight flexion
and is unable to fully straighten the joint. He is incapable of performing a straight leg raise. An X-ray does not
reveal any fracture.
B - Extensor mechanism disruption« CORRECT ANSWER.

B – Extensor mechanism disruption:


Any part of the extensor mechanism can be damaged at any age; however, there are common patterns of Page |
injury. In the middle-aged and elderly the quadriceps tendon and muscle tend to be compromised more 1359
readily. Patellar fractures are seen more often in the young and middle-aged, with tibial tuberosity
avulsion occurring in younger age ranges. Patellar tendon rupture can happen at any age. In this scenario
the man has ruptured his quadriceps tendon. This results from forced contraction of the extensor
mechanism with the foot planted on the floor and can arise from a simple stumble or fall. Clinical
examination may reveal a haemarthrosis and there is inevitably a palpable gap in the tendon at the site of
rupture. The patient is unable to perform a straight leg raise (particularly against resistance) or to actively
straighten the knee. Direct repair of the tendon using nonabsorbable material is required, because
without intervention the extensor mechanism is severely compromised and the patient is unable to walk
properly.

Scenario 3
A 35-year-old skier has been flown back to the UK from the Alps after injuring himself on the slopes. He describes
a „twisting‟ injury to his left knee as his ski caught in some soft snow. Immediately he felt an agonising pain, heard
a „pop‟ and noticed instantaneous swelling, under his salopettes. He was unable to carry on skiing and after
hobbling down the piste was seen by the resort doctor who has documented a positive Lachman test.
C - Knee dislocation« YOUR ANSWER
A - Anterior cruciate ligament (ACL) injury« CORRECT ANSWER.

A – Anterior cruciate ligament (ACL) injury:


Injury to the ACL is fairly common. Patients often describe a non-contact, deceleration, twisting injury
(although sometimes the mechanism is of hyperextension). In over 50% of cases the history will include
hearing a ‘pop’ or feeling a sensation of something tearing inside the knee, with earlyonset swelling. (NB
Up to 80% of patients attending Casualty with an acute haemarthrosis, have sustained an injury to their
ACL and of these, 60% will have another associated pathology within the knee.) Specific examination of
the ACL via the anterior draw and Lachman tests should be undertaken. Routine use of magnetic
resonance imaging is probably unnecessary; however, it does have the advantage of delineating other
associated ligamentous, meniscal, or bony damage. With regard to long-term management of ACL
injuries there is still debate as to the necessity and timing of reconstructive surgery.

Scenario 4
A 28-year-old woman is brought into Casualty by ambulance following an accident while parachuting with the
Territorial Army. She landed badly and forcefully on her left knee, which was flexed at the time. She complains of
pain and tingling on the dorsum of her foot, and she cannot dorsiflex at the left ankle. X-ray of the knee does not
clearly show any fracture.
D - Lateral collateral ligament injury« YOUR ANSWER
K - Proximal tibio-fibular dislocation« CORRECT ANSWER.

K – Proximal tibio-fibular dislocation:


This is an uncommon injury and tends to be caused by twisting of the weight-bearing flexed knee. It can
occur as an isolated injury or in association with major trauma. Classically known as ‘horseback rider’s
---------------------------------------------------------------------------------------------------------------------------------------------------------------------------------------------------------------
Dr Mohammed Shamsul Islam Khan, Medical Officer, Clinical Neuro-Surgery, National Institute of Neuro-Sciences and Hospital
Sher-E-Bangla Nagar, Dhaka-1207, Bangladesh. Mobile: +880 1713 455 662, +880 1685 811979. E-mail: drsikhan@gmail.com
MyPasTest: MRCS A Online - Jan Exam 2015
22. Surgical Specialities; Trauma & Orthopaedic Surgery (76Qs)
----------------------------------------------------------------------------------------------------------------------------------
knee’, it is more commonly associated with parachute jumping (requiring a considerable amount of force).
Hypermobile individuals (eg those with Ehlers–Danlos syndrome) are more susceptible. Examination
reveals tenderness over the proximal tibio-fibular joint and movement of the ankle tends to cause pain in
the knee. It is paramount to assess the integrity of the common peroneal nerve, as it passes close to the
joint and can easily be injured. Reduction is effected by pressure over the fibular head with the knee
flexed.
Page |
51. Theme: Distal radial fracture management 1360
A Closed reduction and plaster of paris (POP) cast
B POP cast without manipulation
C Intramedullary nailing
D Plate and screws
E K-wires
F External fixation.

Select the most appropriate method of management for each of the patients below. Each option may be
used once, more than once, or not at all.

Scenario 1
A 76-year-old lady fell on her right outstretched arm sustaining a non-comminuted Colles‟ fracture with 20 degrees
of dorsal tilt.
A - Closed reduction and plaster of paris (POP) cast« CORRECT ANSWER.

Any dorsal tilt in Colles’ fracture will need closed reduction if not comminuted to restore function.

Scenario 2
A 25-year-old male fell on the right outstretched hand while playing football sustaining a non-comminuted distal
radial fracture with volar tilt.
B - POP cast without manipulation« YOUR ANSWER
D - Plate and screws« CORRECT ANSWER.

Smith fracture needs ORIF(open reduction and internal fixation) using a buttress plate because the distal
fragment is unstable.

Scenario 3
A 55-year-old lady gets involved in a road traffic accident sustaining an open comminuted distal radial fracture
with dorsal displacement.
C - Intramedullary nailing« YOUR ANSWER
F - External fixation« CORRECT ANSWER.

Ex-Fix will ease care of the wound which should not be closed as it may need debridement later;
otherwise internal fixation will run the risk of implant infection and failure.

Scenario 4
A 10-year-old girl falls on the outstretched arm sustaining a buckled fracture of the distal radius. No angulation is
evident on X-ray.
D - Plate and screws« YOUR ANSWER
B - POP cast without manipulation« CORRECT ANSWER.
---------------------------------------------------------------------------------------------------------------------------------------------------------------------------------------------------------------
Dr Mohammed Shamsul Islam Khan, Medical Officer, Clinical Neuro-Surgery, National Institute of Neuro-Sciences and Hospital
Sher-E-Bangla Nagar, Dhaka-1207, Bangladesh. Mobile: +880 1713 455 662, +880 1685 811979. E-mail: drsikhan@gmail.com
MyPasTest: MRCS A Online - Jan Exam 2015
22. Surgical Specialities; Trauma & Orthopaedic Surgery (76Qs)
----------------------------------------------------------------------------------------------------------------------------------
Buckled and greenstick fractures are not manipulated if there is no angulation or tilt. Some surgeons
don’t even manipulate it if the age is very young and there is no CLINICAL deformity despite a few
degrees of tilt or angulation on the X-rays.

52. Theme: Lower limb nerve injury


A Common peroneal nerve Page |
B Femoral nerve 1361
C Lateral cutaneous nerve of thigh
D Lateral plantar nerve
E Medial plantar nerve
F Pudendal nerve
G Saphenous nerve
H Sciatic nerve
I Sural nerve
J Tibial nerve.

For each of the following statements, select the most likely cause of nerve injury from the above list. Each
option may be used once, more than once, or not at all.

Scenario 1
A 28-year-old patient (37 weeks pregnant) presents to her GP with pain and paraesthesia over the upper outer
aspect of her left thigh. She is able to walk and there is no restriction of movements in her hips or knees.
A - Common peroneal nerve« YOUR ANSWER
C - Lateral cutaneous nerve of thigh« CORRECT ANSWER.

C – Lateral cutaneous nerve of thigh:


Lateral cutaneous nerve of the thigh compression (meralgia paraesthetica) may cause pain and
paraesthesia over the upper, lateral aspect of the thigh. Sensation may also be decreased over this area.
It may be seen in pregnancy or any condition which causes a pressure on this nerve within the pelvis (eg
tumours). The symptoms are usually self-limiting.

Scenario 2
A 32-year-old motorcyclist is brought to the Emergency Department after being involved in an RTA. He is unable
to dorsiflex and evert his left foot. He has also reduced sensation over the lateral aspect of his lower leg and the
dorsum of this foot. X-ray shows a fracture of the fibular neck.
B - Femoral nerve« YOUR ANSWER
A - Common peroneal nerve« CORRECT ANSWER.

A – Common peroneal nerve:


Common peroneal nerve (lateral popliteal nerve; L4–S2) injury is common following fibular neck fractures
since the nerve winds down the neck and is relatively superficial at this point. This nerve gives motor
supply to the dorsiflexor and eversion muscles of the ankle and toes. Its sensory branches supply the
anterior and lateral aspect of the leg and whole of the dorsum of the foot and toes, except the skin
between the great and the second toe (supplied by deep peroneal nerve). Injuries result in foot drop and
the patient is unable to dorsiflex or evert the foot. Sensory loss is over the anterior and lateral aspect of
the leg, and dorsum of the foot and toes.

---------------------------------------------------------------------------------------------------------------------------------------------------------------------------------------------------------------
Dr Mohammed Shamsul Islam Khan, Medical Officer, Clinical Neuro-Surgery, National Institute of Neuro-Sciences and Hospital
Sher-E-Bangla Nagar, Dhaka-1207, Bangladesh. Mobile: +880 1713 455 662, +880 1685 811979. E-mail: drsikhan@gmail.com
MyPasTest: MRCS A Online - Jan Exam 2015
22. Surgical Specialities; Trauma & Orthopaedic Surgery (76Qs)
----------------------------------------------------------------------------------------------------------------------------------
Scenario 3
A 27-year-old man is brought to the Emergency Department following a gunshot injury to his right thigh. He has
numbness over the anterior thigh and medial aspect of his leg. He is unable to extent his knee and the knee jerk is
diminished.
C - Lateral cutaneous nerve of thigh« YOUR ANSWER
B - Femoral nerve« CORRECT ANSWER.
Page |
B – Femoral nerve: 1362
The femoral nerve (L2–4) may be injured by a gunshot wound, traction during surgery, injury to the
femoral triangle or by massive haematoma within the thigh, and in patients with diabetes mellitus and
lumbar spondylosis. There is weakness of the quadriceps muscle causing weak knee extension. Patients
find that the knee gives way on walking and have difficulty climbing stairs. There is numbness over the
anterior thigh and medial aspect of the leg. The knee jerk is depressed.
Scenario 4
A 75-year-old patient presents to the orthopaedic outpatient clinic with a right foot drop and decreased sensation
below the knee on the lateral side. He had a right total hip replacement 8 weeks ago.
D - Lateral plantar nerve« YOUR ANSWER
H - Sciatic nerve« CORRECT ANSWER.
H – Sciatic nerve:
Sciatic nerve (L4–S3) injury is common following traumatic dislocations of the hip (posterior dislocation),
total hip replacement and other traction injuries to the nerve. A complete lesion will affect all the muscles
below the knee, leading to loss of knee flexion, foot drop and an inability to walk. Patients drag their feet
behind them and are often unable to stand for prolonged periods. Calf muscle wasting is a long-term
complication. There is loss of sensation below the knee on the lateral side (medial side is supplied by the
saphenous nerve). The knee jerk is normal but the ankle jerk is lost.

53. Theme: Knee injuries


A Medial meniscus
B Lateral meniscus
C Medial collateral ligament
D Lateral collateral ligament
E Patellar fracture
F Anterior cruciate ligament
G Posterior cruciate ligament.

Scenario 1
A young footballer injured his right knee in a tackle, and had swelling of the knee a few days later. He presented
to A&E with tenderness in an area 2–3 cm above the joint line on the medial aspect. There is increased valgus
deformity on examination.
A - Medial meniscus« YOUR ANSWER
C - Medial collateral ligament« CORRECT ANSWER.

Scenario 2
A 60-year-old patient developed a painful left knee after a fall. On examination, swelling on the left knee was seen
in the orthopaedic clinic a few days later. Flexion views of both knees showed that the left tibial plateau lay more
posteriorly than that on the right.
B - Lateral meniscus« YOUR ANSWER
---------------------------------------------------------------------------------------------------------------------------------------------------------------------------------------------------------------
Dr Mohammed Shamsul Islam Khan, Medical Officer, Clinical Neuro-Surgery, National Institute of Neuro-Sciences and Hospital
Sher-E-Bangla Nagar, Dhaka-1207, Bangladesh. Mobile: +880 1713 455 662, +880 1685 811979. E-mail: drsikhan@gmail.com
MyPasTest: MRCS A Online - Jan Exam 2015
22. Surgical Specialities; Trauma & Orthopaedic Surgery (76Qs)
----------------------------------------------------------------------------------------------------------------------------------
G - Posterior cruciate ligament« CORRECT ANSWER.

Scenario 3
A patient involved in a road traffic accident (RTA) hit his knee on the dashboard, producing an immediate
haemarthrosis. The X-ray showed no bone injury, but he later experienced difficulty going downstairs.
C - Medial collateral ligament« YOUR ANSWER
G - Posterior cruciate ligament« CORRECT ANSWER. Page |
1363
The history in the young footballer is most suggestive of a medial collateral ligament injury because there
is a valgus deformity and tenderness above the joint line. Tenderness over the joint line is suggestive of
meniscal injury. The posterior cruciate ligament prevents the femur from sliding forwards off the tibial
plateau. In the weight-bearing flexed knee, it is the only stabilising factor for the femur and its attached
quadriceps. The most frequent injury mechanism in isolated posterior cruciate ligament (PCL) tears is a
direct blow on the anterior tibia with the knee flexed thus driving the tibia posteriorly. Automobile
accidents (in which the knee hits the dashboard) and soccer injuries (in which an athlete receives a blow
to the anterior surface of the tibia during knee flexion) characteristically produce this type of injury. PCL
tears may present as pain while walking down stairs due to anterior displacement of the femur over the
tibia. On examination, the ‘posterior sag’ sign (i.e. as demonstrated in the second vignette) is suggestive
of a PCL tear.

54. Theme: Disorders of bone


A Achondroplasia
B Craniocleidodysostosis
C Diaphyseal aclasis
D Ollier’s disease
E Osteochondrodystrophy
F Osteogenesis imperfecta
G Osteopetrosis
H Perthe’s disease
I Rickets
J Scurvy.

For each of the following situations, select the most appropriate cause for the presentation from the
above list. Each option may be used once, more than once, or not at all.

Scenario 1
A 13-year-old boy is brought to the orthopaedic outpatient clinic with a history of tiredness, recurrent throat and
chest infections and gradual loss of hearing. X-ray reveals a „marble bone‟ appearance.
A - Achondroplasia « YOUR ANSWER
G - Osteopetrosis« CORRECT ANSWER.

G – Osteopetrosis:
Osteopetrosis is an autosomal recessive condition. The patient, usually a young adult, may present with
symptoms of anaemia (tiredness) or thrombocytopenia (easy bruising) and leucopaenia (recurrent throat
and chest infections) because of decreased marrow space. Deafness and optic atrophy can result from
compression of the cranial nerves. Blood investigations may reveal a leukoerythroblastic picture. The
---------------------------------------------------------------------------------------------------------------------------------------------------------------------------------------------------------------
Dr Mohammed Shamsul Islam Khan, Medical Officer, Clinical Neuro-Surgery, National Institute of Neuro-Sciences and Hospital
Sher-E-Bangla Nagar, Dhaka-1207, Bangladesh. Mobile: +880 1713 455 662, +880 1685 811979. E-mail: drsikhan@gmail.com
MyPasTest: MRCS A Online - Jan Exam 2015
22. Surgical Specialities; Trauma & Orthopaedic Surgery (76Qs)
----------------------------------------------------------------------------------------------------------------------------------
bones are very dense and brittle, and X-ray reveals a lack of differentiation between the cortex and the
medulla described as ‘marble bone’.

Scenario 2
A 14-year-old boy, who is small for age, is brought to his GP with loss of hearing in both ears. On examination, he
has a blue sclera, knock-knees, and hypermobile fingers. X-rays show multiple fractures (old) of the long bones
and irregular patches of ossification. Page |
B - Craniocleidodysostosis « YOUR ANSWER 1364
F - Osteogenesis imperfecta« CORRECT ANSWER.

F – Ostegenesis imperfect:
Osteogenesis imperfecta (brittle bone disease) is defective osteoid formation due to the congenital
inability to produce adequate intercellular substances, such as osteoid, collagen and dentine. In addition,
there is a failure of maturation of collagen in all the connective tissues. Some typical clinical features of
this condition include: a broad skull, blue sclera, premature deafness, scoliosis, ligament laxity, coxa vara
and knock knees. X-rays may reveal translucent bones, multiple fractures particularly of the long bones,
wormian bones (irregular patches of ossification), and a trefoil pelvis.

Scenario 3
A 3-year old boy is brought to the GP surgery with a swollen and painful right knee joint. The parents also say that
they recently noticed some bleeding from his gums. He lies still and refuses to move the limb. X-ray shows
generalised rarification of the bones in his legs.
C - Diaphyseal aclasis « YOUR ANSWER
J - Scurvy« CORRECT ANSWER.

J – Scurvy:
Scurvy (vitamin C deficiency) causes a failure of collagen synthesis and osteoid formation. The patient,
usually a child or an infant, may present with swelling and tenderness near the large joints. There may be
bleeding from the gums as they are spongy. Spontaneous bleeding may lead to subperiosteal haematoma
and the child remains still (pseudoparalysis) as a result of subperiosteal bleeding. X-rays shows
generalised bone rarefaction. The metaphysis may be deformed or fractured. Vitamin C in the form of
ascorbic acid should be given in doses of 1 gm/day and the child should be encouraged to eat fresh fruit
and vegetables.

Scenario 4
A 2-year-old infant is brought to the Emergency Department with convulsions. On examination, the child lies
listless and flaccid, appears small for age and there is noticeable thickening of both wrists. X-ray shows an
increase in the depth and width of the epiphysis of the lower ends of the radius and ulna.
D - Ollier‟s disease « YOUR ANSWER
I - Rickets« CORRECT ANSWER.

I – Rickets:
Rickets is the childhood form of osteomalacia. Because of vitamin D deficiency osteoid fails to ossify.
Symptoms start from about the age of 1 year. The child may present with tetany or convulsions. The child
is small for age and there is a history of failure to thrive. Bony deformities include: bowing of the femur
and tibia, deformity of the skull (craniotabes), deformity of the chest wall with thickening of the
costochondral junction (ricketty rosary), and a transverse sulcus in the chest caused by the pull of the
diaphragm (Harrison’s sulcus). The characteristic X-ray change is an increase in the depth and width of
---------------------------------------------------------------------------------------------------------------------------------------------------------------------------------------------------------------
Dr Mohammed Shamsul Islam Khan, Medical Officer, Clinical Neuro-Surgery, National Institute of Neuro-Sciences and Hospital
Sher-E-Bangla Nagar, Dhaka-1207, Bangladesh. Mobile: +880 1713 455 662, +880 1685 811979. E-mail: drsikhan@gmail.com
MyPasTest: MRCS A Online - Jan Exam 2015
22. Surgical Specialities; Trauma & Orthopaedic Surgery (76Qs)
----------------------------------------------------------------------------------------------------------------------------------
the epiphysis and the adjacent metaphysis has a ‘cupped’ appearance; these changes are most
noticeable in the wrist.

55. Theme: Bone and soft tissue tumours


A Bone metastases
B Chondroma
C Chondrosarcoma Page |
D Ewing ’s sarcoma 1365
E Fibroma
F Fibrosarcoma
G Leiomyoma
H Leiomyosarcoma
I Lipoma
J Liposarcoma
K Osteoma
L Osteoblastoma
M Osteochondroma
N Osteosarcoma (osteogenic sarcoma)
O Osteoid osteoma
P Simple bone cyst.

The following patients all present with bone or soft tissue tumours. From the list above, select the most
likely diagnosis. The items may be used once, more than once, or not at all.
Scenario 1
A 15-year-old boy attends The Emergency Department. He describes pain affecting his left femur that was initially
an ache but that has now become severe and constant. In addition, he reports generalised malaise and a
persistent cough. On examination, he has an antalgic gait and there is asymmetrical swelling affecting the distal
left femur. Radiology reveals breach of the periosteum, which is elevated, and a „sunray‟ appearance affecting the
distal femur.
A - Bone metastases « YOUR ANSWER
N - Osteosarcoma (osteogenic sarcoma)« CORRECT ANSWER.

N – Osteosarcoma (osteogenic sarcoma):


This is one of the commonest primary malignant bone tumours affecting the young with a slight male
predominance. There is also a second peak in incidence in the elderly related to Paget’s disease. The long
bone metaphyses are usually affected, most commonly around the knee. Bloodborne metastases develop
early and spread to the lungs and other parts of the skeleton. Cortical penetration with peri-osteal
elevation gives rise to ‘Codman’s triangle’ on X-ray. In addition, a characteristic sunray appearance may
be evident on a plain radiograph as a result of new bone formation.

In terms of histology, some may be largely fibroblastic, others are osteoblastic (with pleomorphic cells),
some comprise chondroid osteoblasts, and they may even be highly vascular (telangiectatic). All form
osteoid and/or bone-incorporating malignant cells. Computed tomography and magnetic resonance
scanning are important in staging. Treatment involves chemotherapy combined with wide surgical
excision (with limb salvage if possible). Five-year survival rates of 60% can be expected in those without
distant metastases.

Scenario 2
---------------------------------------------------------------------------------------------------------------------------------------------------------------------------------------------------------------
Dr Mohammed Shamsul Islam Khan, Medical Officer, Clinical Neuro-Surgery, National Institute of Neuro-Sciences and Hospital
Sher-E-Bangla Nagar, Dhaka-1207, Bangladesh. Mobile: +880 1713 455 662, +880 1685 811979. E-mail: drsikhan@gmail.com
MyPasTest: MRCS A Online - Jan Exam 2015
22. Surgical Specialities; Trauma & Orthopaedic Surgery (76Qs)
----------------------------------------------------------------------------------------------------------------------------------
A 51-year-old woman is referred urgently by her general practitioner with „a right upper quadrant mass, which he
suspects is malignant‟. On examination, there is no evidence of an abdominal mass but there is a tender swelling
arising from the right costal margin. An X-ray of the lesion reveals a localised area of bone destruction with
mottled appearances, affecting the ninth rib near the costal margin.
B - Chondroma « YOUR ANSWER
C - Chondrosarcoma « CORRECT ANSWER.
Page |
C – Chondrosarcoma: 1366
This is a malignant tumour of cartilage that affects the flat bones, vertebrae, girdles and the proximal limb
bones in middle-aged patients. X-ray images may be diagnostic and reveal localised bone destruction,
punctuated by mottled densities from calcification or ossification, as in the case described. Histological
differentiation from enchondroma and osteosarcoma may be difficult. Treatment is similar to that for
osteosarcomas, and 5-year survival rates are approximately 50%.

Scenario 3
A 12-year-old girl is referred with pain and swelling affecting her left lower leg. On examination, there is a tender,
irregular swelling arising from the mid-tibia. X-ray reveals a destructive lesion, associated with a soft tissue mass,
and peri-osteal „onion-skinning‟. Biopsy demonstrates the presence of sheets of „small round cells‟.
C - Chondrosarcoma « YOUR ANSWER
D - Ewing‟s sarcoma « CORRECT ANSWER.

D – Ewing’s sarcoma:
This aggressive tumour is unusual in that it often affects the mid-diaphysis (shaft) of the bone, typically in
children and adolescents. It is slightly less prevalent than osteosarcoma. It may masquerade as an
osteomyelitis. Metastases arise in the liver, lung and other bones. Radiography reveals a characteristic
‘onion-skin’ appearance of the periosteum. Survival rates of 60% can be achieved at 5 years with the use
of combined surgery, radiotherapy and chemotherapy.

Primary tumours of bone and soft tissue are uncommon. They usually present with unremitting pain,
swelling and loss of function. They may also present with a pathological fracture, a joint effusion or
systemic symptoms when there is metastatic spread. In the diagnosis of bone tumours, the frequency of
various tumours, the patient’s age, the bone affected and the location of the lesion within the bone, and
its radiological appearances are all important.

56. Theme: Myotomes


A C6 and C7
B C7 and C8
C C5 and C6
D C6, C7 and C8
E C6.

For each of the following movements, select the most likely answer from the above list. Each option may
be used once, more than once, or not at all.

Scenario 1
The segmental innervation for pronation of the forearm.
A - C6 and C7« YOUR ANSWER
---------------------------------------------------------------------------------------------------------------------------------------------------------------------------------------------------------------
Dr Mohammed Shamsul Islam Khan, Medical Officer, Clinical Neuro-Surgery, National Institute of Neuro-Sciences and Hospital
Sher-E-Bangla Nagar, Dhaka-1207, Bangladesh. Mobile: +880 1713 455 662, +880 1685 811979. E-mail: drsikhan@gmail.com
MyPasTest: MRCS A Online - Jan Exam 2015
22. Surgical Specialities; Trauma & Orthopaedic Surgery (76Qs)
----------------------------------------------------------------------------------------------------------------------------------
B - C7 and C8« CORRECT ANSWER.

Pronation of the forearm involves segments C7 and C8.

Scenario 2
The segmental innervation for supination of the forearm.
B - C7 and C8« YOUR ANSWER Page |
E - C6« CORRECT ANSWER. 1367
Segment C6 is involved in supination of the forearm.
Scenario 3
The segmental innervation for elbow flexion.
C - C5 and C6« CORRECT ANSWER.
Elbow flexion involves segments C5 and C6, whereas elbow extension involves segments C7 and C8.

Scenario 4
The segmental innervation for finger flexion.
D - C6, C7 and C8« YOUR ANSWER
B - C7 and C8« CORRECT ANSWER.
The segmental innervation for finger and thumb flexion is C7 and C8, and also C7 and C8 for extension.

57. A 7-year-old boy falls over sustaining a fractured right distal radius. On x-ray the fracture line passes
through the epiphysis into the physis with separation of the fragment, the metaphysis is intact. How
would you classify this fracture? Select one answer only.
Salter Harris I« YOUR ANSWER
Salter Harris II
Salter Harris III« CORRECT ANSWER
Salter Harris IV
Salter Harris V.

The Salter-Harris classification is a commonly used classification of epiphyseal injuries in children. There
were 5 types originally described. Type II injuries are the most common type seen.

The types are:


Salter Harris I – Separation of the epiphysis from the metaphysis
Salter Harris II – Fracture through the physis with a metaphyseal fragment
Salter Harris III – Fracture through the epiphysis and physis
Salter Harris IV – Fracture through the epiphysis, physis and metaphysis
Salter Harris V – Crush injury to the physis.

58. You are examining a 78-year-old lady with pain in both hands and difficulty with dropping objects. You
note on examination that her middle finger PIP joint is fixed in flexion of approximately 300 and the DIP
joint is fixed in hyperextension. How would you describe this finding? Select one answer only.
Bouchard‟s node« YOUR ANSWER
---------------------------------------------------------------------------------------------------------------------------------------------------------------------------------------------------------------
Dr Mohammed Shamsul Islam Khan, Medical Officer, Clinical Neuro-Surgery, National Institute of Neuro-Sciences and Hospital
Sher-E-Bangla Nagar, Dhaka-1207, Bangladesh. Mobile: +880 1713 455 662, +880 1685 811979. E-mail: drsikhan@gmail.com
MyPasTest: MRCS A Online - Jan Exam 2015
22. Surgical Specialities; Trauma & Orthopaedic Surgery (76Qs)
----------------------------------------------------------------------------------------------------------------------------------
Boutonniere deformity« CORRECT ANSWER
Heberden‟s nodes
Mallet finger
Swan neck deformity.
Page |
Bouchard’s and Heberden’s nodes, Boutonniere and swan neck deformities are all features of arthritis of 1368
the hands. Bouchard’s and Heberden’s nodes are swellings over the dorsal surface, Bouchard’s at the
PIP and Heberden’s at the DIP. In a Boutonniere deformity there is flexion of the PIP joint and extension of
the DIP. In a swan neck deformity there is flexion of the DIP and extension of the PIP. A mallet finger is
caused by rupture of the extensor tendon at the DIP joint resulting in flexion of the DIP.

59. Theme: Peripheral nerve injury


A Femoral nerve
B Common peroneal nerve
C Deep peroneal nerve
D Superficial peroneal nerve
E Sural nerve
F Tibial nerve
G Saphenous nerve.

Pick the most appropriate option from the above list. Each option may be used once only, more than once
or not at all.

Scenario 1
A 28-year-old man sustains a varus injury to his left knee while skiing. He notes loss of sensation over the
anterolateral aspect of the leg and dorsum of the foot, together with weakness of dorsiflexion and eversion of the
foot.
A - Femoral nerve« YOUR ANSWER
B - Common peroneal nerve« CORRECT ANSWER.

Common peroneal nerve:


Deep peroneal nerve injury may give rise to an anterior compartment syndrome and loss of sensation in
the first web space between the first and second toes. The common peroneal nerve is often damaged at
the level of the fibular neck by severe traction when the knee is forced into varus (eg lateral ligament
injuries and fractures around the knee) or from pressure from a splint or plaster cast. The patient has foot
drop and loss of sensation over the front and outer half of the leg and dorsum of the foot.

Scenario 2
A 32-year-old motorcyclist is involved in a road traffic accident and sustains a severe laceration 6 cm above the
ankle on the lateral aspect of his leg. He is unable to evert his foot and has noted some numbness over the
dorsum of foot and medial four toes.
B - Common peroneal nerve« YOUR ANSWER
D - Superficial peroneal nerve« CORRECT ANSWER.

Superficial peroneal nerve:

---------------------------------------------------------------------------------------------------------------------------------------------------------------------------------------------------------------
Dr Mohammed Shamsul Islam Khan, Medical Officer, Clinical Neuro-Surgery, National Institute of Neuro-Sciences and Hospital
Sher-E-Bangla Nagar, Dhaka-1207, Bangladesh. Mobile: +880 1713 455 662, +880 1685 811979. E-mail: drsikhan@gmail.com
MyPasTest: MRCS A Online - Jan Exam 2015
22. Surgical Specialities; Trauma & Orthopaedic Surgery (76Qs)
----------------------------------------------------------------------------------------------------------------------------------
The superficial peroneal nerve innervates the peroneal muscles and emerges through the deep fascia 5-
10 cm above the ankle to supply the skin over the dorsum of the foot and medial four toes.

60. An 8-year-old boy presents to the emergency department with pain in the left hip and limping. The
symptoms have been present for 24 hours, but the boy has had previous episodes of hip pain
intermittently for the past 3 months. He is small for his age, systemically well, does not recall any injury
and his bloods are normal. What is the most likely diagnosis? Select one answer only. Page |
Developmental dysplasia of the hip« YOUR ANSWER 1369
Perthes disease« CORRECT ANSWER
Septic arthritis
Slipped upper femoral epiphysis
Transient synovitis.

Limping in children can be caused by a number of conditions including all those listed. The most likely
condition varies with the patients’ age and symptoms. Developmental dysplasia of the hip is most
commonly diagnosed in babies, and is screened for, but may present late usually when the child starts
walking. Septic arthritis can occur at any age, but is more common in younger babies, it is generally
associated with systemic upset and raised inflammatory markers. Transient synovitis is a diagnosis of
exclusion. Perthes disease is a transient disruption in the blood supply to the femoral head, which is
more common in boys, those who are small for their age and between the ages of 3-12 years. Slipped
upper femoral epiphysis is also more common in boys, but tends to occur in older children, most
commonly 11-16 years, at times of growth spurts.

61. Theme: Limping child


A Septic arthritis
B Femoral neck fracture
C Slipped femoral capital epiphysis
D Rheumatoid arthritis
E Transient synovitis
F Perthes' disease
G Developmental dislocation of the hip (DDH)
H Osteoid osteoma
I Osteochondritis dessicans.

Select the most appropriate diagnosis for each of the patients below. Each option may be used once,
more than once, or not at all.

Scenario 1
A 3-year-old child has been having difficulty walking since his early steps. He walks with a waddling gait. On
examination you notice that the child has an excessive lumbar lordosis. X-ray of the hip shows small femoral
heads.
A - Septic arthritis« YOUR ANSWER
G - Developmental dislocation of the hip (DDH)« CORRECT ANSWER.

Bilateral developmental dislocation of the hip will make the child lurch to both sides and will show lumbar
lordosis to compensate for the hip flexion. Perthes’ disease is rare below the age of 4 years.
---------------------------------------------------------------------------------------------------------------------------------------------------------------------------------------------------------------
Dr Mohammed Shamsul Islam Khan, Medical Officer, Clinical Neuro-Surgery, National Institute of Neuro-Sciences and Hospital
Sher-E-Bangla Nagar, Dhaka-1207, Bangladesh. Mobile: +880 1713 455 662, +880 1685 811979. E-mail: drsikhan@gmail.com
MyPasTest: MRCS A Online - Jan Exam 2015
22. Surgical Specialities; Trauma & Orthopaedic Surgery (76Qs)
----------------------------------------------------------------------------------------------------------------------------------
Scenario 2
A 6-year-old boy presented with a painful left hip that has been non-weight bearing for the last 24 h. He feels
unwell and his mother says that he has been having a sore throat for the last week. On examination you note that
he is pyrexial (38.5°C) and any movement in the left hip joint is restricted.
B - Femoral neck fracture« YOUR ANSWER
A - Septic arthritis« CORRECT ANSWER. Page |
1370
An unwell pyrexial child with a splinted joint that does not allow movement in any direction without a
history of trauma should always be considered as septic arthritis until proven otherwise. Septic arthritis
of the hip joint is more common than of the knee in the paediatric age group.

Scenario 3
A 6-year-old child presented with a limp in his right leg. His mother reports that he has been having a sore throat
for the last week. On examination he is afebrile and feels well. Right hip range of movement is only slightly
affected. His white blood count and erythrocyte sedimentation rate are within the normal range. Upon follow-up
five days later after being on simple analgesia he completely recovers.
C - Slipped femoral capital epiphysis« YOUR ANSWER
E - Transient synovitis« CORRECT ANSWER.

This is a diagnosis of exclusion. The history of a preceding upper respiratory infection, normal
inflammatory markers and a self-limited course encourages the diagnosis.

Scenario 4
A 7-year-old child presented with a 5-week history of progressively worsening right hip pain. He has had previous
episodes of similar hip pain. He does not complain of any constitutional symptoms (fever, nausea, anorexia etc)
and on examination there is a limited abduction of the hip joint. X-ray of the hip shows increased bone density and
flattening of the femoral head.
D - Rheumatoid arthritis« YOUR ANSWER
F - Perthes' disease« CORRECT ANSWER.

Perthes’ disease is common in the age group 4–10 years; it runs a chronic course and if missed then
avascular necrosis of the femoral head will result in its collapse (flattening) and increased density as new
bone is laid.

62. Theme: Bone tumours


A Multiple myeloma
B Non-ossifying fibroma
C Metastatic malignancy
D Ollier's disease
E Giant cell tumour
F Osteosarcoma
G Enchondroma
H Ewing sarcoma
I Osteoid osteoma.

Select the most appropriate diagnosis for each of the patients below. Each option may be used once,
more than once, or not at all.
---------------------------------------------------------------------------------------------------------------------------------------------------------------------------------------------------------------
Dr Mohammed Shamsul Islam Khan, Medical Officer, Clinical Neuro-Surgery, National Institute of Neuro-Sciences and Hospital
Sher-E-Bangla Nagar, Dhaka-1207, Bangladesh. Mobile: +880 1713 455 662, +880 1685 811979. E-mail: drsikhan@gmail.com
MyPasTest: MRCS A Online - Jan Exam 2015
22. Surgical Specialities; Trauma & Orthopaedic Surgery (76Qs)
----------------------------------------------------------------------------------------------------------------------------------
Scenario 1
A 19–year-old male complains of right thigh pain for the last 3 months especially at night which responds well to
aspirin. X-ray of his right femur showed a translucent nidus in the diaphysis surrounded by an intense sclerotic
reaction.
A - Multiple myeloma« YOUR ANSWER
I - Osteoid osteoma« CORRECT ANSWER. Page |
1371
Osteoid osteoma can occur anywhere in the skeleton. The characteristic pain worse at night responding
to aspirin is peculiar to this tumour.

Scenario 2
A 28-year-old female teacher presented with left knee pain and swelling for the last two weeks, otherwise she is fit
and healthy. X-ray of the left knee showed a large translucent lesion in the proximal tibia with hazy margins
extending into the epiphysis and eroding the articular cartilage.
B - Non-ossifying fibroma« YOUR ANSWER
E - Giant cell tumour« CORRECT ANSWER.

Giant cell tumours most commonly affect the epiphysis more than any other tumour in this age group. A
non-ossifying fibroma can be included in the differential diagnosis but it will have a sclerotic margin.

Scenario 3
A 49-years-old female presented to the emergency department sustaining a right neck of femur following a trivial
trauma. X-ray of the right hip also showed multiple lytic lesions. Past medical history reveals that she had a left
mastectomy two years ago.
C - Metastatic malignancy« CORRECT ANSWER
A history of previous malignancy with this characteristic appearance is strongly suggestive of metastatic
malignancy even if the original tumour is not clinically evident yet. Multiple myeloma may be a differential
diagnosis but it occurs in older age group.

Scenario 4
A 13 year-old-boy presented with a painful and swollen left leg for the last 3 weeks that is progressively
worsening. On examination he is also febrile and blood investigations show leucocytosis. X-ray of the left leg
shows a patchy density in the left fibula diaphysis surrounded by a lamellated periosteal reaction.
D - Ollier's disease« YOUR ANSWER
H - Ewing sarcoma« CORRECT ANSWER.

Ewing sarcoma is common in this age group mainly confused with osteomyelitis. Onion skin appearance
(lamellated periosteal reaction) is typical of this tumor.

63. A 35-year-old man presents to the emergency department with a 24-hour history of severe throbbing
pain in the finger tip of his right index finger. He injured his hand whilst gardening 3 days before and
thinks he cut his hand on a rose thorn. The fingertip is erythematous and swollen up to the DIP joint, but
not proximal to it. What is the most likely diagnosis? Select one answer only.
Apical infection« YOUR ANSWER
Felon« CORRECT ANSWER
Herpetic whitlow
---------------------------------------------------------------------------------------------------------------------------------------------------------------------------------------------------------------
Dr Mohammed Shamsul Islam Khan, Medical Officer, Clinical Neuro-Surgery, National Institute of Neuro-Sciences and Hospital
Sher-E-Bangla Nagar, Dhaka-1207, Bangladesh. Mobile: +880 1713 455 662, +880 1685 811979. E-mail: drsikhan@gmail.com
MyPasTest: MRCS A Online - Jan Exam 2015
22. Surgical Specialities; Trauma & Orthopaedic Surgery (76Qs)
----------------------------------------------------------------------------------------------------------------------------------
Paronychia
Flexor tendon sheath infection.

A felon is an abscess in the compartments of the pulp. This is usually more painful than a paronychia.
The swelling doesn’t extend proximal to the distal interphalangeal joint. The infection often follows a
penetrating trauma and is most common in the thumb and index finger. Page |
1372
Paronychia occur between the tip of the nail and the cuticle. Apical infections occur between the top of
the nail and the underlying nail bed. Herpetic whitlow iscaused by the herpes simplex virus, small clear
vesicles are seen. Flexor tendon sheath infection is a surgical emergency with a sausage shaped digit,
flexed position, tenderness over the flexor tendon sheath and pain on passive extension.
64. Theme: Bone conditions
A Osteoporosis
B Scheuermann's kyphosis
C Metastatic carcinoma
D Osteoid osteoma
E Giant cell tumour.

For each of the patients described below, select the single most likely diagnosis from the options listed
above. Each option may be used once, more than once, or not at all.

Scenario 1
A 68-year-old frail woman has suffered with back pain for 4 years and has developed slight kyphosis in the last
year. She underwent a hysterectomy and bilateral salpingo-oophorectomy at the age of 35. She presents with a
fracture of the left neck femur after slipping at home.
A - Osteoporosis« CORRECT ANSWER.

Osteoporosis can be caused by menopausal hormonal failure, Cushing’s disease, steroids, thyrotoxicosis
and prolonged bed rest. Bone scan is diagnostic.

Scenario 2
A 30-year-old woman presents with pain and swelling in her thoracic region. Radiographs show a small lesion,
with a well-demarcated central nidus surrounded by dense reactive bone in T6.
B - Scheuermann's kyphosis « YOUR ANSWER
D - Osteoid osteoma« CORRECT ANSWER.

Osteiod osteomas are benign osteoblastic lesions. They typically occur in people between 5 and 30 years
old.

Scenario 3
An x-ray shows a large radiolucent zone in the head of the right humerus extending to the subchondral plate in a
29-year-old man, with localised pain.
C - Metastatic carcinoma« YOUR ANSWER
E - Giant cell tumour« CORRECT ANSWER.

Giant cell tumours typically affect patients between 20 and 60 years old. They occur more commonly in
women then men.
---------------------------------------------------------------------------------------------------------------------------------------------------------------------------------------------------------------
Dr Mohammed Shamsul Islam Khan, Medical Officer, Clinical Neuro-Surgery, National Institute of Neuro-Sciences and Hospital
Sher-E-Bangla Nagar, Dhaka-1207, Bangladesh. Mobile: +880 1713 455 662, +880 1685 811979. E-mail: drsikhan@gmail.com
MyPasTest: MRCS A Online - Jan Exam 2015
22. Surgical Specialities; Trauma & Orthopaedic Surgery (76Qs)
----------------------------------------------------------------------------------------------------------------------------------
Scenario 4
A 70-year-old presents with a large tumour in the left shoulder. He also complains of increasing difficulty in
swallowing and most recently in breathing. On examination, he has a large hard irregular thyroid swelling.
D - Osteoid osteoma« YOUR ANSWER
C - Metastatic carcinoma« CORRECT ANSWER.
Page |
Metastatic carcinoma is the commonest malignancy in bone. The majority originate from the breast, 1373
prostate, lung, kidney and thyroid.

65. Theme: Peripheral nerve anatomy


A Axillary
B Long thoracic
C Medial pectoral
D Median
E Musculocutaneous
F Radial
G Suprascapular
H Thoracodorsal
I Ulnar
J Upper subscapular.

For each of the patients listed below, select the nerve most likely to be involved from the above list. Each
option may be used once, more than once, or not at all.

Scenario 1
While playing football, a young man dislocates his right shoulder. The dislocation is reduced soon after. Once the
shoulder is painfree, he notices that he cannot carry weights with his right arm and is unable to raise his arm from
his side for more than a few degrees. Neurological examination reveals loss of abduction and blunted sensation
over the skin covering the lateral part of the deltoid muscle. All reflexes are normal.
A - Axillary« CORRECT ANSWER.

A – Axillary:
The axillary nerve passes just below the capsule of the shoulder joint and is damaged here in about 5% of
shoulder dislocations. The nerve gives off the upper lateral cutaneous nerve of the arm, and also motor
branches to the deltoid and teres minor muscles. The multipennate fibres of the deltoid muscle contract
isometrically when carrying weights in the hand. The strap/unipennate anterior and posterior slips of the
muscle are used for flexion and extension and, when contracting together, take over from the
supraspinatus muscle to abduct the arm beyond the first 15º.

Scenario 2
After a mastectomy, a 40-year-old woman loses the ability to fold her right arm behind her back and reach up to
the opposite scapula.
B - Long thoracic « YOUR ANSWER
H - Thoracodorsal« CORRECT ANSWER.

H – Thoracodorsal:

---------------------------------------------------------------------------------------------------------------------------------------------------------------------------------------------------------------
Dr Mohammed Shamsul Islam Khan, Medical Officer, Clinical Neuro-Surgery, National Institute of Neuro-Sciences and Hospital
Sher-E-Bangla Nagar, Dhaka-1207, Bangladesh. Mobile: +880 1713 455 662, +880 1685 811979. E-mail: drsikhan@gmail.com
MyPasTest: MRCS A Online - Jan Exam 2015
22. Surgical Specialities; Trauma & Orthopaedic Surgery (76Qs)
----------------------------------------------------------------------------------------------------------------------------------
The thoracodorsal nerve is most vulnerable to damage during axillary surgery, when the arm is laterally
rotated and abducted, because it bows into the axilla from the posterior wall. Paralysis of the latissimus
dorsi muscle is detected clinically if the patient is unable to fold the arm behind the back and reach up to
the opposite scapula. The intercostobrachial (sensory) nerve is also vulnerable and, occasionally, has to
be sacrificed.

Scenario 3 Page |
A 15-year-old boy riding in the passenger seat of a car escapes any apparent injury after a head-on collision 1374
because he was wearing a seat belt. However, after the accident he was unable to raise his arm easily and has
visited the Emergency Department twice with spontaneous dislocation of the shoulder.
C - Medial pectoral « YOUR ANSWER
G - Suprascapular« CORRECT ANSWER.

G – Suprascapular:
The suprascapular nerve, which is motor to the supraspinatus and infraspinatus muscles, may be
damaged by sudden tightening of a car seatbelt (upper trunk injuries of the brachial plexus and clavicular
fractures may also occur). Paralysis of both muscles weakens the rotator cuff, destabilising the shoulder
joint. In addition, the supraspinatus abducts the arm from 0º to 15º, and the infraspinatus is a powerful
lateral rotator of the humerus. Since the teres minor muscle is unaffected (axillary nerve), some lateral
rotation is preserved after suprascapular nerve damage.

Scenario 4
A builder falls off a scaffolding on to his right side, fracturing his right humerus. Because of the patient‟s shocked
state and the pain, only a limited neurological examination is possible. This reveals an absence of the
brachioradialis reflex, blunted cutaneous sensation over the first dorsal interosseous muscle and wrist drop.
D - Median« YOUR ANSWER
F - Radial« CORRECT ANSWER.

F – Radial:
Mid-shaft fractures of the humerus can damage the radial nerve in the spiral groove. The branches to the
triceps are given off before the nerve enters the groove and so the muscle remains functional. All other
extensors are paralysed, resulting in wrist drop. Although the cutaneous branches no longer conduct,
compensatory overlap by adjacent nerves restricts the paraesthesia/anaesthesia to the dorsal skin over
the first interosseous muscle. The brachioradialis reflex is mediated by the radial nerve and is thus lost.
The triceps reflex remains intact for reasons explained above.

Scenario 5
After a radical mastectomy, a 41-year-old woman is unable to push a loaded supermarket trolley with her right
arm. Her husband has noticed a deformity in her upper back on the right side that becomes more prominent when
she pushes against resistance with the outstretched right arm.
E - Musculocutaneous« YOUR ANSWER
B - Long thoracic « CORRECT ANSWER.

B – Long thoracic:
The long thoracic nerve usually escapes damage during axillary surgery because it is bound to the
serratus anterior muscle by overlying fascia on the medial wall, posterior to the mid-axillary line. When
the nerve is injured, however, part or all of the serratus anterior muscle is paralysed, resulting in a

---------------------------------------------------------------------------------------------------------------------------------------------------------------------------------------------------------------
Dr Mohammed Shamsul Islam Khan, Medical Officer, Clinical Neuro-Surgery, National Institute of Neuro-Sciences and Hospital
Sher-E-Bangla Nagar, Dhaka-1207, Bangladesh. Mobile: +880 1713 455 662, +880 1685 811979. E-mail: drsikhan@gmail.com
MyPasTest: MRCS A Online - Jan Exam 2015
22. Surgical Specialities; Trauma & Orthopaedic Surgery (76Qs)
----------------------------------------------------------------------------------------------------------------------------------
‘winged scapula’. There is loss of protraction and weakness of rotation of the scapula (the latter
movement is, however, preserved by the action of the intact trapezius muscle – spinal accessory nerve).

66. A 70-year-old man presents with increasing pain in the lumbar spine for 3 months with night pain. He
has no neurological signs or symptoms. X-rays reveal a sclerotic lesion in the L4 vertebra. What is the
most likely primary diagnosis? Select one answer only.
Page |
Breast cancer« YOUR ANSWER
1375
Lung cancer
Prostate cancer« CORRECT ANSWER
Renal cancer
Thyroid cancer.

The five commonest tumours metastasizing to bone are breast, lung, prostate, renal and thyroid. A
prostate cancer is most likely in a gentleman of this age, and prostate metastases tend to be sclerotic.
Thyroid and renal metastases tend to be lytic, and breast and lung metastases tend to be mixed.

67. Theme: Peripheral nerve entrapment


A Ulnar nerve
B Anterior interosseous nerve
C Tibial nerve
D Common peroneal nerve
E Radial nerve
F Median nerve
G Femoral nerve
H Lateral femoral cutaneous nerve
I Axillary nerve.

Select the nerve that is most likely to be involved in each of the patients below. Each option may be used
once, more than once, or not at all.

Scenario 1
A 35-year-old pregnant lady in her third trimester presented with pain on the side of her left thigh. She noticed that
the pain is worse on walking and is relieved by sitting. On examination you notice that motor power and tone of
the left lower limb is not affected.
A - Ulnar nerve« YOUR ANSWER
H - Lateral femoral cutaneous nerve« CORRECT ANSWER

Meralgia paresthetica is entrapment of the lateral femoral cutaneous nerve (purely sensory) at the
inguinal ligament especially in pregnant women and obese people.

Scenario 2
A 56-year-old lady with known rheumatoid arthritis complains of tingling sensation in her thumb and index finger
especially at night. On examination you notice there is wasting of the thenar muscles.
B - Anterior interosseous nerve« YOUR ANSWER
F - Median nerve« CORRECT ANSWER.

---------------------------------------------------------------------------------------------------------------------------------------------------------------------------------------------------------------
Dr Mohammed Shamsul Islam Khan, Medical Officer, Clinical Neuro-Surgery, National Institute of Neuro-Sciences and Hospital
Sher-E-Bangla Nagar, Dhaka-1207, Bangladesh. Mobile: +880 1713 455 662, +880 1685 811979. E-mail: drsikhan@gmail.com
MyPasTest: MRCS A Online - Jan Exam 2015
22. Surgical Specialities; Trauma & Orthopaedic Surgery (76Qs)
----------------------------------------------------------------------------------------------------------------------------------
Carpal tunnel syndrome (median nerve entrapment) is common in the hands of the rheumatoid arthritis
population.

Scenario 3
A 30-year-old lady was put in a left below-knee cast for lateral malleolus fracture. Upon removal of the cast 4
weeks later you notice that there is a foot drop and anaesthesia of dorsum of the foot.
C - Tibial nerve« YOUR ANSWER Page |
D - Common peroneal nerve« CORRECT ANSWER. 1376
In below-knee casts, care must be taken not trap the common peroneal nerve against the upper edge of
the cast, because the nerve here is very superficial and lies against bone (neck of fibula).
Scenario 4
A 25-year-old male presented with weakness of his left hand grip. Past medical history revealed that he sustained
an elbow fracture when he was a child. On examination you notice wasting of the dorsal interosseous muscles
and altered sensation in the little and ring fingers.
I - Axillary nerve« YOUR ANSWER
A - Ulnar nerve« CORRECT ANSWER.
Tardive ulnar nerve palsy can follow a fracture of the lateral epicondyle at the elbow during childhood that
halts growth laterally, while the medial epicondyle will continue to grow causing the carrying angle to
increase and ulnar nerve entrapment.
68. Theme: Life-threatening chest injuries
A Traumatic aortic rupture
B Massive haemothorax (right)
C Tension pneumothorax (left)
D Cardiac tamponade.

For each of the physical signs described below, select the most likely diagnosis from the list of
conditions above. Each option may be used once, more than once or not at all.

Scenario 1
Beck‟s triad
A - Traumatic aortic rupture« YOUR ANSWER
D - Cardiac tamponade« CORRECT ANSWER
Cardiac tamponade

Tension pneumothorax and cardiac tamponade produce similar signs, in that they cause distension of the
neck veins (on inspiration = Kussmaul's sign), cyanosis and hypotension. Beck’s triad comprises
hypotension, distended neck veins and muffled heart sounds and is characteristic of cardiac tamponade.

Scenario 2
Tracheal deviation to the right
B - Massive haemothorax (right)« YOUR ANSWER
C - Tension pneumothorax (left)« CORRECT ANSWER.

Tension pneumothorax (left):


Tension pneumothorax causes tracheal deviation away from the side of the injury. Traumatic aortic
rupture and massive haemothorax can cause tracheal deviation but these are uncommon signs.
---------------------------------------------------------------------------------------------------------------------------------------------------------------------------------------------------------------
Dr Mohammed Shamsul Islam Khan, Medical Officer, Clinical Neuro-Surgery, National Institute of Neuro-Sciences and Hospital
Sher-E-Bangla Nagar, Dhaka-1207, Bangladesh. Mobile: +880 1713 455 662, +880 1685 811979. E-mail: drsikhan@gmail.com
MyPasTest: MRCS A Online - Jan Exam 2015
22. Surgical Specialities; Trauma & Orthopaedic Surgery (76Qs)
----------------------------------------------------------------------------------------------------------------------------------
Scenario 3
Most commonly caused by penetrating right chest injury
C - Tension pneumothorax (left)« YOUR ANSWER
B - Massive haemothorax (right)« CORRECT ANSWER.

Massive haemothorax (right): Page |


Tension pneumothorax and traumatic aortic rupture are usually due to blunt trauma. Cardiac tamponade 1377
can be due to penetrating trauma, but haemothorax is much more common as a result of penetrating
trauma.

69. Theme: Multiple trauma


A Cardiac tamponade
B Flail chest
C Haemothorax
D Tension pneumothorax
E Aortic rupture
F Ruptured spleen.

For each patient described below, select the most likely single diagnosis from the list of options above.
Each option may be used once, more than once or not at all.

Scenario 1
A 57-year-old man is brought into the emergency department following a road traffic accident. He is conscious
with an adequate airway. The Glasgow Coma Score (GCS) is 15. He is tachypnoeic (30 rpm). His pulse is 150
bpm and reduced in volume. The distal pulses are present. His blood pressure (BP) is 85/45 mm Hg. The chest X-
ray shows multiple rib fractures on the left side (ribs 7, 8 and 9) and a small pneumothorax. A left chest drain is
inserted and bubbles in the waterseal bottle. Pelvic X-ray is normal. Analysis of the diagnostic peritoneal lavage
fluid reveals a white cell count (WCC) of 1000/mm3 , 200,000 RBCs/mm3 and an amylase of 400 units. He
remains hypotensive (BP 90/55 mm Hg) despite 2 l of fluid resuscitation.
A - Cardiac tamponade« YOUR ANSWER
F - Ruptured spleen« CORRECT ANSWER.

Ruptured spleen:
Hypotension, tachycardia and low CVP suggest hypovolaemia. Multiple rib fractures on the left side and
positive DPL support the diagnosis of splenic rupture.

Scenario 2
A 30-year-old man is brought into the emergency department following a road traffic accident. He is conscious. He
has tachypnoea (46rpm), tachycardia (135 bpm) and distended neck veins. His systemic BP is 90/45 mmHg. His
heart sounds are greatly diminished. A chest X-ray shows three fractured ribs (ribs 5, 6 and 7) on the left side and
a small pneumothorax. A left chest drain drains air and no blood, the BP is 95/50 mmHg after 2 l of crystalloid
infusion. The electrocardiogram (ECG) shows reduced voltage in QRS complexes.
B - Flail chest« YOUR ANSWER
A - Cardiac tamponade« CORRECT ANSWER.

Cardiac tamponade:

---------------------------------------------------------------------------------------------------------------------------------------------------------------------------------------------------------------
Dr Mohammed Shamsul Islam Khan, Medical Officer, Clinical Neuro-Surgery, National Institute of Neuro-Sciences and Hospital
Sher-E-Bangla Nagar, Dhaka-1207, Bangladesh. Mobile: +880 1713 455 662, +880 1685 811979. E-mail: drsikhan@gmail.com
MyPasTest: MRCS A Online - Jan Exam 2015
22. Surgical Specialities; Trauma & Orthopaedic Surgery (76Qs)
----------------------------------------------------------------------------------------------------------------------------------
The presence of distended neck veins, and failure to respond to fluid resuscitation strongly suggest the
diagnosis of cardiac tamponade and exclude rupture of the aorta and the spleen as the possible cause of
hypotension. Diminished heart sounds and reduced voltage of QRS complexes are recognised features of
cardiac tamponade.

70. Theme: Backache


A Disc herniation Page |
B Sickle cell disease 1378
C Ankylosing spondylitis
D Tuberculosis
E Psoriatic arthritis
F Multiple myeloma
G Paget's disease
H Osteoporosis
I Ruptured abdominal aortic aneurysm.

Select the most appropriate diagnosis for each of the patients below. Each option may be used once,
more than once, or not at all.

Scenario 1
A 70-year-old lady presented with low back pain, lethargy, malaise, fever and weight loss. On examination she
appears to be anaemic. Blood investigations show a high erythrocyte sedimentation rate (95 mm/h),
hypercalcaemia and anaemia. Urine analysis shows Bence–Jones proteins. X-ray of the lumbar spine shows
multiple punched out lytic lesions which are also evident in both innominate bones.
A - Disc herniation« YOUR ANSWER
F - Multiple myeloma« CORRECT ANSWER.

Multiple punched out lytic lesions are typical of multiple myeloma. Another place to look is the skull
which also shows multiple round punched out lytic lesions. Plasma protein electrophoresis will show a
prominent M-band.

Scenario 2
A 41-year-old lady presented with a low backache for the last 4 weeks. On examination you find that she has a
goitre and exophthalmos. Blood investigations show thyrotoxicosis, normal plasma calcium, phosphate and
alkaline phosphatase. Lumbar spine X-ray shows a compression fracture of L4.
B - Sickle cell disease« YOUR ANSWER
H - Osteoporosis« CORRECT ANSWER.

Thyrotoxicosis is an established cause of osteoporosis by increasing bone turnover. Serum alkaline


phosphatase levels are typically normal in osteoporosis (as are serum calcium and serum phosphate
levels).

Scenario 3
A 35-year-old male presents with low backache radiating to the left leg for the last 2 weeks after starting body-
building exercises. On examination you notice he has scoliosis and left-sided weakness of ankle plantar flexion
and hip extension
C - Ankylosing spondylitis« YOUR ANSWER
A - Disc herniation« CORRECT ANSWER.
---------------------------------------------------------------------------------------------------------------------------------------------------------------------------------------------------------------
Dr Mohammed Shamsul Islam Khan, Medical Officer, Clinical Neuro-Surgery, National Institute of Neuro-Sciences and Hospital
Sher-E-Bangla Nagar, Dhaka-1207, Bangladesh. Mobile: +880 1713 455 662, +880 1685 811979. E-mail: drsikhan@gmail.com
MyPasTest: MRCS A Online - Jan Exam 2015
22. Surgical Specialities; Trauma & Orthopaedic Surgery (76Qs)
----------------------------------------------------------------------------------------------------------------------------------
Intervertebral disc herniation is common in the young age group because the nucleus is still gelatinous,
unlike in the elderly when it is degenerative. Commonly taking place at level L5/S1 it will affect the nerve
root below i.e. S1 in this case hence the ipsilaleral weakness of ankle plantar flexion and hip extension.

Scenario 4
A 75-year-old grossly obese male presented with sudden onset of low backache associated with nausea,
vomiting, palpitation and feeling clammy and sweaty. He has a long-standing history of ischaemic heart disease. Page |
On examination you notice pallor, tachycardia and hypotension. Abdominal examination is difficult due to body 1379
habitus.
D - Tuberculosis« YOUR ANSWER
I - Ruptured abdominal aortic aneurysm« CORRECT ANSWER.

In an elderly patient presenting with sudden back pain associated with dynamic instability particularly
with comorbidity suggestive of atherosclerosis, ruptured AAA should be ruled out first.

71. Theme: Common disorders of the hand


A Carpal tunnel syndrome
B Dupuytren’s disease
C Extensor tendon injury
D Flexor tendon injury
E Gamekeeper’s thumb
F Ganglion
G Heberden’s node
H Mallet finger
I Paronychia
J Phalangeal enchondroma
K Pulp space infection
L Pyogenic granuloma
M Rheumatoid arthritis
N Trigger finger.

The following patients present with disorders of the hand. From the list above, select the most likely
diagnosis. The items may be used once, more than once, or not at all.

Scenario 1
A 44-year-old man attends the clinic and reports that his index finger often gets stuck and „clicks‟ when he
straightens it. He denies any history of trauma. He has no relevant past medical history. On examination, the
finger, and rest of the hand, appears normal. There is no contracture of the skin or subcutaneous tissues. Initially,
the index finger is fixed in flexion, but it suddenly extends fully during active movement. There is no associated
pain.
B - Dupuytren‟s disease « YOUR ANSWER
N - Trigger finger« CORRECT ANSWER.

N – Trigger finger:
This condition is caused by thickening of the flexor tendon, paratenon, or a narrowing of the flexor
sheath. Consequently, the affected finger becomes locked in full flexion and will only extend after
excessive voluntary effort, or assistance from the other hand. When extension begins it does so
suddenly, and with a click, hence the name of the condition. The condition is usually painless. Steroid
---------------------------------------------------------------------------------------------------------------------------------------------------------------------------------------------------------------
Dr Mohammed Shamsul Islam Khan, Medical Officer, Clinical Neuro-Surgery, National Institute of Neuro-Sciences and Hospital
Sher-E-Bangla Nagar, Dhaka-1207, Bangladesh. Mobile: +880 1713 455 662, +880 1685 811979. E-mail: drsikhan@gmail.com
MyPasTest: MRCS A Online - Jan Exam 2015
22. Surgical Specialities; Trauma & Orthopaedic Surgery (76Qs)
----------------------------------------------------------------------------------------------------------------------------------
infiltration may be effective in mild cases, although surgical release of the proximal portion of the A1
pulley may be necessary.

Scenario 2
A 64-year-old woman attends The Emergency Department with a fracture of the distal phalanx of her left middle
finger. She informs you that this finger has become increasingly painful over the last few months, and that she has
been aware of a „bony swelling‟ affecting this finger. She indicates that this was most marked on the volar aspect Page |
of the distal phalanx, close to the distal interphalangeal joint. 1380
A - Carpal tunnel syndrome « YOUR ANSWER
J - Phalangeal enchondroma« CORRECT ANSWER.

J – Phalangeal enchondroma:
This benign tumour is composed of mature, hyaline cartilage, and presents as a slow-growing mass
within a phalanx. Pain, swelling or deformity of the affected finger may be evident. The presentation may
be acute, with a ‘pathological’ fracture through the weakened cortex, as in the case described. There is a
characteristic appearance on radiography. The usual opacity of the bony phalanx is lost, and the cavity of
the mass appears radiolucent with stippled calcification. The cortex of the bone may be thinned as the
internal mass expands. Treatment involves curettage followed by cancellous bone grafting.

Scenario 3
A 28-year-old manual labourer attends The Emergency Department with a painful left index finger. He sustained a
minor abrasion to the palmar aspect of his left index finger at work 6 days ago. On examination, there is erythema
and swelling affecting the distal aspect of the left index finger on the palmar surface.
C - Extensor tendon injury « YOUR ANSWER
K - Pulp space infection« CORRECT ANSWER.

K – Pulp space infection:


Pulp space infections usually arise from minor penetrating injuries. Pressure in the infected compartment
causes marked pain. Infection may spread into adjacent compartments because of infarction of
surrounding tissues secondary to rapidly increasing pressure. Occasionally, this may lead to rupture
through the overlying skin, or into the distal phalanx. Treatment should involve early incision and
drainage, to avoid permanent loss of pulp tissue, and subsequent reduction of cushioning of the distal
phalanx.

72. Theme: Complications of orthopaedic surgery


A Aseptic loosening
B Arthrofibrosis
C Compartment syndrome
D Deep venous thrombosis
E Fat embolism syndrome
F Delayed union
G Periprosthetic fracture
H Pulmonary embolism
I Septic arthritis.

For each of the following clinical scenarios select the most likely diagnosis from the list above. Each
option may be used once, more than once, or not at all.

---------------------------------------------------------------------------------------------------------------------------------------------------------------------------------------------------------------
Dr Mohammed Shamsul Islam Khan, Medical Officer, Clinical Neuro-Surgery, National Institute of Neuro-Sciences and Hospital
Sher-E-Bangla Nagar, Dhaka-1207, Bangladesh. Mobile: +880 1713 455 662, +880 1685 811979. E-mail: drsikhan@gmail.com
MyPasTest: MRCS A Online - Jan Exam 2015
22. Surgical Specialities; Trauma & Orthopaedic Surgery (76Qs)
----------------------------------------------------------------------------------------------------------------------------------
Scenario 1
A 25-year-old male patient sustained a closed right femoral shaft fracture in a motorcycling accident 1 day
previously. He now complains of difficulty breathing. On examination he is increasingly anxious, with a low-grade
pyrexia, tachycardia and respiratory rate of 36 breaths/min. In the past few hours nursing staff have noted a
declining urine output and a rash on his upper chest.
A - Aseptic loosening« YOUR ANSWER
E - Fat embolism syndrome« CORRECT ANSWER Page |
1381
FES is a clinical diagnosis. Trauma to the long bone or pelvis accounts for ~90% of cases. The exact
frequency is unknown, as the diagnosis is often missed or masked in the trauma patient. However it
carries a mortality of 10–20% and must always be considered. Patients present with tachycardia,
tachypnoea, pyrexia, reduced consciousness and hypoxia. They may develop petechial rash on the upper
chest (<50%) and/or conjunctiae, oral mucosa and retinae.

Searching for fat droplets in the urine, blood or cerebrospinal fluid (CSF) may help in confirming the
diagnosis. Management is mainly supportive and consists primarily of maintaining good arterial
oxygenation. Prompt surgical stabilization of fractures reduces the risk of occurrence. The
pathophysiology is still debated, but it is thought to be caused by embolic marrow fat coalescing in
pulmonary vasculature (and other areas of important microcirculation, eg cerebral) leading to pulmonary
insufficiency and an acute respiratory distress syndrome (ARDS)-like picture.

Scenario 2
A 47-year-old lady is admitted to the orthopaedic ward with a closed fractured tibia and fibula in her right leg.
Whilst awaiting surgery the following day she complains of increasing pain in the right calf and knee preventing
her from sleeping. This is unrelieved with analgesia and is sufficient to prevent flexion the ankle joint on the
affected side.
B - Arthrofibrosis« YOUR ANSWER
C - Compartment syndrome« CORRECT ANSWER.

This arises from swelling within a closed fascial compartment impairing capillary bed blood flow, in turn
causing further swelling due to progressing ischaemia. There should be a low index of suspicion for
making this diagnosis, as irreversible ischaemia can rapidly develop. The initial swelling may result from
fracture (commonly tibial or forearm), soft-tissue injury (crush, burns) or vascular insult (ischaemia-
reperfusion injury).

Clinical features include:


01. excessive pain on passive movement of distal joints supplied by muscles of the affected
compartment/s
02. increasing pain despite immobilisation and analgesia
03. altered sensation in the distribution of nerves passing through the affected compartment/s.

Peripheral pulses may still be present as compartment pressure will rise high enough to occlude diastolic
venous flow, but rarely systolic arterial flow. Diagnosis can be aided by measuring intracompartmental
pressure. Definitive treatment is by prompt fasciotomy with the wound left open.

Scenario 3

---------------------------------------------------------------------------------------------------------------------------------------------------------------------------------------------------------------
Dr Mohammed Shamsul Islam Khan, Medical Officer, Clinical Neuro-Surgery, National Institute of Neuro-Sciences and Hospital
Sher-E-Bangla Nagar, Dhaka-1207, Bangladesh. Mobile: +880 1713 455 662, +880 1685 811979. E-mail: drsikhan@gmail.com
MyPasTest: MRCS A Online - Jan Exam 2015
22. Surgical Specialities; Trauma & Orthopaedic Surgery (76Qs)
----------------------------------------------------------------------------------------------------------------------------------
Ten years following a left total-hip arthroplasty for osteoarthritis a 75-year-old lady is referred to orthopaedic out-
patients clinic with worsening left hip pain and stiffness. This is worse on walking, when she feels this leg has
become a little shorter. Inflammatory markers are normal.
C - Compartment syndrome« YOUR ANSWER
A - Aseptic loosening« CORRECT ANSWER.

Aseptic loosening means loosening of a prosthetic joint without the involvement of bacteria. It occurs as Page |
a late complication, affecting approximately 5–10% of total-hip arthroplasties. Foreign body components 1382
worn off the artificial surfaces initiate an inflammatory reaction involving macrophages, causing
osteolysis of the surrounding bone. Patients present with pain and if the femoral component loosens,
sinking further into the femoral shaft, a decreased leg length may ensue. It is important to exclude late
infection as a cause of the pain. Diagnosis is aided by serial X-rays showing changing orientation of the
components and progressive lines around the bone–prosthesis interface. Treatment is by revision of the
joint.

Scenario 4
A teenage smoker suffers a closed right lateral malleolar fracture during a fall on an ice rink. The fracture is
undisplaced and she is treated non-operatively in a plaster cast. Six weeks later she still feels severe pain on
moving the ankle joint and attempting to weight bear, and there is continuing tenderness over the lateral
malleolus.
D - Deep venous thrombosis« YOUR ANSWER
F - Delayed union« CORRECT ANSWER.

This is a failure of fractured bone-ends to form a bony union within the usual time period for that fracture
and patient. There is no exactly defined time period that this can be diagnosed by. Typically the fracture
site remains tender and may be mobile. It remains visible on X-rays with little callous formation. Causes
include: infection, inadequate immobilization (allowing movement at the fracture site), poor blood supply.
Smoking has been linked to delayed-union and non-union. If no union develops after an appropriate time
period, internal fixation or bone grafting could be considered.

73. Theme: Fracture management


A Cannulated screws
B Dynamic hip screw
C External fixator
D Fix with plate and screws
E Fix with screws
F Fix with wires
G Hemiarthroplasty
H Intramedullary nail
I Manipulate under general anaesthesia (MUA)
J Manipulation followed by plaster.

For each of the following situations, select the most likely answer from the above list. Each option may be
used once, more than once, or not at all.

Scenario 1
A 33-year-old lady sustains an inversion injury to her left ankle and presents with a bimalleolar fracture.
A - Cannulated screws« YOUR ANSWER
---------------------------------------------------------------------------------------------------------------------------------------------------------------------------------------------------------------
Dr Mohammed Shamsul Islam Khan, Medical Officer, Clinical Neuro-Surgery, National Institute of Neuro-Sciences and Hospital
Sher-E-Bangla Nagar, Dhaka-1207, Bangladesh. Mobile: +880 1713 455 662, +880 1685 811979. E-mail: drsikhan@gmail.com
MyPasTest: MRCS A Online - Jan Exam 2015
22. Surgical Specialities; Trauma & Orthopaedic Surgery (76Qs)
----------------------------------------------------------------------------------------------------------------------------------
D - Fix with plate and screws« CORRECT ANSWER.

Stable, undisplaced malleolar fractures can be treated with POP. Bimalleolar and trimalleolar fractures of
the ankle are generally unstable injuries and require open reduction and internal fixation (ORIF) usually
with plates and screws.

Scenario 2 Page |
An 88-year-old lady who lives in a nursing home falls onto her outstretched right hand. An X-ray reveals a fracture 1383
of the distal radius with dorsal and radial displacement.
B - Dynamic hip screw« YOUR ANSWER
J - Manipulation followed by plaster« CORRECT ANSWER.

The most appropriate management for a patient in this situation would be to reduce the fracture by
manipulation and apply a cast.

Scenario 3
A 23-year-old gentleman falls off his bicycle onto his left wrist. An X-ray reveals an intra-articular fracture of the
distal radius with volar displacement.
C - External fixator« YOUR ANSWER
D - Fix with plate and screws« CORRECT ANSWER.

Intra-articular fractures require accurate anatomical reduction and stabilisation. Distal radial fractures
with volar displacement are usually unstable injuries. Therefore in this situation, the most appropriate
management would be ORIF with a buttress plate and screws.

Scenario 4
A 55-year-old lady trips and falls onto her left hip. An X-ray shows an undisplaced intracapsular fracture of the
femoral neck.
D - Fix with plate and screws« YOUR ANSWER
A - Cannulated screws« CORRECT ANSWER.

Intracapsular neck of femur fractures in young patients require anatomical reduction and fixation with
cannulated screws as soon as possible to try and preserve the blood supply to the femoral head and
therefore prevent avascular necrosis.

Scenario 5
A 76-year-old gentleman falls onto his right hip. An X-ray shows a displaced intracapsular fracture of the femoral
neck.
E - Fix with screws« YOUR ANSWER
G - Hemiarthroplasty« CORRECT ANSWER.

Conversely, a hemiarthroplasty is used in older patients with intracapsular femoral fractures.

Scenario 6
An 81-year-old lady falls onto her right hip whilst getting out of the shower. On examination, the hip is short and
externally rotated. An X-ray shows a three-part intertrochanteric fracture.
F - Fix with wires« YOUR ANSWER
B - Dynamic hip screw« CORRECT ANSWER.
---------------------------------------------------------------------------------------------------------------------------------------------------------------------------------------------------------------
Dr Mohammed Shamsul Islam Khan, Medical Officer, Clinical Neuro-Surgery, National Institute of Neuro-Sciences and Hospital
Sher-E-Bangla Nagar, Dhaka-1207, Bangladesh. Mobile: +880 1713 455 662, +880 1685 811979. E-mail: drsikhan@gmail.com
MyPasTest: MRCS A Online - Jan Exam 2015
22. Surgical Specialities; Trauma & Orthopaedic Surgery (76Qs)
----------------------------------------------------------------------------------------------------------------------------------
An intertrochanteric fracture of the femoral neck is an example of an extracapsular fracture. The femoral
head blood supply is usually preserved so the treatment of choice would be to reduce the fracture and fix
it with a dynamic hip screw.

Scenario 7
A 37-year-old man twists his leg and falls over whilst skiing. His X-ray shows a spiral fracture of the midshaft of his Page |
left tibia. 1384
G - Hemiarthroplasty« YOUR ANSWER
H - Intramedullary nail« CORRECT ANSWER.

An intramedullary nail is used to treat midshaft fractures of the femur and tibia. This allows optimal
stabilisation and early mobilisation.

74. Theme: Orthopaedics


A Medial plantar nerve
B Femoral nerve
C Common peroneal nerve
D Deep peroneal nerve (old name anterior tibial nerve)
E Saphenous nerve.

For each of the scenarios match the appropriate nerve causing the palsy. Each option may be used once,
more than once, or not at all.

Scenario 1
Compartment syndrome with paraesthesia of the first web space.
A - Medial plantar nerve« YOUR ANSWER
D - Anterior tibial nerve« CORRECT ANSWER.

The deep peroneal nerve (old name anterior tibial nerve) supplies the first toe cleft. The skin of the sole of
the foot is supplied by the medial and lateral plantar nerves. The plantar surfaces of the toes are supplied
by the digital branches of the medial (three and a half) and lateral (one and a half) plantar nerves. They
also supply the skin on the dorsum of each toe proximal to the nail bed.

Scenario 2
Paraesthesia of the ball of the big toe.
B - Femoral nerve« YOUR ANSWER
A - Medial plantar nerve« CORRECT ANSWER.

The deep peroneal nerve (old name anterior tibial nerve) supplies the first toe cleft. The skin of the sole of
the foot is supplied by the medial and lateral plantar nerves. The plantar surfaces of the toes are supplied
by the digital branches of the medial (three and a half) and lateral (one and a half) plantar nerves. They
also supply the skin on the dorsum of each toe proximal to the nail bed.

The deep peroneal nerve (old name anterior tibial nerve) supplies the first toe cleft. The skin of the sole of
the foot is supplied by the medial and lateral plantar nerves. The plantar surfaces of the toes are supplied
by the digital branches of the medial (three and a half) and lateral (one and a half) plantar nerves. They
also supply the skin on the dorsum of each toe proximal to the nail bed.
---------------------------------------------------------------------------------------------------------------------------------------------------------------------------------------------------------------
Dr Mohammed Shamsul Islam Khan, Medical Officer, Clinical Neuro-Surgery, National Institute of Neuro-Sciences and Hospital
Sher-E-Bangla Nagar, Dhaka-1207, Bangladesh. Mobile: +880 1713 455 662, +880 1685 811979. E-mail: drsikhan@gmail.com
MyPasTest: MRCS A Online - Jan Exam 2015
22. Surgical Specialities; Trauma & Orthopaedic Surgery (76Qs)
----------------------------------------------------------------------------------------------------------------------------------
75. Theme: Cutaneous sensation
A The median nerve
B The radial nerve
C The dorsal cutaneous branch of the ulnar nerve
D The superficial branch of the ulnar nerve
E The palmar cutaneous branch of the ulnar nerve. Page |
1385
For each of the following anatomical position descriptions, select the most likely answer from the above
list. Each option may be used once, more than once, or not at all.

Scenario 1
The sensory innervation of the nail bed of the index finger.
A - The median nerve« CORRECT ANSWER.

The median nerve supplies the lateral 3½ digits of the hand on both the anterior and posterior surfaces.

Scenario 2
The sensory innervation of the medial side of the palm.
B - The radial nerve« YOUR ANSWER
E - The palmar cutaneous branch of the ulnar nerve« CORRECT ANSWER.

The ulnar nerve enters the palm anterior to the flexor retinaculum, and as it crosses the flexor retinaculum
it divides into a superficial and a deep terminal branch. The palmar cutaneous branch of the ulnar nerve is
given off in the front of the forearm anterior to the flexor retinaculum, and supplies the skin over the
medial aspect of the palm.

Scenario 3
The sensory innervation of the dorsal surface of the base of the thumb.
C - The dorsal cutaneous branch of the ulnar nerve« YOUR ANSWER
B - The radial nerve« CORRECT ANSWER.

The superficial branch of the radial nerve descends over the extensor retinaculum and supplies the lateral
two-thirds of the dorsum of the hand. It divides into a number of dorsal digital nerves which supply the
dorsal surface of the thumb, the dorsal surface of the index finger and dorsal surface of the lateral side of
the middle finger.

Scenario 4
The sensory innervation of the medial side of the palmar surface of the ring finger.
D - The superficial branch of the ulnar nerve« CORRECT ANSWER.

The superficial branch of the ulnar nerve descends into the palm and gives off a muscular branch to the
palmaris brevis, and cutaneous branches to the palmar aspect of the medial side of the little finger and
the adjacent sides of the little and ring fingers.

76. Theme: Painful conditions of the foot


A Claw toe
B Freiberg ’s disease
---------------------------------------------------------------------------------------------------------------------------------------------------------------------------------------------------------------
Dr Mohammed Shamsul Islam Khan, Medical Officer, Clinical Neuro-Surgery, National Institute of Neuro-Sciences and Hospital
Sher-E-Bangla Nagar, Dhaka-1207, Bangladesh. Mobile: +880 1713 455 662, +880 1685 811979. E-mail: drsikhan@gmail.com
MyPasTest: MRCS A Online - Jan Exam 2015
22. Surgical Specialities; Trauma & Orthopaedic Surgery (76Qs)
----------------------------------------------------------------------------------------------------------------------------------
C Gout
D Hallux rigidus
E Hallux valgus
F Hammer toe
G March fracture
H Morton’s metatarsalgia
I Pes cavus Page |
J Rheumatoid arthritis 1386
K Sever’s disease.

The following descriptions are of patients who have presented with painful conditions of the foot. Please
select the most appropriate diagnosis from the above list. The items may be used once, more than once,
or not at all.

Scenario 1
A 45-year-old woman presents with a history of a sharp pain over the dorsum of the foot, which radiates into her
toes. Examination reveals a fine point of tenderness in the cleft between the third and fourth toes.
A - Claw toe « YOUR ANSWER
H - Morton‟s metatarsalgia« CORRECT ANSWER.

H – Morton’s metatarsalgia:
The description is a classical presentation of this condition. The exact aetiology is unclear; it is thought
to occur following entrapment of a digital nerve between the metatarsal heads with secondary thickening
and formation of a neuroma. The nerve most commonly affected lies between the third and fourth
metatarsal heads. Pain is usually acute and may be associated with a sensory disturbance in the
distribution of the nerve. If symptoms are troublesome treatment is by excision of the neuroma.
YOUR ANSWER WAS INCORRECT
Scenario 2
A 25-year-old nurse asks for your opinion regarding her painful left foot. She qualified 5 months ago and has been
working in The Emergency Department since. The pain started 1 week ago and causes her to limp. On
examination she is tender over the second metatarsal, which feels unusually thick.
B - Freiberg‟s disease « YOUR ANSWER
G - March fracture« CORRECT ANSWER.

G – March fracture:
This is a metatarsal stress fracture, usually of the second or third metatarsal, which occurs in young
adults after a period of unaccustomed walking. The initial complaint is that of pain in the forefoot and the
affected bone feels thick and tender. The fracture may not be evident on initial X-rays of the foot but
eventually reveals itself by the appearance of abundant callus. The condition is self limiting with no long-
term sequelae. Treatment is symptomatic.

Scenario 3
A 42-year-old man complains of sudden onset of pain over the medial aspect of his right forefoot. Of note, he has
recently returned from a „stag‟ weekend in Dublin. On examination the base of his big toe is swollen,
erythematous, hot and tender to touch.
C - Gout« CORRECT ANSWER.

C – Gout:
---------------------------------------------------------------------------------------------------------------------------------------------------------------------------------------------------------------
Dr Mohammed Shamsul Islam Khan, Medical Officer, Clinical Neuro-Surgery, National Institute of Neuro-Sciences and Hospital
Sher-E-Bangla Nagar, Dhaka-1207, Bangladesh. Mobile: +880 1713 455 662, +880 1685 811979. E-mail: drsikhan@gmail.com
MyPasTest: MRCS A Online - Jan Exam 2015
22. Surgical Specialities; Trauma & Orthopaedic Surgery (76Qs)
----------------------------------------------------------------------------------------------------------------------------------
This condition is the commonest form of inflammatory joint disease in men over 40 with the first
metatarsophalangeal joint affected in over 90% of cases. The condition closely resembles septic arthritis;
however, there is an absence of systemic features of infection. The primary cause is hyperuricaemia and
there may be a precipitating cause such as alcohol excess or foods high in purines and diuretics.
Treatment during the acute phase is with anti-inflammatory drugs followed by a xanthine oxidase
inhibitor, eg allopurinol to prevent further episodes).
Page |
Scenario 4 1387
A 23-year-old woman presents with a history of pain in the sole of her foot and on the dorsal aspect of her second
toe. On examination there are callosities over the proximal interphalangeal joint of the second toe, which appears
flexed and under the second metatarsal head. The second metatarsophalangeal and distal interphalangeal joints
appear hyperextended.
D - Hallux rigidus « YOUR ANSWER
F - Hammer toe« CORRECT ANSWER.

F – Hammer toe:
The clinical findings are characteristic with pain occurring as a result of callosities forming over the
pressure areas. This should not be confused with claw toes where there is hyperextension of the
metatarsophalangeal joint and flexion of both the interphalangeal joints. If pain is severe, treatment is by
excision arthrodesis.

Scenario 5
A 10 year-old boy presents with pain in his right heel. On examination, the foot appears grossly normal, however
there is significant tenderness over the calcaneum close to the insertion of the Achilles tendon.
E - Hallux valgus « YOUR ANSWER
K - Sever‟s disease« CORRECT ANSWER.

K – Sever’s disease:
This condition is the most likely cause of a painful heel in a child and is a form of osteochondritis of the
calcaneal epiphysis. Pain and tenderness occur close to the insertion of the Achilles tendon. Radiographs
may demonstrate epiphyseal fragmentation or sclerosis. The condition is self limiting with symptoms
controlled by means of a pressure-relieving pad.

Editor:
Dr Mohammed Shamsul Islam Khan
MBBS (CMC); FCPS-II (Neuro-Surgery)
Medical Officer, Clinical Neuro-Surgery
National Institute of Neuro-Sciences and Hospital
Sher-E-Bangla Nagar, Dhaka-1207, Bangladesh.
Mobile: +880 1713 455 662, +880 1685 811979.
E-mail: drsikhan@gmail.com

---------------------------------------------------------------------------------------------------------------------------------------------------------------------------------------------------------------
Dr Mohammed Shamsul Islam Khan, Medical Officer, Clinical Neuro-Surgery, National Institute of Neuro-Sciences and Hospital
Sher-E-Bangla Nagar, Dhaka-1207, Bangladesh. Mobile: +880 1713 455 662, +880 1685 811979. E-mail: drsikhan@gmail.com

You might also like